You are on page 1of 176

Oculoplasty

for Ophthalmologists

Questions and Answers


Essam A. El Toukhy
Editor

123
Oculoplasty for Ophthalmologists
Essam A.  El Toukhy
Editor

Oculoplasty
for Ophthalmologists
Questions and Answers
Editor
Essam A. El Toukhy
Oculoplasty Service
Cairo University
Cairo, Egypt

ISBN 978-3-030-68468-6 ISBN 978-3-030-68469-3 (eBook)


https://doi.org/10.1007/978-3-030-68469-3

© The Editor(s) (if applicable) and The Author(s), under exclusive license to Springer Nature
Switzerland AG 2021
This work is subject to copyright. All rights are solely and exclusively licensed by the Publisher,
whether the whole or part of the material is concerned, specifically the rights of translation,
reprinting, reuse of illustrations, recitation, broadcasting, reproduction on microfilms or in any
other physical way, and transmission or information storage and retrieval, electronic adaptation,
computer software, or by similar or dissimilar methodology now known or hereafter developed.
The use of general descriptive names, registered names, trademarks, service marks, etc. in this
publication does not imply, even in the absence of a specific statement, that such names are
exempt from the relevant protective laws and regulations and therefore free for general use.
The publisher, the authors and the editors are safe to assume that the advice and information in
this book are believed to be true and accurate at the date of publication. Neither the publisher
nor the authors or the editors give a warranty, expressed or implied, with respect to the material
contained herein or for any errors or omissions that may have been made. The publisher remains
neutral with regard to jurisdictional claims in published maps and institutional affiliations.

This Springer imprint is published by the registered company Springer Nature Switzerland AG
The registered company address is: Gewerbestrasse 11, 6330 Cham, Switzerland
Preface

This book presents the most up-to-date oculoplastic knowledge base in a


question and answer format. Multiple choice questions are the most com-
monly used assessment and teaching technique, and this question format
will continue to assume a greater significance in the era of online exams and
virtual consultations.
This book is designed to target ophthalmologists at varying professional
levels; from residents and fellows to consultants. Whether they are prepar-
ing for an exam, sitting for a final degree, pursuing subspecialty training
in oculoplasty, or requiring a detailed knowledge base of the subspecialty
as they encounter clinical cases, this book provides a comprehensive guide
from the very basic principles to the most advanced. The book will also be
of special interest for consultant ophthalmologists and university professors
organizing and preparing exams.
The book follows the same structure of the accompanying textbook
“Oculoplastic Surgery—A Practical Guide to Common Disorders.” Section
by section, it includes over 1000 classic multiple choice questions as well
as 150 high-quality clinical photos and illustrations with a variety of case
presentations, clinical scenarios, radiological scans, and pathological speci-
mens. The questions are meant to supplement the information provided in
the textbook, which is available for further clarification and in-depth study
of the various subjects. This supplementary approach and extensive illustra-
tive support makes this book series unique in the world of oculoplasty (at
present, no high-quality referenced question books are available at interna-
tional level).
We hope this book becomes a staple on every ophthalmologist’s desk as
they pursue the specialty and encounter oculoplasty cases in their practice.

Cairo, Egypt Essam A. El Toukhy

Acknowledgements To all those who supported me during my life, to all the wonderful
people in my life, my dear parents, my lovely wife, and my beloved daughters.

v
Contents

1 Basics of Oculoplasty and Anaesthesia. . . . . . . . . . . . . . . . . . . . . . . . 1


Essam A. El Toukhy
2 Lid Lesions and Malpositions. . . . . . . . . . . . . . . . . . . . . . . . . . . . . . . 11
Essam A. El Toukhy
3 Ptosis. . . . . . . . . . . . . . . . . . . . . . . . . . . . . . . . . . . . . . . . . . . . . . . . . . . 37
Essam A. El Toukhy
4 Lid Reconstruction . . . . . . . . . . . . . . . . . . . . . . . . . . . . . . . . . . . . . . . 51
Essam A. El Toukhy
5 Cosmetics and Injectables. . . . . . . . . . . . . . . . . . . . . . . . . . . . . . . . . 59
Noha El Toukhy
6 The Lacrimal System . . . . . . . . . . . . . . . . . . . . . . . . . . . . . . . . . . . . . 69
Nadeen El Toukhy
7 Trauma. . . . . . . . . . . . . . . . . . . . . . . . . . . . . . . . . . . . . . . . . . . . . . . . . 91
Essam A. El Toukhy
8 The Orbit. . . . . . . . . . . . . . . . . . . . . . . . . . . . . . . . . . . . . . . . . . . . . . 101
Essam A. El Toukhy
9 Orbital Implants and Prosthesis . . . . . . . . . . . . . . . . . . . . . . . . . . . 135
Essam A. El Toukhy
10 Oculoplasty Interactions with Other Specialities. . . . . . . . . . . . . . 143
Essam A. El Toukhy
11 Thyroid Eye Disease . . . . . . . . . . . . . . . . . . . . . . . . . . . . . . . . . . . . . 159
Essam A. El Toukhy

vii
Basics of Oculoplasty
and Anaesthesia 1
Essam A. El Toukhy

Oculoplastic surgery is the subspecialty that management, with tailored approaches of local,
combines the art and principles of plastic and regional and general anaesthesia techniques.
reconstructive surgery with the delicacy and Anaesthetic management for oculoplastic sur-
precision of ophthalmic surgery. An oculoplas- geries mainly requires a thorough knowledge of
tic surgeon should be aware of the principles the anatomy as well as local anaesthetic pharma-
of both worlds as well as surgical skills to get cology. Regional blocks have gained widespread
optimum cosmetic and functional results while enthusiasm and are being used more and more
protecting the globe and the patient’s vision. A frequently now. They can be used alone or in
thorough knowledge of wound healing, the types combination with each other to cover the sur-
of sutures, needles, flaps and grafts is mandatory gery site. They cause minimal discomfort, lower
to gain a cosmetically accepted result. cost, and lower perioperative morbidity in com-
Similarly, anaesthesia is an indispensa- parison to general anaesthesia. They also pro-
ble component of Oculoplastic procedures. vide the advantages of less local anaesthetic use
As a subspecialty, oculoplasty has its own and minimal tissue distortion when compared
needs and requirements regarding anaesthesia with infiltration anaesthesia.

E. A. El Toukhy (*)
Oculoplasty Service, Cairo University, Cairo, Egypt
e-mail: eeltoukhy@yahoo.com

© The Author(s), under exclusive license to Springer Nature Switzerland AG 2021 1


E. A. El Toukhy (ed.), Oculoplasty for Ophthalmologists, https://doi.org/10.1007/978-3-030-68469-3_1
2 E. A. El Toukhy

Basics of Oculoplasty and Anaesthesia

2. The following is a:

1. The following is a: A. Vertical mattress suture


B. Horizontal mattress suture
A. Vertical mattress suture C. Interrupted suture
B. Horizontal mattress suture D. Continuous suture.
C. Interrupted suture
D. Continuous suture.
1 Basics of Oculoplasty and Anaesthesia 3

3. The following is a:
A. Vertical mattress suture
B. Horizontal mattress suture
C. Interrupted suture
D. Continuous suture. 4. The above diagram is an example of:
A. Advancing flap
B. Rotational flap
C. Rhomboid flap
D. Transpositional flap.
4 E. A. El Toukhy

6. The above diagram is an example of:


A. Advancing flap
B. Rotational flap
C. Rhomboid flap
D. Transpositional flap.

5. The above diagram is an example of:


A. Advancing flap
B. Rotational flap
C. Rhomboid flap
D. Transpositional flap.
1 Basics of Oculoplasty and Anaesthesia 5

7. The above diagram is an example of: 10. The following are true about the nasociliary
nerve EXCEPT:
A. Advancing flap
B. Rotational flap A. Gives off supratrochlear nerve which
C. Rhomboid flap innervates the medial forehead
D. Transpositional flap. B. It supplies the lateral wall of the nose
C. Innervates the cornea
D. Carries within it the sympathetic fibers
from the internal carotid plexus.
11. The long ciliary nerve:
A. Enters the globe at the equator
B. Contains parasympathetic nerve fibres
C. Synapse at the ciliary ganglion
D. Contains sensory fibres from the cornea.
12. The following is TRUE about the superior
ophthalmic vein:
A. It is the main venous channel of the orbit
B. It is formed by the union between the
facial vein and the temporal vein
C. It passes backward in the orbit between
the levator and the superior rectus muscle
D. It does not receive the central retinal vein.
13. The peripheral arterial arcade in the upper
eyelid is present:
A. 3 mm above the eyelid margin
B. Along the anterior surface of the tarsus
C. Between the levator aponeurosis and
Muller’s muscle
D. Between the orbicularis oculi muscle
and levator aponeurosis.
8. The following technique can be useful in
the management of all except: 14. All of the following structures attach to the
Whitnal’s tubercle except
A. Scar revision
B. Cicatricial ectropion A. Superior transverse ligament of the orbit
C. Cicatricial entropion B. Aponeurosis of the levator palpebrae
D. Eyelid Webbing. superioris muscle
C. Suspensory ligament of the eyeball
9. The following are true EXCEPT: D. Lateral check ligament of the inferior
A. The supraorbital ridge extends only over oblique muscle.
the medial one half to two thirds of the 15. All of the following structures pass through
superior orbital rim the superior orbital fissure, except
B. The frontalis muscle of the forehead sup-
ports medial two thirds of the eyebrow A. Sympathetic nerve fibers
C. The sensory nerves to the forehead B. Superior ophthalmic vein
travel on the underside of the frontalis C. Trochlear nerve
D. The supratrochlear nerve supplies most D. Zygomatic nerve.
of the sensation of the forehead.
6 E. A. El Toukhy

16. What tissue plane is the temporal branch of 21. Which statement concerning the medial
the facial nerve located in, superior to the canthal area is true?
zygomatic arch?
A. All of the attachments anchoring the
A. Deep temporal fascia tarsi to the medial orbital wall lie ante-
B. Loose areolar tissue rior to the lacrimal sac and attach to the
C. Subcutaneous tissue maxillary portion of the frontal bone
D. Temporoparietal fascia. B. The lacrimal sac lies posterior to the
orbital septum
17. The gray line of the eyelid margin is formed
C. The muscle pump of the lacrimal pump
by
mechanism is innervated by the fifth
A. Meibomian glands cranial nerve
B. Tarsal border D. Lockwood ligament attaches posterior
C. Mucocutaneous junction to the lacrimal sac.
D. Orbicularis muscle.
22. Which statement regarding fat encountered
18. The capsulopalpebral fascia is analogous to during eyelid surgery is false?
which upper eyelid structure?
A. Preaponeurotic fat is orbital fat
A. Levator aponeurosis B. Extraconal orbital fat is an important
B. Orbital septum landmark in identifying the levator
C. Superior transverse ligament aponeurosis
D. Muller’s muscle. C. The removal of fat from the upper eye-
lid nasal, central, and lateral fat pads
19. The normal horizontal measurement of the
may be done with impunity
palpebral fissure is approximately
D. In the upper eyelid, the nasal fat pad is
A. 20 mm small, whereas the lateral fat pad is the
B. 25 mm small fat pad in the lower eyelid.
C. 30 mm
23. Which statement regarding Whitnall ­ligament
D. 35 mm.
(superior transverse ligament) is false?
20. Which statement about the orbital septum is
A. Whitnall ligament attaches medially
false?
to the trochlea, laterally to the capsule
A. During entropion repair, it is very of the lacrimal gland, and to the lateral
important to recognize the orbital sep- orbital wall
tum of the lower eyelid as being differ- B. This ligament is a condensation of the
ent from the aponeurosis or lower eyelid sheath of the levator muscle and serves
retractors as a check ligament to prevent excessive
B. The orbital septum arises from a con- elevation of the eyelid
densation of the periosteum of the C. Whitnall ligament acts to change the
orbital rim called the arcus marginalis direction of pull of the levator muscle
C. The orbital septum inserts on the supe- from horizontal to vertical
rior border of the tarsus in the upper D. This ligament passes anterior to the lac-
eyelid rimal gland.
D. The orbital septum serves as a barrier to
the spread of infection from the superfi-
cial eyelids to the orbital tissues.
1 Basics of Oculoplasty and Anaesthesia 7

24. Which statement about eyelid anatomy is C. Inferior orbital fissure-maxilla, zygo-
false? matic bone, palatine bone, and greater
wing of the sphenoid bone
A. The gray line is formed by the muscle
D. Anterior and posterior ethmoidal fora-
of Riolan and represents the observable
men-ethmoid and frontal bones.
edge of the pretarsal orbicularis at the
eyelid margin 28. All of the following statements concerning
B. The posterior lamella of the eyelid con- lymphatic and venous drainage are true except:
sists of the conjunctiva and tarsus
A. Lymphatic vessels of the orbit drain along
C. The mucocutaneous junction occurs where
the lateral portion of the cavernous sinus
the eyelashes emerge from the eyelid
B. Lymphatic vessels serving the medial
D. The peripheral and marginal arterial
portion of the upper eyelid drain into
arcades allow for anastomosis between
submandibular lymph nodes
the internal and external carotid systems.
C. Lymphatic vessels serving the lateral
25. Features of the orbicularis muscle include: portions of the upper eyelid drain into
preauricular nodes
A. Closure of the eyelid, depression of the
D. Pretarsal venous drainage of the medial
eyebrow, and facilitation of tear drainage
upper eyelid is into the angular vein and
B. Pretarsal orbicularis inserts temporally to
the lateral venous drainage is into the
become the lateral canthal tendon, ontrac-
superficial temporal vein system.
tion narrows the palpebral fissure, and the
orbital portion of the muscle inserts medi- 29. Regarding the orbital septum, which is
ally on the posterior lacrimal crest incorrect:
C. The deep head of the medial pretarsal
A. Is separated from the levator aponeuro-
muscle is called Homer tensor tarsi and
sis by orbital fat
innervation of the orbicularis muscle by
B. Is firmly attached to Whitnall’s ligament
cranial nerve III is divided into three seg-
C. Fuses with the capsulopalpebral fascia
ments (pretarsal, preseptal, and orbital)
in the lower lid
D. The zygomaticofacial nerve innervates
D. Inserts on the levator aponeurosis about
the upper lid orbicularis, the frontal
3 to 5 mm above the tarsal plate.
branch of cranial nerve VII sends motor
fibers to the upper lid orbicularis, and 30. Regarding the tarsal plates, which is
the preseptal orbicularis divides to incorrect:
encompass the lacrimal gland.
A. Of the upper lid are about 10 mm in
26. Which one of the following muscle groups height
is paired incorrectly? B. Of the lower lid are about 8 mm in
height
A. Tensor tarsi muscle-deep head of the
C. Impart structural integrity to the eyelids
pretarsal orbicularis
D. Do not contain lash follicles.
B. Nasalis-preseptal orbicularis
C. Superciliary corrugator muscle-orbital 31. Regarding the orbital floor is, which is
orbicularis incorrect:
D. Frontalis-procerus muscle.
A. Composed primarily of the maxillary
27. Which structure and its bony framework are bone
paired incorrectly? B. Composed of the zygomatic and pala-
tine bones
A. Lacrimal sac fossa-lacrimal and maxil-
C. Separated from the lateral wall by the
lary bones
inferior orbital fissure
B. Optic canal-greater and lesser wings of
D. The largest of the orbital walls, running
the sphenoid bone
to the orbital apex.
8 E. A. El Toukhy

32. Regarding the medial orbital wall, which is 33. Regarding the ophthalmic artery, which is
incorrect: incorrect:
A. Contains the frontal process of the max- A. Crosses over the optic nerve in 85% of
illary bone individuals
B. Contains the optic foramen B. Enters the orbit through the optic canal
C. Is composed of the sphenoid bone and C. Gives off the lacrimal artery as its first
the lacrimal bone. orbital branch
D. Is composed largely of the ethmoid D. Gives off the central retinal artery which
bone. runs under the optic nerve.

34. The above nerve block is indicated in sur- 36. Lidocaine is:
geries on all except:
A. An amide anesthetic
A. Lower eyelid B. Has a rapid onset of action
B. Lower canaliculus C. Is an intermediate acting drug
C. DCR D. Has low systemic toxicity.
D. Lower lid entropion.
37. Phantom eye syndrome can be prevented
35. Malignant hyperthermia occurs mostly in by:
all except:
A. General anesthesia
A. Children B. Performing evisceration rather than
B. Adults enucleation
C. Ptosis C. Removing a long stump of the optic
D. Strabismus. nerve
D. Insertion of an orbital implant.
1 Basics of Oculoplasty and Anaesthesia 9

38. All the following are phases of wound heal- 45. Pain during local anesthesia injection can
ing except: be reduced by;
A. Inflammatory phase A. Slow injection
B. Scarring phase B. Smaller needles
C. Proliferation phase C. Addition of bicarbonate
D. Remodeling phase. D. Needle-free jet injections.
39. Scar formation is influenced by all except: 46. Disadvantages of local infiltration anesthe-
sia during ptosis surgery include all except:
A. Site
B. Age A. Hematoma formation
C. Sex B. Stimulation of Muller muscle by
D. Skin type. epinephrine
C. Diffusion of the anesthetic to orbicularis
40. A bad scar can be due to all except:
muscle
A. Smoking D. Diffusion of the anesthetic to the levator
B. Poor surgical technique muscle.
C. Wound tension
47. Reflex sneezing while injecting local anes-
D. Undermining of surrounding tissues.
thesia (sternutatory reflex) is mediated by:
41. Monofilament sutures are preferable in:
A. Infraorbital nerve
A. Tarsal suturing B. Supraorbital nerve
B. Muscle suturing C. Nasociliary nerve
C. Skin suturing D. Lacrimal nerve.
D. Tendon suturing.
Answers of Basics of Oculoplasty and
42. 1/2 circle needles are used in
Anaesthesia
A. Skin closure to reduce scarring
B. Ptosis surgery to suture the levator 1 C 13 C 25 A 37 B
aponeurosis to the tarsus 2 A 14 D 26 B 38 B
C. DCR surgery for closure of the posterior
3 B 15 D 27 B 39 C
flaps
4 A 16 D 28 A 40 D
D. Brow pexy procedures.
5 B 17 D 29 B 41 C
43. Graft “take” means occurrence of:
6 C 18 A 30 B 42 C
A. Fibroblast proliferation 7 D 19 C 31 D 43 C
B. Fibrin deposition 8 C 20 C 32 B 44 D
C. Vascularization 9 D 21 D 33 C 45 D
D. Neurotization.
10 A 22 C 34 D 46 C
44. In lid reconstruction; one can use all except: 11 D 23 D 35 B 47 C

A. A flap for the anterior lamella and a flap 12 A 24 C 36 D


for the posterior lamella
B. A flap for the anterior lamella and a
graft for the posterior lamella
C. A graft for the anterior lamella and a
flap for the posterior lamella
D. A graft for the anterior lamella and a
graft for the posterior lamella.
Lid Lesions
and Malpositions 2
Essam A. El Toukhy

Introduction The majority of the lid lesions are benign, but


their identification is important for proper treat-
Various lesions can be detected in the eyelid due ment and to rule out malignancy.
to its diverse composition. The skin epidermis is The main goal of the ophthalmologist is to
keratinized stratified squamous epithelium while exclude malignancy. Certain points in history
its dermis contains cilia in addition to modified taking and clinical examination help to rule out
sweat and sebaceous glands. The tarsus also malignant lesions.
contains Meibomian glands which are modified Benign lesions are usually uniform with
sebaceous glands while the lining conjunctiva regular borders and show slow growth. They
contains accessory lacrimal glands and goblet usually don’t show induration, ulceration or lid
cells. margin destruction and can be classified accord-
Skin adnexa including sebaceous and sweat ing to;
glands as well as hair follicles are placed in the
dermis and can give an origin to a wide variety – Structure of origin to epidermal, dermal or
of, usually, benign lesions. adnexal.
The sebaceous glands of the eye lid include; – Clinical appearance either solid or cystic.
the Meibomian gland of the tarsus, glands of – Location whither related to the lid margin,
Zeis that are related to the eye lashes and the pretarsal area or supra/infra tarsal region.
sebaceous glands related to hair of the eyelid
skin as well as the hair of the eye brow. On the other hand, features suspicious of malig-
The sweat glands of the eyelid are either nancy include:
eccrine glands (that have a true secretory duct)
that are present everywhere in the body skin • Recent onset.
including the eyelid or apocrine glands (that • Increasing in size.
have no duct and secrete by cellular decapita- • Change in color or multiple colors.
tion) that are present in relation to eyelashes and • Ulceration.
known as glands of Moll. • Telangiectasia.
• Pearly borders.
• Ill-defined margins.
• Distorted anatomy e.g. loss of lashes, dis-
E. A. El Toukhy (*) torted lid margin.
Oculoplasty Service, Cairo University, Cairo, Egypt • Recurrent lesion e.g. recurrent chalazion.
e-mail: eeltoukhy@yahoo.com

© The Author(s), under exclusive license to Springer Nature Switzerland AG 2021 11


E. A. El Toukhy (ed.), Oculoplasty for Ophthalmologists, https://doi.org/10.1007/978-3-030-68469-3_2
12 E. A. El Toukhy

• Pain disproportional to the lesion i.e. perineu- adjuvant, or instead of, surgical excision in some
ral spread. cases.
• History of irradiation e.g. for acne, The eyelids are the primary defense of the
retinoblastoma. eye against dryness, exposure, and trauma.
• History of other malignancies. Therefore, proper lid positioning is important
• Immunosuppression. to ocular health. Lid malpositions are among
the most common problems encountered by
Generally, the clinical appearance is highly sug- the ophthalmologist. Visual loss may occur in
gestive of the lesion nature yet, when in doubt, these conditions due to keratopathy secondary to
a biopsy is required to confirm the diagnosis. exposure or lashes rubbing on the ocular surface.
Biopsies are either incisional which entails A thorough understanding of the anatomy,
removal of a part of the lesion or excisional in pathophysiology, appropriate evaluation, and
which the lesion is totally removed thus, addi- treatment options of these lid malpositions is
tionally provides a cure. essential for the practicing ophthalmologist.
Treatment options in general include total The classification of lid malpositions is based
excision of the lesion, with special attention to according to their respective etiologies. There
removal of the walls in case of cysts, marsupi- are five main types of ectropion: involutional,
alization i.e. removal of the top of the cyst if paralytic, mechanical, cicatricial, and congeni-
excision is not feasible and surface ablation in tal. Entropion is subdivided into 4 categories:
superficial lesions. involutional, acute spastic, cicatricial, and con-
The goals of therapy for periocular lid malig- genital. Facial nerve affection can result in more
nancy are threefold: to completely excise the than one type of lid malpositions. Lid retraction,
tumor; to maintain the integrity and the func- Centurion syndrome, Floppy eyelids syndrome
tion of the eye; and to achieve a good cosmetic are less common lid malpositions seen in clini-
result. cal practice.
It may be difficult to accomplish all of these The preoperative evaluation is essential for
objectives in every patient or by one surgery. determining the etiology of the lid malposition
The dilemma of removal of the tumor while pre- and deciding on the surgical procedure neces-
serving normal tissue is more challenging in the sary for correction of the malposition.
periocular area than it is on other areas on the The goal of a successful surgical repair
skin. includes a good apposition of the lid mar-
Lid reconstruction should aim at restora- gin to the globe, corneal irritation symp-
tion of normal lid anatomy with replacement of toms relief, good cosmetic outcome with
defect in the anterior and/or posterior lamella lasting results, while addressing the underlying
using the appropriate reconstructive surgical pathophysiology.
technique, individualized for each case. Being the most common cause of infectious
Radiotherapy, Photodynamic therapy, cryo- blindness and a leading cause of lid lesions and
therapy, Topical immunotherapy, topical and malpositions globally, trachoma deserves a spe-
systemic chemotherapy can all be used as an cial emphasis.
2 Lid Lesions and Malpositions 13

Lid Lesions

3. The above figure is


A. Cyst of Moll
B. Molluscum contagiousum
C. Hidrocystoma
D. Keratoacanthoma.

1. The above figure is


A. Sebaceous cyst
B. Molluscum contagiousum
C. Nevus
D. Keratoacanthoma.

4. The above figure is


A. Cyst of Moll
B. Molluscum contagiousum
C. Hidrocystoma
D. Keratoacanthoma.
2. The above figure is;
A. Junctional nevus
B. Compound nevus
C. Congenital nevus
D. Kissing nevus.
14 E. A. El Toukhy

7. The above lesion can be treated by all


except:
A. Observation
5. The above figure is B. Steroids
A. Sebaceous cyst C. B blocker
B. Molluscum contagiousum D. Surgery.
C. Seborrheic keratosis
D. Keratoacanthoma.

6. The above lesion is best treated by:


A. Observation
B. Radiofrequenc
C. Laser
D. Surgery.
2 Lid Lesions and Malpositions 15

8. The most likely diagnosis of the above


lesion is:
A. Sebaceous cell carcinoma
B. Basal cell carcinoma
C. Squamous cell carcinoma
D. Amelanotic malignant melanoma.

9. The most likely diagnosis of the above


lesion is:
A. Sebaceous cell carcinoma
B. Basal cell carcinoma
C. Squamous cell carcinoma
11. Features suggesting malignancy in this
D. Amelanotic malignant melanoma.
lesion include all except:
A. Loss of lashes
B. Short history
C. Recurrence after previous excision
D. Telangiectacic vessels.

10. The most likely diagnosis of the above


lesion is: 12. The likely diagnosis of the above lesion
A. Sebaceous cell carcinoma includes all except:
B. Basal cell carcinoma A. Pigmented Sebaceous cell carcinoma
C. Squamous cell carcinoma B. Pigmented Basal cell carcinoma
D. Amelanotic malignant melanoma. C. Pigmented Squamous cell carcinoma
D. Malignant melanoma.
16 E. A. El Toukhy

17. In treatment of chalazion:


A In the acute phase topical antibiotics is
recommended
B. Systemic doxycycline is used in acute
secondary infection for short time
C. Surgical incision for chronic cystic
chalazion is recommend
D. Histopathology is performed on excised
chalazion for the possibility of malig-
nant transformation of chalazion.

18. One of the following signs suggest eyelid


malignancy:
13. Management of the lacrimal drainage sys- A. Superficial vascularization
tem in this patient with a malignant lid B. Hypopigmentation
lesion in a young patient is: C. Painful lesions
A. Preservation of the lower canaliculus D. Fast growing.
B. Excision of the lower canaliculus with-
out intubation 19. Regarding lid pigmentary lesions, the most
C. Excision of the lower canaliculus with appropriate statement is:
intubation A. Nevi are the least common
D. Excision of the lower canaliculus with B. Freckle are result of hyperpigmentation
placement of Jones tube. of the basal layer of epidermis
C. Malignant transformation is common
14. The Breslow scale: with compound nevi
A. Is a measurement of the thickness of D. Blue nevus has no potential malignant
cutaneous malignant melanoma transformation.
B. Highly correlates with prognosis
C. The prognosis is poor if more than 2 mm 20. The most common precancerous lesion is:
D. Is incorporated in the TNM staging. A. Solar lentigo
B. Dermal melanocytosis
15. The Clark scale: C. Actinic keratosis
A. Is a histopathological scale used to D. Keratoacanthoma.
grade malignant melanoma
B. Depends on the level of invasion of the 21. A 52 years old white male presenting with
tumor cells lower lid ulcerating lesion near the medial
C. The higher the scale, the worse the canthus. Family history is positive for xero-
prognosis derma pigmentosa. The most likely diagno-
D. Is incorporated in the TNM staging. sis of this lesion is:
A. Squamous cell carcinoma
16. The treatment of choice for keratocanthoma B. Basal cell carcinoma
is: C. Melanoma
A. Observation D. Sebaceous cell carcinoma.
B. Steroid injection
C. Incisional biopsy followed by complete 22. In patients with Bowen disease:
surgical excision A. Histopathology shows limitation of the
D. Cryotherapy. disease to dermal layer of the skin
2 Lid Lesions and Malpositions 17

B. Cryotherapy is good treatment modality 28. A 45 year old man presents with rapidly
for the skin lesions enlarging mass below the eyelid margin the
C. 50% can progress to squamous cell lesion has a central crater with an elevated
carcinoma rolled edge. The most likely diagnosis is:
D. Skin lesions are rapidly increasing in A. Epidermal inclusion cyst
size. B. Keratoacanthoma
C. Verruca vulgaris
23. Regarding keratoacanthoma one of the fol- D. Pilomatricoma.
lowing statement is true:
A. Considered a benign lesion 29. Apocrine hidrocystoma is
B. Rapidly progressing in size A. Considered a true adenoma
C. Typically occurs in young adults B. Deep cyst requires marsupialization
D. Excisional biopsy is not required. C. It’s also known as cylindromas
D. Histopathologically it is squamous
24. Regarding basal cell nevus syndrome cystic structure containing keratin.
(Gorlin-Goltz syndrome), which is incorrect:
A. Is inherited as an autosomal dominant 30. A 70-year-old patient presents with a his-
trait tory of a painless, progressively enlarging
B. Includes jaw cysts mass in the central aspect of the right upper
C. Includes mental retardation lid. On examination, there is some distor-
D. Generally appears before age 10 years. tion of the eyelid margin and loss of lashes.
The most likely diagnosis is:
25. The least appropriate statement regarding A. Basal cell carcinoma
basal cell carcinoma is: B. Sebaceous gland carcinoma
A. Head and neck account for 90% of cases C. Squamous cell carcinoma
B. 10% of head and neck cases involve D. Amelanotic melanoma.
eyelids
C. Usually adults between 50-80 years of 31. Squamous cell carcinoma of the eyelid, one
age is false:
D. 15% of eyelid cases in patients under A. It is 40 times less common than basal
35 years. cell carcinoma
B. It is more aggressive than basal cell
26. The least likely Indications for removing carcinoma
nevus is: C. Surgical excision with wide margin is
A. Acquired lesion preferred
B. Congenital lesion D. Only metastasize through blood borne
C. Irritation induced area transmission.
D. Sun exposed area.
32. Which of the following pairs of eyelid
27. Regarding sebaceous adenocarcinoma: lesions and their histological features is
A. It can arise from eyelid skin sebaceous FALSE?
glands A. Basal cell carcinoma—peripheral pali-
B. More common in males sading nuclei
C. Lower lid is more frequently involved B. Squamous cell carcinoma—keratin pearls
D. Regional lymph nodes involvement C. Keratoacanthoma—hypokeratosis
is sentinel lymph node biopsy is not D. Sebaceous cell carcinoma—pagetoid
recommended. spread.
18 E. A. El Toukhy

33. Which of the following eyelid tumors is B. Systemic doxycycline is used in acute
NOT an indication for sentinel lymph node secondary infection for short time
biopsy? C. Surgical incision for a chronic cystic
A. Sebaceous cell carcinoma chalazion is recommended
B. Malignant melanoma D. A horizontal incision is recommended.
C. Basal cell carcinoma
D. Squamous cell carcinoma. 39. An elderly female presented with recur-
rent swelling of the upper eyelid.
34. A 60 years old patient presented with large Histopathological evaluation revealed it to
upper lid lesion of 6 months duration, lid be a chalazion. What would be the histo-
margin irregularities was seen with loss of pathological finding?
eyelashes overlying the lesion.The best next A. Lipogranuloma
step will be; B. Suppurative granuloma
A. Excision and drainage of the chalazion C. Foreign body granuloma
B. Initial treatment with topical antibiotics D. Xanthogranuloma.
C. Excisional biopsy
D. Orbital CT. 40. All of the following are true regarding
chalazion, except:
35. All of the followings are correct for squa- A. Sebaceous cyst
mous cell carcinoma of the eyelids except: B. It is due to staphylococcal infection
A. Is more common in the lightly pig- C. Recurrence may imply malignancy
mented individuals than in highly pig- D. Occlusion of the meibomian gland.
mented ones
B. May occur in scar tissue of highly pig- 41. Treatment of chalazion includes:
mented individuals A. Incision and drainage
C. Does not arise from actinic lesions B. Intralesional steroid
D. Is associated with psoralen plus UV-A C. Pressure bandage
light therapy for psoriasis. D. Antibiotics.

36. What is the sebaceous cell carcinoma’s 42. A recurrent chalazion should be subjected
response to radiation therapy? to histopathological examination to rule out
A. Very susceptible when used as an the possibility of
adjunct to surgery A. Squamous cell carcinoma
B. Responsive when combined with photo- B. Sebaceous cell carcinoma
dynamic agents C. Malignant melanoma
C. Relatively radio-resistant D. Basal cell carcinoma.
D. Needs multiple sessions.
43. All of the following are true about BCC,
37. Which one of the following is NOT a fea- except:
ture of basal cell carcinoma? A. Spread to the regional lymph nodes
A. Pearly elevated margins occurs late
B. Spread to regional lymph nodes B. Occurs more frequently in immunosup-
C. Ulcerated epithelium pressed individuals
D. Telangiectatic vessels. C. Complete surgical excision is advised
D. The lesion may involute over several
38. In treatment of chalazion: months.
A. In the acute phase topical antibiotics are
recommended
2 Lid Lesions and Malpositions 19

44. What is the most appropriate initial step in 49. A 14-years-old patient presents with a left
the management of a suspicious lesion on upper eyelid lesion. Histopathology of the
the lid margin of a 50 years old male? lesion showed shadow cells and areas of
A. Incision and curettage calcification surrounded by basophilic cells.
B. Observation All of the following are true of the patient’s
C. Full-thickness excisional biopsy condition, except:
D. Incisional biopsy. A. Young adults are most often affected
B. The lesion is epithelial in origin
45. Which of the following papillomatous C. Surgical excision of the lesion is
lesions of the eyelid is premalignant? curative
A. Acanthosis nigricans D. The eyebrow is also a common site of
B. Actinic keratosis involvement.
C. Seborrheic keratosis
D. Verruca vulgaris. 50. A recurrent basal cell carcinoma extending
deeply in the lateral orbit requires which
46. Which of the following papillomatous treatment:
lesions of the eyelid may be associated with A. Orbital exenteration
underlying systemic malignancy? B. Full-thickness pentagonal wedge
A. Acanthosis nigricans resection
B. Verruca vulgaris C. Wide excision with cryotherapy
C. Ephelis D. Radiation therapy.
D. Actinic keratosis.
51. A patient with sebaceous carcinoma of the
47. All of the following are true regarding seba- eyelid presents with an enlarged subman-
ceous carcinoma, except: dibular lymph node, which of the follow-
A. The primary focus may be either eyelid ing is most likely to be the location of this
or caruncle patient’s eyelid neoplasm?
B. Shave biopsy techniques are adequate A. Medial, lower eyelid
C. The hallmarks of the histopathology B. Lateral, lower eyelid
of the condition include skip areas and C. Medial, upper eyelid
pagetoid D. Lateral, upper eyelid.
D. Recognition is often delayed due to mis-
diagnosis as benign eyelid inflammation. 52. Which one of the following is a feature of
basal cell carcinoma?
48. All of the following are true regarding A. Always has a predisposing precursor
malignant melanoma of eyelid skin, except: lesion
A. Lentigo maligna melanoma and nodular B. Possible spread to regional lymph nodes
melanoma are the most common forms C. Respects tissue planes
affecting the eyelid D. Telangiectatic vessels.
B. Nodular melanoma has the worst
prognosis
53. The following factors are all associated with
C. The factor of greatest prognostic signifi-
cutaneous cancers except:
cance is depth of invasion
A. Increased sun exposure
D. Like conjunctival melanosis, eye-
B. Increased age
lid melanoma may be controlled with
C. Red hair
cryotherapy.
D. Increased natural skin pigmentation.
20 E. A. El Toukhy

54. Features most consistent with a malignant D. Microscopically contain foamy


eyelid lesion include: histiocytes.
A. Tenderness, erythema, alteration in pig-
58. Which of the following statements does not
ment pattern
accurately describe a sebaceous cell adeno-
B. Disruption of tarsal architecture, raised
carcinoma lesion?
pearly margins, pruritus
A. The lower eyelid is more frequently
C. Lash loss, central ulceration, rapid growth
involved than the upper eyelid
D. Ipsilateral lymph node metastasis,
B. Radiation therapy is thought to be a
hyperkeratosis, dark pigmentation.
causative factor
C. Most arise from the meibomian glands
of the tarsus
D. The initial course is indolent and often
misdiagnosed.

59. Which of the following does not indicate a


poor prognosis for a sebaceous cell adeno-
carcinoma lesion?
A. Duration of symptoms less than 6 months
B. Infiltrative growth pattern
C. Moderate or poor differentiation
D. Lymphatic or vascular invasion.

60. Chronic unilateral blepharoconjunctivitis is


commonly a presenting sign of which one
of the following?
55. This lesion most likely represents: A. Squamous cell carcinoma
A. Nodular basal cell carcinoma B. Basal cell carcinoma
B. Morpheaform basal cell carcinoma C. Cutaneous melanoma
C. Nodular squamous cell carcinoma D. Sebaceous carcinoma.
D. Sebaceous adenocarcinoma.
61. Which of the following statements most
56. Features of a keratoacanthoma include all accurately describes the behavior and man-
of the following except: agement of congenital nevi?
A. Spontaneous resolution A. The risk of melanoma is directly related
B. Loss of eyelashes to the size of congenital nevi
C. Ulcerated crater filled with lipids B. Small congenital nevi do not need to be
D. Rapid growth. followed
C. Biopsy of congenital nevi is
57. Xanthelasma eyelid lesions have all the fol- contraindicated
lowing features except: D. Large congenital nevi require complete
A. Associated with systemic hyperlipi- surgical excision.
demic conditions in approximately 25%
of patients 62. A patient presents with lentigo maligna involv-
B. Located in the basal epithelial layer of ing the majority of the lower eyelid. What is
the skin the most appropriate management option?
C. Associated with the Erdheim-Chester A. Observation for thickening of the lesion
disease B. Radiation
2 Lid Lesions and Malpositions 21

C. Cryotherapy raising concern about sebacaeous cell carci-


D. Surgical excision with pathologic con- noma. What is the optimum next step?
firmation and delayed reconstruction. A. Sentinal lymph node evaluation
B. Full thickness biopsy and conjunctival
63. Biopsy of a broad area of pigmentation of map biopsies
the eyelid has been read as lentigo maligna. C. Shave biopsy
What is the treatment of choice? D. Corticosteroid injection and curretage.
A. Complete excision with adequate surgi-
cal margins 68. What is the role of cryotherapy in the treat-
B. Map biopsies looking for localized ment of eyelid melanoma?
invasion A. Possibly useful in conjunctival mela-
C. Cryotherapy to the broad area noma, but not in skin
D. Close observation. B. Recommended for both skin and
conjunctiva
64. Which of the following statements describes C. Useful for skin, not conjunctiva
how to differentiate a compound from a D. Not useful for skin or conjunctiva.
junctional nevus?
A. Compound nevi are larger 69. What is the most important predicator for
B. Junctional nevi are darker and macular, recurrence and survival in patients with eye-
or thinly papular, while compound nevi lid melanocytic skin lesions?
are lighter and elevated compared to A. Excision margins
uninvolved surrounding skin B. Tumor thickness
C. Junctional nevi show melanocytes in the C. Diameter
superficial dermis D. Geographic Location on eyelid.
D. Junctional nevi are more dome-shaped.
70. For a basal-cell carcinoma of the eyelids, in
65. You have removed a medial canthal lesion what location is associated with the worst
which is diagnosed as basal cell carcinoma prognosis for recurrence and mortality?
with morpheaform characteristics. The A. Lower eyelid margin
pathologist confirms the margins are nega- B. Lower eyelid (not involving lid margin)
tive in four quadrants (0°, 15°, 30°, 45°). C. Central upper eyelid
What is the optimum next step? D. Medial canthus.
A. Adjunctive cryotherapy
B. Excision with margin control because of 71. What is the most common type of mela-
the aggressive nature of the tumor noma which occurs on the eyelids?
C. Adjunctive alkylating agents A. Superficial spreading
D. Observation with return in one year. B. Nodular
C. Acrolentiginous
66. Horizontal stability of the eyelid margin is D. Lentigo maligna.
essentially maintained by?
A. Lid retractors 72. What clinical association is characteristic of
B. Muscular tone lentigo maligna?
C. Two point fixation A. It presents as a thickened and nodular
D. Lid protractors. pigmented mass
B. It is characterized by rapid growth
67. A suspected upper eyelid chalazion in a C. It may progress to lentigo maligna
68-year-old patient demonstrates surround- melanoma
ing palpebral conjunctival inflammation, D. Premalignant changes are confined to
the clinically involved area.
22 E. A. El Toukhy

73. Regarding cutaneous horns, which is false: 79. Regarding molluscum contagiosum, which
A. May develop from seborrheic keratosis is false:
B. May develop from basal cell carcinoma A. Usually results from sexual contact and
C. May develop from keratoacanthoma transmission in adults
D. Should undergo biopsy. B. May produce a follicular conjunctival
reaction
74. Regarding keratoacanthoma, which is false: C. May be confluent in immunocompro-
A. Usually develops over a period of weeks mised patients
B. Does not exhibit cellular atypia D. Is caused by a large RNA poxvirus.
C. May be associated with systemic
malignancy 80. Regarding basal cell carcinoma, which is
D. Usually undergoes spontaneous false:
involution. A. Commonly metastasizes
B. May be pigmented
75. Regarding Actinic keratosis, which is false: C. Affects the lower lids in two-thirds of
A. Requires biopsy or excision for cyto- patients
pathologic study D. Is related to ultraviolet light exposure in
B. Develops into squamous cell carcinoma fair-skinned individuals.
in about 20% of lesions
C. Exhibits hyperkeratosis, dyskeratosis 81. Acceptable treatment techniques for basal
and parakeratosis cell carcinoma include all except:
D. Commonly affects the eyelids. A. Cryotherapy
B. Mohs’ micrographic surgery
76. Regarding capillary hemangiomas, which is C. Initial radiation therapy
false: D. Radiation therapy to advanced or recur-
A. Are usually present at birth rent lesions.
B. Regress by age 7 years in 75% of
affected individuals 82. Regarding sebaceous gland carcinoma of
C. May be associated with the Kasabach- the eyelids, which is false:
Merritt syndrome A. Is the third most common eyelid
D. Affect girls more frequently than boys. malignancy
B. Is more common in women than in men
77. Regarding congenital melanocytic nevi, C. Must be confirmed by full thickness
which is false: wedge biopsy
A. Occur in 1% of newborns D. Arises from the meibomian and moll
B. May be seen in “kissing nevi” of the lids glands.
C. Are usually junctional nevi
D. May degenerate into malignant melanoma. 83. Regarding malignant melanoma of eyelid
skin, which is false:
78. Regarding the nevus of Ota, which is false: A. Is usually nodular
A. Is composed of pigmented dendritic B. May arise from congenital nevi
melanocytes C. May arise from acquired melanosis
B. Is usually unilateral and congenital D. May be successfully treated with
C. Often undergoes malignant degenera- cryotherapy.
tion in blacks
D. Arises from dermal melanocytes. 84. Regarding dog bites, which is false:
A. Involve the orbit in 5 to 10% of patients
especially kids
2 Lid Lesions and Malpositions 23

B. Contain Pasteurella multocida organ-


isms in up to 50% of cases
C. Should not be primarily closed for 24 h
because of rabies risk
D. From healthy dogs may contain
Capnocytophaga canimorsus organ-
isms, which can cause meningitis.

85. Lipogranulomatous inflammation is seen in:


A. Fungal infection
B. Tuberculosis
C. Chalazion
D. Viral infection.
87. Regarding the above lesion:
A. Is a stationary disease
B. Results from laxity of the canthal
tendons
C. Treated by disinsertion of the lid
retractors
D. Can occur in both upper and lower lids.

88. Regarding the above lesion, all are true


except:
A. Is caused by medial canthal laxity
B. The lateral distraction test is positive
C. Punctal dilatation is required in the
management
D. A medial spindle procedure is usually
sufficient.
86. This raised skin lesion is likely to be which
of the following?
A. Keratoacanthoma
B. Squamous cell carcinoma
C. Basal cell carcinoma
D. Malignant melanoma.
24 E. A. El Toukhy

89. The above patient has chronic conjunctivi- 91. Treatment of the above lesion would be best
tis with upper eyelids that easily evert. What accomplished by:
additional feature of this disorder would A. Suturing the orbicularis to the inferior
you expect to be present? fornix
A. Tarsal biopsy showing decreased fibrillin B. Suturing the retractors to the tarsus
B. History of hypoglycemia C. Suturing the orbital septum to the cap-
C. Follicular conjunctivitis sulopalpebral head
D. History of sleep apnea. D. Suturing the Lockwood ligament to the
conjunctiva and suspensory ligament of
the fornix.

90. The above patient has:


A. Xeroderma
92. What is the pathogenesis of the above lesion?:
B. Icthyosis
A. Horizontal lid laxity and eyelid retractor
C. Rosacea
disinsertion
D. Tuberous sclerosis.
B. Lower lid retractor dysgenesis
C. Over-riding of preseptal orbicularis
oculi muscle
D. Vertical contracture of tarsoconjunctiva.
2 Lid Lesions and Malpositions 25

93. The above is a test of: 95. The above procedure is used in the surgical
A. Lateral canthal tendon weakness treatment of all except:
B. Medial canthal tendon weakness A. Ectropion
C. Tarsal weakness B. Entropion
D. Orbicularis weakness. C. Ptosis
D. Lid retraction.

94. The most likely cause of the lesion in the


left side is:
A. Paralytic
B. Mechanical
C. Involutional
D. Iatrogenic.
26 E. A. El Toukhy

96. The mechanism by which this procedure 99. Regarding gold weight implants, one is
works is: false:
A. Inward shortening of the conjunctiva A. The most common procedure used for
B. Dilatation and repositioning of the treatment of paralytic lagophthalmus
punctum B. The appropriate weight selection is car-
C. Reinsertion of the lower lid retraction ried out through a process of intraopera-
onto the tarsus tive tapping
D. Rotation of the lid margin. C. The gold weight implant is sutured to
the anterior surfaces of the tarsal plate
D. Platinum can be used as alternative.

100. In involutional ectropion, the most appro-


priate statement is:
A. Caused by Skin laxity
B. There is chronic conjunctival
inflammation
C. Presence of horizontal laxity in tarsal
plate
D. Presence of trichiasis.

101. Epicanthus inversus occurs when:


A. Fold of skin is most prominent in the
upper eyelid
B. Fold of skin is most prominent in the
lower eyelid
C. Fold of skin is distributed equally in
the upper and lower lids
D. Fold of skin arises from the caruncle.

102. In the surgical treatment of upper lid


retraction due to thyroid associated oph-
thalmopathy, which is correct:
A. Contraindicated in exposure keratopathy
97. The above procedure is used in the treat- B. Can be corrected with excision of
ment of: Müller muscle
A. Involutional ectropion C. Insertion of a spacer between the
B. Cicatricial ectropion Müller muscle and levator aponeurosis
C. Involutional entropion D. Lateral tarsorrhapy is the surgical
D. Cicatricial entropion. modality of choice.

98. In a tarsal strip lateral canthoplasty, the strip 103. Quickert sutures:
is sutured to the: A. Have a long lasting effect
A. Opposite eyelid margin tarsus B. Are used for ectropion
B. Opposite limb of the lateral canthal C. Involve lateral tarsal strip
ligament D. Are used for reinsertion of the retractors.
C. Periosteum inside the lateral orbital rim
D. Periosteum external to the lateral orbital
rim.
2 Lid Lesions and Malpositions 27

104. Regarding entropion the least appropriate 109. Which one of the following is the LEAST
statement is: common form of ectropion?
A. Acute spastic entropion follows sclera A. Congenital
buckle procedure B. Paralytic
B. Involutional entropion is usually asso- C. Mechanical
ciated with the lower lid D. Cicatricial.
C. An inferior fornix that is deeper than
normal may indicate lower lid retrac- 110. Repair of lower eyelid involutional entro-
tors disinsertion pion would be BEST accomplished by:
D. The lateral tarsal strip operation is useful. A. Suturing the orbicularis to the inferior
fornix
105. Which one of the following would be the B. Suturing the retractors to the tarsus
best treatment for a patient with typical C. Suturing the orbital septum to the cap-
Bell’s palsy with severe corneal exposure? sulopalpebral head
A. Temporary lateral tarsorrhaphy D. Suturing the Lockwood’s ligament to
B. Pentagonal wedge resection of the the conjunctiva and suspensory liga-
lower eyelid ment of the fornix.
C. Punctual electrocautery
D. Inferior retractor recession with full- 111. The most common cause of upper eyelid
thickness skin grafting of the lower lid. retraction is:
A. Recession of the superior rectus muscle
106. Etiological factors in involutional B. Congenital eyelid retraction
entropion,one is false: C. Surgical overcorrection of blepharoptosis
A. Horizontal eyelid laxity D. Thyroid eye disease.
B. Shortening of the anterior lamella
C. Laxity of eyelid retractors 112. The following are true about the facial
D. Overriding presptal orbicularis muscle. nerve EXCEPT:
A. Does not contain sensory nerves
107. The surgical procedures to correct lid retrac- B. Supplies secretomotor fibers to the
tion with lateral flare include all except: submandibular glands
A. Recession of the levator aponrurosis C. Exits the skull through the styloid
with space foramen
B. Measured myotomy of the levator D. Lies lateral to the external carotid
muscle with lateral tarsorrhaphy artery within the parotid gland.
C. Full thickness transverse blepharotomy
D. Lid splitting, lateral tarsorrhaphy with 113. What is the pathogenesis of acute spastic
recession of lid retractors. entropion?
A. Horizontal lid laxity and eyelid retrac-
108. Clinical clues to the disinsertion of the tor disinsertion
lower lid retractors include all of the fol- B. Ocular irritation or inflammation
lowing EXCEPT: C. Over-riding of preseptal orbicularis
A. White line below the tarsal border oculi muscle
caused by the dehisced edge of the D. Vertical contracture of tarsoconjunctiva.
disinserted retactors
B. Higher than normal lower eyelid position 114. A 65 year old male develops inturning of
C. Decreased movement of the lower lid both lower lid margins. Ophthalmic exam-
on downgaze ination reveals a white subconjunctival
D. Shrinking of the inferior conjunctival line several millimetres below the inferior
fornix.
28 E. A. El Toukhy

tarsal border with no movement of the 119. The operation of plication of inferior lid
lower lid on downgaze. What pathology retractors is indicated in:
has happened in these lower eyelids? A. Senile ectropion
A. Cicatrization of the tarsoconjunctiva B. Senile entropion
B. Disinsertion of the lower lid retractors C. Cicatricial entropion
C. Horizontal lower lid laxity D. Paralytic ectropion.
D. Symblepharon.
120. Fibrous attachment of the lid to the eye-
115. Cicatricial entropion is generally associ- ball is called:
ated with all except: A. Symblepharon
A. Trichiasis B. Entropion
B. Anterior lamellar shortage C. Ectropion
C. Blepharospasm D. Ankyloblepharon.
D. Symblepharon.
121. Telecanthus means:
116. A 50 year old female presents with A. Widened interpupillary distance
inward turning of both lower lid margins. B. Widened root of nose with normal
Ophthalmic examination reveals chronic interpupillary distance
conjunctival inflammation in both eyes C. Widely separated medial orbital wall
with the diagnosis of ocular cicatricial D. Widely separated canthi.
pemphigoid. What is the appropriate plan
of action? 122. Distichiasis means:
A. Anti-inflammatory therapy and sur- A. Increased number of eyelashes in the
gery for entropion lower lid
B. Corneal shielding and anti-inflamma- B. Second row of eyelashes
tory therapy only C. Increased thickness of eyelashes
C. Corneal shielding and anti-inflamma- D. Increased pigmentation of eyelashes.
tory therapy then surgery for entropion
D. Corneal shielding and surgery for entro- 123. In facial nerve palsy; Prevention of gold
pion then anti-inflammatory therapy. weight exposure is best achieved by:
A. Using a small gold weight implant
117. A 50 year old male presents with outward B. Using a large gold weight implant
turning of the left lower eyelid margin. C. Inserting the weight under the orbicu-
The patient has no other significant his- laris muscle
tory. Ophthalmic examination reveals a D. Meticulous suture closure of the skin.
large chalazion in the left lower eyelid.
What treatment is indicated for correction
of the lower eyelid margin malposition?
A. Chalazion incision and curettage
B. Lateral and medial canthal tightening
C. Lateral tarsal strip procedure
D. Medial spindle procedure.

118. When is Van Millingen’s operation indicated?


A. Trichiasis & entropion of upper eyelid
B. Pure trichiasis of upper eyelid
C. Trichiasis & entropion of lower eyelid
D. Pure trichiasis of lower eyelid.
2 Lid Lesions and Malpositions 29

124. Surgical management of the shown patient B. Cryptophthalmos is a rare condition that
includes all of the following, except is caused by a lack of differentiation of
A. Gold weight implantation eyelid structures and is characterized by
B. Lower lid tightening procedure absence of a palpebral fissure with unin-
C. Blepharoplasty terrupted skin from the forehead over
D. Brow lifting. the eye to the skin of the cheek
C. Ankyloblepharon filiforme adnatum is
125. All of the following may occur in a patient a form of ankyloblepharon in which
with a palsy of the seventh cranial nerve, the eyelid margins are connected by
except thin strands of tissue
A. Epiphora D. Distichiasis is a condition in which
B. Keratitis an accessory row of eyelashes grows
C. Ectropion from or are posterior to the meibomian
D. Ptosis. orifices.

126. All of the following are characteristic of 130. Trachoma can cause all of the following
blepharophimosis except changes except:
A. Autosomal dominant A. Distichiasis
B. Lower eyelid entropion B. Punctal stenosis
C. Deformed ears C. Conjunctival scarring
D. Hypoplasia of the superior orbital rims. D. Entropion.

127. A patient undergoes placement of hard 131. All of the following pairs match mecha-
palate graft for lower eyelid retraction. nisms of involutional entropion with the
Which of the following best characterizes surgical repair except:
the epithelium of the graft? A. Horizontal lower lid laxity-lateral tar-
A. Retention of native epithelium sal strip
B. Metaplasia into nonkeratinized epithelium B. Dehiscence of the lower lid retractors-
C. Survival of submucosal glands retractor advancement
D. Conjunctivalization of epithelium. C. Overriding of the pretarsal orbicularis
by the preseptal orbicularis-excision of
128. An old patient has chronic left eye irritation. a strip of preseptal orbicularis
He has a snap back test of greater than 6 mm D. Inward rotation of the lid by steato-
and normal palpebral and forniceal conjunc- blepharon-lower lid blepharoplasty.
tiva. all of the following are true, except:
A. No inferior movement of lower eyelid 132. An old patient with a previous stroke lives
during down gaze in a nursing home. He is on oral anticoag-
B. Deeper than usual inferior fornix ulants.The patient continually complains
C. Presence of a white subconjunctival of foreign-body sensation and discharge
line below the inferior tarsal border n one eye. Which of the following proce-
D. Lower than normal position of lower dures is most appropriate In this setting?
eyelid. A. Rattachment of the capsulopalpebral
fascia
129. Which statement about eyelid abnormali- B. A lateral tarsal strip procedure
ties is false? C. Rotational sutures (Quickert sutures)
A. Congenital coloboma of the eyelid D. Tarsal wedge excision.
always involves the lower eyelid and
can vary from a small notch to a com-
plete absence of the eyelid
30 E. A. El Toukhy

133. Which one of the following is likely to 137. What is the pathophysiologic mechanism
occur, with respect to the epithelium, of the underlying this condition?
transplanted tissue of a hard palate graft? A. Laxity of the tarsal plat
A. It will maintain some form of keratiniza- B. Abnormal attachment of the orbital
tion (orthokeratosis and/or parakeratosis) septum
B. It will remain fully keratinized C. Abnormal attachment of the skin and
C. It will convert from keratinized to orbicularis oculi muscle
nonkeratinized D. Laxity of the canthal tendons.
D. All epithelium will be lost.
138. Ectropion and loss of eyelashes should
134. How does lower eyelid retractor repair for alert one to the possibility of which one of
involutional entropion of the lower eyelid the following?
work by? A. Facial nerve (VII) palsy
A. Reattaching the capsulopalpebral fas- B. Chronic eyelid webbing
cia to the tarsus C. Involutional ectropion
B. Shortening the septum D. Malignancy.
C. Repairing cicatricial changes
D. Horizontally shortening the orbicularis. 139. Unilateral rounding of the medial canthal
tendon is a feature of which disorder?
135. What is the most common complaint fol- A. Fracture of the medial wall of the orbit
lowing successful correction of paralytic B. Connective tissue disease involving
ectropion? the medial canthal tendon
A. Consecutive entropion C. Lacrimal sac tumor
B. Prolonged chemosis D. Avulsion of the medial canthal tendon.
C. Persistent epiphora
D. Overelevation of the lateral canthal 140. A Quickert suture is most effectively used
angle. when repairing what disorder?
A. Spastic entropion
136. When performing a lateral tarsal strip for B. Distichiasis
horizontal lid laxity of the lower lid, what is C. Involutional entropion
the correct placement of the lateral canthus? D. Cicatricial entropion.
A. 2 mm lower than the medial canthus
B. 2 mm above the medial canthus 141. A 4-year-old child is referred for bilateral
C. Outside the lateral orbital rim epiphora. Examination shows eyelashes
D. At Lockwood’s tubercle. on both lower eyelids rubbing against the
inferior cornea. The parents state that an
older sibling has the similar symptoms,
which resolved without treatment. What is
the most likely diagnosis?
A. Entropion
B. Epiblepharon
C. Euryblepharon
D. Trichiasis.

142. What is the preferred treatment for cicatri-


cial ectropion?
A. Lateral tarsal strip plus repair of lower
eyelid retractors
2 Lid Lesions and Malpositions 31

B. Lateral tarsal strip plus skin graft 148. Entropion may be mimicked by all except:
C. Fascia lata suspension of the lower A. Epiblepharon
eyelid B. Distichiasis
D. Lateral tarsal strip plus medial spindle C. Trichiasis
procedure. D. Symblepharon.

143. What term describes an abnormally wide 149. Regarding techniques for entropion repair,
distance between the medial canthi in the which is incorrect:
presence of a normal interpupillary distance? A. Lid retractor reattachment
A. Exorbitism B. Botulinum toxin injection
B. Hypertelorism C. Transverse tarsorrhaphy
C. Telorbitism D. Kuhnt-Szymanowski procedure.
D. Telecanthus.
150. Ectropion has been associated with all
144. Which is incorrect; Eyelid retraction may: except:
A. Result from Muller’s muscle stimula- A. Medial or lateral canthal tendon laxity
tion alone B. Orbicularis muscle weakness
B. Be declared when the lower lid margin C. Cicatricial skin changes
is below the limbus D. Tightening of the inferior lid retractors.
C. Be caused by seventh nerve palsy
D. Be a manifestation of Hering’s law in 151. Techniques available for correction of
the setting of contralateral ptosis. ectropion include all except:
A. Lateral tarsal strip procedure
145. Neurogenic causes of eyelid retraction B. Full-thickness wedge excision
does not include: C. Y-plasty
A. Dorsal midbrain syndrome D. Medial canthal tendon resection.
B. Wernicke’s encephalopathy
C. Palatal myoclonus
D. Impending tentorial herniation.

146. Myogenic causes of eyelid retraction does


not include:
A. Myasthenia gravis
B. Graves’ dysthyroid orbitopathy
C. Familial periodic paralysis
D. Down syndrome.

147. Regarding entropion, which is incorrect:


A. Is often caused by attenuation of the
capsulopalpebral fascia and orbital
septum
B. May be caused by age-related horizon-
tal lower lid laxity
C. Is commonly caused by enophthalmos
D. May be caused by reduced posterior
lid lamellar support.
32 E. A. El Toukhy

154. The above photo is a reported complica-


tion of:
A. Electrolysis
B. Diathery
C. Cryotherapy
D. Laser therapy.
152. The above patient has:
A. Jaw winking syndrome
B. Aberrant regeneration of 3rd nerve
C. Aberrant regeneration of 7th nerve
D. Duane syndrome.

153. The surgical technique shown in the figure


includes all except:
A. Intraoperative choice of the proper
implant
B. Fixation to tarsus with nylon sutures
C. Closure in two layers 155. The above figure is an example of:
D. Recession of the levator muscle. A. Trichiasis
B. Distichiasis
C. Entropion
D. Epibleharon.
2 Lid Lesions and Malpositions 33

B. Immunofluorescence demonstrates
IgG, IgM positivity in the epithelial
basement membrane zone
C. A negative result of immunofluores-
cence rule out possibility of OCP
D. Histology shows subepithelial band
of inflammatory cells, predominantly
neutrophils.

160. Regarding tarsal rotation procedures, all


are correct except:
A. Can be done transcutaneously
B. Can be done transconjunctivally
C. Requires a tarsus that is not deformed
or atrophic
D. Requires more than 3 sutures to induce
rotation.

161. The centurion syndrome is characterized


by:
A. Epiphora in young adults
156. The above anomaly is best treated by: B. Low Hertel exophthalmometer readings
A. Intense lubrication C. Patent nasolacrimal system on irrigation
B. Immediate reduction and tarsorraphy D. Favorable response to intubation.
C. Cinjunctival excision
D. Horizontal lid shortening. 162. The management of the centurion syn-
drome may include all except:
157. Blepharitis is: A. Disinsertion of both limbs of the
A. An acute inflammation of the lid margin medial canthal tendon
B. A chronic inflammation of the lid margin B. Disinsertion of only the anterior limb
C. Inflammation of the eyelid skin of the medial canthal tendon
D. Inflammation of the eyelid skin and C. Medial spindle procedure
underlying soft tissues. D. Lower eyelid retractor plication.

158. What is the most effective treatment of 163. Regarding floppy eyelid syndrome, the
active trachoma? most appropriate statement is:
A. Single dose of 1gm oral azithromycin A. Presence of follicular conjunctivitis
B. Topical neomycin ointment B. Obesity is a strong association
C. Topical fucidic acid ointment C. Sleeping supine is a risk factor
D. Topical quinolone drops. D. No surgical management is required
condition usually resolves with con-
159. Which of the followings statements about servative management.
conjunctival biopsy in Ocular Cicatricial
Pemphigoid (OCP) is true:
A. The part of specimen for immunofluo-
rescence analysis should be submitted
in formalin
34 E. A. El Toukhy

166. Patients with the above condition has


blepharoptosis in one or both eyes. Which
of the following statements accurately
describes the surgical approach to their
form of ptosis?
A. Frontalis suspension is often required
to provide adequate eyelid elevation
and contour
B. Levator advancement/resection is not
useful when treating this disorder
C. Horizontal shortening of the upper lid is
often enough to elevate the affected lid
D. Lash ptosis does not respond to hori-
zontal tightening of the eyelid.

167. A patient presents with obesity, a soft


­rubbery tarsus, watery tearing, and mucus
discharge from one eye. He prefers to
sleep in a face down position. What sur-
gical treatment would be preferred to
improve his symptom?
164. Which of the following is characteristic of A. Horizontal eyelid tightening
the histopathology of the above condition? B. Canthotomy of lateral canthal tendon
A. Decrease in tarsal elastin fibers C. Canthotomy of superior limb of lateral
B. Decrease in conjunctival elastin fibres canthus
C. Decrease in tarsal collagen type I D. Vertical shortening of tarsus.
D. Decrease in tarsal collagen type III.
168. The S in the SAFE strategy can be all
165. A mildly obese patient complains of except:
chronic irritation in both eyes which is A. Bilamellar tarsal rotation procedure
worse in the morning. In one eye, the B. Transconjunctival tarsal rotation
patient has ptosis. What question would procedure
address a risk factor for the patient’s ptosis? C. Lateral tarsal strip procedure
A. Do you sleep face down? D. Surgical resection of maldirected lashes.
B. Do you suffer from recurring, unilat-
eral facial spasms? 169. Regarding trachoma staging, TF means:
C. Do you suffer from dementia? A. Trachomatous inflammation-follicular
D. Do you have a history of Bell’s palsy? B. Trachomatous inflammation–intense
C. Trachomatous conjuctival scarring
D. Trachomatous trichiasis.

170. Regarding trachoma staging, TI means:


A. Trachomatous inflammation-follicular
B. Trachomatous inflammation–intense
C. Trachomatous conjuctival scarring
D. Trachomatous trichiasis.
2 Lid Lesions and Malpositions 35

171. Regarding trachoma staging, the blinding C. Ipsilateral paralysis of the lower facial
stage is: muscles
A. TF D. Contralateral paralysis of the lower
B. TS facial muscles.
C. TI
D. CO. 173. A 52-year-old male patient has a grow-
ing eyelid lesion. He has a strong family
172. Upper motor neuron facial nerve lesion history for colonic cancer. Ophthalmic
results in: examination reveals lid changes sugges-
A. Bilateral paralysis of the upper facial tive of sebaceous cell carcinoma. What is
muscles the most likely diagnosis?
B. Bilateral paralysis of the lower facial A. Bazex syndrome
muscles B. Gardner syndrome
C. Gorlin-Golz syndrome
D. Muir-Torre syndrome.

Answers in this chapter lid lesions

1 A 22 B 43 A 64 B 85 C 106 B 127 A 168 C


2 A 23 B 44 D 65 B 86 D 107 D 128 D 148 D 169 A
3 A 24 D 45 B 66 C 87 B 108 D 129 A 149 D 170 B
4 C 25 D 46 A 67 B 88 C 109 A 130 A 150 D 171 D
5 C 26 A 47 B 68 A 89 D 110 B 131 D 151 C 172 D
6 D 27 A 48 D 69 B 90 B 111 D 132 C 152 C 173 D
7 D 28 B 49 B 70 D 91 B 112 A 133 A 153 A
8 B 29 A 50 A 71 D 92 D 113 B 134 A 154 C
9 A 30 A 51 C 72 C 93 D 114 B 135 C 155 D
10 C 31 D 52 D 73 C 94 D 115 B 136 B 156 B
11 B 32 C 53 D 74 B 95 C 116 C 137 C 157 B
12 A 33 C 54 C 75 D 96 C 117 A 138 D 158 A
13 B 34 C 55 C 76 A 97 D 118 B 139 D 159 B
14 C 35 C 56 C 77 C 98 C 119 B 140 C 160 D
15 D 36 C 57 B 78 C 99 B 120 A 141 B 161 D
16 C 37 B 58 C 79 D 100 B 121 B 142 B 162 A
17 C 38 A 59 A 80 A 101 B 122 B 143 D 163 B
18 A 39 A 60 D 81 C 102 B 123 C 144 B 164 A
19 B 40 B 61 A 82 D 103 D 124 C 145 B 165 A
20 C 41 B 62 D 83 D 104 A 125 D 146 D 166 C
21 B 42 B 63 A 84 C 105 A 126 B 147 C 167 A
Ptosis
3
Essam A. El Toukhy

Blepharoptosis refers to drooping of the upper although its palpebral part shares in the blinking
eyelid and is one of the most common surgical mechanisms.
eyelid disorders. It can occur in both children Both upper eyelids are symmetrical. The
and adults, and can be classified based on the brain considers both lid retractor as yoke mus-
aetiology of the ptosis: neurogenic, myogenic, cle. They receive equal innervations form single
aponeurotic, mechanical and pseudoptosis. subdivision of the oculomotor nucleus in the
Ptosis is the most common lid malposition midbrain. Changes in the position of one lid will
encountered in clinical practice in both adults lead to affection of the position of the other.
and children population and is the most surgi- Evaluation of the ptotic patient should
cally correctable lid disorder. include an attempt to determine the precise aeti-
The upper lid position is a function of the ology of the ptosis. Congenital ptosis is a local-
delicate balance between the lid retractors ized dystrophy of the levator muscle. There is
including levator muscle, Muller’s muscle, and fibrous tissue where striated muscle would be
frontalis muscle, and the lid protractors includ- expected. This correlates well with the sever-
ing the orbital pat and palpebral part of the ity of the ptosis. Mueller’s muscle is normal.
orbicularis oculi muscle. Congenital ptosis may be unilateral or bilateral.
Normally the upper lid covers the upper It maybe classified as simple or complicated by
1–2 mm of the cornea in the primary position, ophthalmoplegia (superior rectus weakness),
providing no obstacle to image formation on blepharophimosis syndrome, and Marcus Gunn
the retina. It follows the globe on looking down jawing winking ptosis.
with no lag. It provides complete coverage of Patients may have amblyopia resulting from
the eye on lid closure. Finally, it rises up for up anisometropia, strabismus, pupil occlusion, or
to 20 mm in extreme up-gaze. meridional amblyopia.
Changing the activity of the levator and In congenital ptosis; the indication for doing
Muller’s muscles, brings all of these movements ptosis surgery is a child who has an eyelid
about. The frontalis muscles are called into obstructing the visual axis, amblyopia, abnor-
action only in extreme up-gaze. The orbicula- mal head position or unsatisfactory facial
ris muscle in mainly used in forceful lid closure appearance. The best time for surgery is around
4 years of age when accurate measurements can
be taken unless the risk of amblyopia and poor
E. A. El Toukhy (*) visual development is high. Most cases of pto-
Oculoplasty Service, Cairo University, Cairo, Egypt sis correction are done under general anesthesia.
e-mail: eeltoukhy@yahoo.com

© The Author(s), under exclusive license to Springer Nature Switzerland AG 2021 37


E. A. El Toukhy (ed.), Oculoplasty for Ophthalmologists, https://doi.org/10.1007/978-3-030-68469-3_3
38 E. A. El Toukhy

However older children around 16–17 years of levator advancement or repair is a good option
age may be performed under monitored anes- for correction of ptosis, with reported success
thesia to allow for the best eyelid height and rates of 70% to more than 95%. Compared to
contour. MMCR and Fasanella-Servat, it has a clear
In adults, the most common cause of ptosis is pathophysiologic-anatomical basis of repair:
aponeurotic (also known as senile ptosis). In this reapproximation of the attenuated/dehisced leva-
condition, the levator muscle is normal, but the tor aponeurosis back to its former anatomical
levator aponeurosis is either attenuated or has position.
undergone dehiscence from its normal insertions The “Age of aponeurotic awareness” directed
on the tarsal plate and in the orbicularis muscle. the trend of ptosis surgery toward the anterior
This may be a naturally occurring involutional approach. The proponents of levator aponeurosis
change, or it may be precipitated by intraocular surgery argued that since the defect of involu-
surgery, long-term daily contact lens wear, ster- tional ptosis was found to be in the aponeuro-
oid use or trauma. sis instead of in the Müller’s muscle or tarsus,
Ptosis surgery for adults is one of the it was improper to violate tissues not directly
most commonly performed procedures by responsible for the disease as per posterior
Oculoplastic surgeons. A detailed preopera- approach ptosis surgery.
tive history and clinical evaluation are crucial Müller’s muscle-conjunctival resection
for determining the cause of ptosis and the best (MMCR), or conjunctivomüllerectomy, is a
procedure for the individual patient. Many sur- good option for correction of mild to moderate
gical procedures have been described to correct upper eyelid ptosis with good levator muscle
ptosis, each with its own indications and advan- function and positive response to phenylephrine
tages. The individual success of any of these preoperatively. Unlike the Fasanella-Servat
procedures depends on its ability to adjust the procedure, MMCR preserves the tarsus and
eyelid position relative to the amount of levator accessory glands and has several advantages:
function present. Ptosis surgery can be broadly predictable, relatively simple to perform, lack of
classified according to whether it is targeting the an external scar, and ability to maintain a natural
posterior upper lid retractor (Müller’s muscle), upper eyelid contour
anterior upper lid retractor (levator aponeurosis) In patients with no or very poor levator func-
or the brow (frontalis muscle). Each procedure tion, the operation that will achieve adequate
has its distinct advantages and own set of com- eyelid elevation is frontalis suspension. In adult
plications. A thorough knowledge of the steps ptosis surgery, this is reserved for patients with
and nuances of each procedure will enable the pre-existing congenital ptosis, myogenic or
surgeon to better use them for the right patients neurogenic ptosis which cannot be corrected
and optimize surgical outcomes. with conventional ptosis surgery on the levator
Levator advancement or resection surgery muscle. In this procedure, the frontalis is used
remains the standard of adult ptosis surgery as a supplemental eyelid retractor as the eyelid
especially in patients with moderate to severe is fixed to the frontalis muscle at the brow. The
ptosis with fair to normal levator function, who patient opens the eye by elevating the brow and
require simultaneous blepharoplasty, do not closes the eye by contracting the orbicularis.
respond to phenylephrine or want lid crease Frontalis suspension surgery may use several
formation. Although most appropriate for surgical techniques and different sling materials.
acquired aponeurotic ptosis, this surgery also Materials include autogenous or banked fascia
works well for neurogenic, myogenic and con- lata and alloplastic materials that include chro-
genital ptosis. It allows for accurate adjustment mic gut, collagen, polypropylene, silicone, stain-
of eyelid height and contour, especially when less steel, silk, nylon monofilament, polyester
performed under local anaesthesia. In most and polytetrafluoroethylene (PTFE). Autogenous
cases with fair to good levator muscle function, fascia lata has proven to give good results with
3 Ptosis 39

comparably low rates of recurrent ptosis and ptosis with fair to normal levator function, who
infections but requires secondary surgery on the require simultaneous blepharoplasty, do not
leg for harvesting of the fascia. respond to phenylephrine or want lid crease
The frontalis muscle flap advancement is formation. Patients who demonstrate mild-to-
a technique of direct transfer of the force of moderate ptosis (<3 mm) with sufficient levator
the frontalis muscle to the eyelid without the function (>8 mm) may benefit from the posterior
insertion of fascia, suture or a graft between lamellar approach, mainly involving Müller’s
the muscle and the tarsus. Frontalis suspen- muscle-conjunctival resection (MMCR) with or
sion by frontalis muscle flap is a well-accepted without tarsectomy. The frontalis suspension is
method of treating severe bleharoptosis. Being less frequently used in adult ptosis surgery but is
from the same patient, there is no risk of rejec- useful in cases with very poor or absent levator
tion or severe body reaction as may occur with function such as pre-existing congenital ptosis,
homogenous or alloplastic materials. There is neurogenic or myogenic ptosis.
no risk of disease transmission. A Frontalis flap Marcus-Gunn ptosis and the blepharophi-
grows with the child’s growth and does not lead mosis syndrome are special types of ptosis that
to cheese-wiring as synthetic materials. The deserves special mention.
frontalis muscle is well developed before fascia
lata maturation. Therefore, this procedure can
be performed earlier, if indicated, in cases of
congenital ptosis. Additional advantages of this
technique include its technical simplicity, lack
of remote scar as the donor site is in the primary
surgical field, minimal ptosis on upgaze, less lid
lag on downgaze, preservation of eyelid contour
and less tendency for the lid to pull away from
the eye. In contrast to traditional frontalis slings,
only one 2 cm brow incision is required. This
direct linkage of the frontalis muscle to the eye-
lid has been documented by postoperative mag-
netic resonance imaging scan.
Blepharoptosis will continue to be a com-
monly presented condition to the ophthal-
mologist and oculoplastic surgeon, given its
interference with the patients’ visual field Ptosis
and quality of life. Numerous surgical tech-
niques have been described in the management 1. A patient present with unilateral ptosis
of blepharoptosis. The choice of treatment is associated with poor levator function, the
dependent upon the severity of the patient’s most appropriate surgical procedure is:
ptosis, the levator function, the response to A. Unilateral frontalis suspension
phenylephrine, and the surgeon’s preference. B. Maximal external levator resection
Levator advancement or resection surgery C. Fasanella-Servat
remains the standard of adult ptosis surgery D. Mullerectomy.
especially in patients with moderate to severe
40 E. A. El Toukhy

2. The following is a feature of the above


disorder:
A. Autosomal Recessive
B. Reduced intercanthal distance
C. Epicanthus inversus
D. Horizontal lid deficiency.

3. Blepharophimosis syndrome, one is false:


A. It is an autosomal recessive
B. Usually present with telecanthus
C. Lower eyelid ectropion 6. The above is:
D. Hypoplasia of the superior orbital rim. A. Is a form of acquired synkinetic neuro-
genic ptosis
4. Blepharophimosis is generally associated B. Caused by aberrant connections
with, one is false: between CN V and levator muscle
A. Ptosis C. Vasculopathic or compressive lesion
B. Epicanthus inversus must be excluded
C. Distichiasis D. Pupil is usually involved.
D. Ectropion.
7. Regarding acquired myogenic ptosis, the
5. Neurogenic ptosis is: least appropriate statement is:
A. Associated with congenital cranial A. Associated with muscular dystrophy
nerve VI palsy B. Surgical procedure is directed towards
B. Associated with congenital Horner levator muscle
syndrome C. Frontalis suspension is used for treatment
C. Associated with abduction defect in eye D. Secondary exposure keratitis can result
movement after surgical correction.
D. Frontalis suspension is contraindicated
due to risk of lagophthalmous.
3 Ptosis 41

8. Fasanella-Servat operation is useful in 11. In this patient with ptosis post cataract sur-
which specific case of ptosis? gery with good levator function and a high
A. Minimal ptosis or effaced upper eyelid crease, what would
B. Ptosis with myasthenia be your procedure of choice?
C. Horner’s syndrome A. Levator muscle resection
D. Congenital ptosis. B. Reinsertion of levator aponeurosis
C. Muller’s muscle resection
D. Bilateral frontalis suspension.

12. The most important determinant in selecting a


corrective procedure for any type of ptosis is:
A. Vertical height of the palpebral fissure
B. Age of the patient
C. Amount of levator function
D. duration of ptosis.

13. All of the following consider ptosis as a


9. A 70-year-old woman presents to you with functional problem that requires correction,
a 3 mm left upper eyelid ptosis with a high one is false:
eyelid crease and normal levator function. A. Ptosis with significant loss of superior
The appropriate treatment of choice is? field
A. Posterior approach standard mullerectomy B. Ptosis with difficulty in reading
B. Levator aponeurosis advancement C. Ptosis that causes sleepy appearance
C. Internal tarsoconjunctival resection D. Ptosis that interferes with daily activity.
(Fasanella-Servat operation)
D. Frontalis muscle suspension using a 14. The poor long term outcome in frontalis sus-
silicone rod to allow postoperative pension surgery is reported with the use of:
adjustment. A. Autogenous tensor fascia lata
B. Banked fascia lata
10. A 12-year-old girl with repeated swelling C. Silicon rods
and ptosis of both upper eyelids complain of D. Gortex suture.
repeated episodes of eyelid inflammation and
swelling. What is the most likely diagnosis? 15. The most common complication of ptosis
A. Dermatochalasis surgery is:
B. Steatoblepharon A. Under correction
C. Blepharospasm B. Over correction
D. Blepharochalasis. C. Eyelid crease asymmetry
D. Lagophthamus with exposure.

16. Which of the following is NOT a feature of


congenital myogenic ptosis?
A. Levator muscle tissue is replaced by
fibrous or adipose tissue.
B. Prominent upper lid crease
C. Lid lag on down gaze
D. Lagophthalmos.
42 E. A. El Toukhy

17. All of the followings can lead to neurogenic C. Epiblebharon


ptosis except D. Telecanthus.
A. Horner’s Syndrome
B. Deep superior sulcus
C. Guillain–Barré syndrome
D. Aberrant regeneration of the oculomotor
nerve.

18. The procedure of choice for moderate uni-


lateral ptosis with good levator function and
a normal upper lid crease
A. Levator muscle resection
B. Mullerectomy
C. Reinsertion of levator aponeurosis
D. Whitnall sling.
22. The proper surgical procedure for repair of
19. A 70-years old female has 4 mm of right this ptotic upper eyelid exhibiting a high
upper lid ptosis and 1 mm of left upper eye eyelid crease, a margin to reflex distance
lid retraction with high eye lid crease on the (MRD) of 0 mm, and excellent levator func-
right side, with normal levator function of tion would be:
both lids. Treatment of choice is A. Resection of the superior tarsal muscle
A. A moderate levator recession of the left B. Unilateral frontalis suspension using
upper eye lid autogenous fascia lata
B. A levator aponeurosis advancement on C. Reattachment of the dehisced levator
the right lid eponeurosis
C. A posterior approach standard mullerec- D. Plication of the levator muscle (16 mm).
tomy on the right upper eye lid
D. A frontalis muscle suspension on the 23. The most common form of blepharoptosis is:
right eye lid using silicone strap to A. Involutional blepharoptosis (aponeu-
allow postoperative adjustment. rotic ptosis)
B. Neurogenic blepharoptosis
20. Which statement regarding fat encountered C. Myogenic blepharoptosis
during eyelid surgery is FALSE? D. Mechanical blepharoptosis.
A. Preaponeurotic fat is orbital fat
B. Extraconal orbital fat is an important 24. In myogenic congenital ptosis, the levator
landmark in identifying the levator complex (in the ptotic eye) is:
aponeurosis A. Disinserted from the tarsus
C. The removal of fat from the upper eye- B. Histologically different from normal
lid nasal, central and lateral fat pads levator complex with decreased muscle
may be done with impunity fibers and fatty infiltrates
D. In the upper eyelid, the nasal fat pad is C. Innervervated by cranial nerve VII
small, whereas the lateral fat pad is the D. Absent below Whitnall’s ligament.
small fat pad in the lower eyelid.
25. In patients with ptosis, the 2.5% phenyle-
21. Which one of the following is found in the phrine hydrochloride test:
blepharophimosis syndrome? A. Will activate the sympathetic receptors
A. Euryblepharon in Műller’s muscle, resulting in eleva-
B. Ankyloblepharon tion of the lid
3 Ptosis 43

B. Can be used to assess the approximate 28. Which measurement represents the margin
elevation of the lid with external levator reflex distance 1 (MRD1)?
advancement A. Difference between vertical fissure
C. Dilates the pupil so that the contralat- height of both eyes
eral eyelid may drop B. From corneal light reflex to lower lid
D. Does not affect blood pressure margin
through systemic absorption of the C. From corneal light reflex to upper lid
phenylephrine. margin
D. From upper lid to lower lid margin.
26. Materials used for frontalis suspension
of the eyelid include all of the following
29. A 70 year old female that has been dia-
EXCEPT:
betic for the preceding 20 years presents
A. Silicone
with right total ptosis. Ophthalmic exami-
B. Gore—Tex
nation is unremarkable except for right
C. Supramid
exotropia. What is the appropriate plan of
D. Polyglactin 910 (Vicryl).
management?
27. Regarding The levator muscle, which is A. Frontalis sling operation
false: B. Frontalis sling operation with medial
A. Is attached to the lesser wing of the rectus resection
sphenoid bone C. Levator resection with medial rectus
B. Is attached to the circle of Zinn resection
C. Turns from muscle into aponeurosis D. Observe for 3–6 months for spontane-
where ligament of Whitnall is found ous resolution.
D. Is penetrated by the superior division
of the oculomotor nerve at the poste- 30. What of the following is a sign of Horner’s
rior one-third and anterior two-third Syndrome?
junction. A. Head tilt
B. Diplopia
C. Mydriasis
D. Mild Ptosis.

31. As regard ptosis:


A. Levator function is good in senile ptosis
B. Lid lag on downgaze is a feature of senile
ptosis
C. Raised skin crease is a feature of congeni-
tal ptosis
DZϭ D. The most common abnormality in congen-
W& ital ptosis is in the levator appeneurosis.

32. Myasthenia patients are at higher risk for all


of the following except;
A. Thymoma
B. Grave’s disease
C. Systemic lupus erythematosus
D. Multiple sclerosis.
44 E. A. El Toukhy

33. Which elevator muscle of the eyelid is 40. Eyelid synkinesis can occur in all of the fol-
involuntary? lowing, except
A. Levator palpebrae superioris A. Congenital neurogenic blepharoptosis
B. Frontalis B. Ocular myasthenia gravis
C. Muller’s muscle C. Aberrant nerve regeneration
D. Orbicularis oculi. D. Duane retraction syndrome.

34. Fasanella Servat operation is indicated in: 41. An early presentation of a 70 years old
A. Congenital ptosis patient with involutional ptosis and good
B. Traumatic ptosis levator function is:
C. Myasthenia gravis A. Eyelid lag
D. Horner’s syndrome. B. Supratarsal thickening
C. Difficulty reading due to downgaze
35. A patient with ptosis presents with retrac- ptosis
tion of the ptotic eyelid on chewing. This is D. Unrelated to cataract surgery.
called:
A. Marcus Gunn jaw winking syndrome 42. Chronic use of contact lenses results in pto-
B. Third nerve misdirection syndrome sis due to:
C. Abducens palsy A. Involutional attenuation of the levator
D. Oculomotor palsy. aponeurosis
B. Repetitive eyelid traction
36. Bilateral ptosis is not seen in: C. Levator muscle dysgenesis
A. Marfan’s syndrome D. Giant papillary conjunctivitis.
B. Myasthenia gravis
C. Myotonic dystrophy 43. An infant presenting with a capillary
D. Kearns Sayre syndrome. hemangioma of the lid has which type of
ptosis?
37. All of the following are potential side A. Aponeurotic
effects of edrophonium testing, except B. Mechanical
A. Tachycardia C. Neurogenic
B. Respiratory arrest D. Myogenic.
C. Syncope
D. Vomitinh. 44. All of the following can be used in the treat-
ment of the capillary hemangioma, except
38. A patient diagnosed with myasthenia gravis A. Dextromethorphan
(MG) requires: B. Propranolol
A. MRI scan of the brain C. Clobetasol propionate
B. B-scan ultrasonography of the eye and D. Interferon-α.
orbit
C. CT scan of the chest 45. Before surgical repair, how long is it
D. Carotid Doppler ultrasonography. advised to observe traumatic ptosis in an
adult?
39. Jaw winking is most commonly due to syn- A. 4 weeks
kinesis of which two cranial nerves? B. 2 months
A. Oculomotor and Facial C. 6 months
B. Abducens and oculomotor D. 12 months.
C. Trigeminal and oculomotor
D. Trochlear and abducens.
3 Ptosis 45

49. All of the following surgeries may be per-


formed by making an incision at the lid
crease except
A. Blepharoplasty
B. Fasanella Servat procedure
C. Lateral orbitotomy
D. Optic nerve sheath fenestration.

50. All of the following are true regarding the


levator palpebrae superioris, except
A. It runs from the posterior lacrimal crest
medially to the lateral orbital tubercle
laterally
B. Its superficial portion inserts into the
orbicularis muscle and subcutaneous
tissues
46. The systemic disorder most commonly C. It originates in close proximity to the
associated with this disorder is: superior rectus origin, just above the
A. Diabetes mellitus annulus of Zinn.
B. Primary amenorrhea D. The muscular portion is shorter than the
C. Hypospadias aponeurotic portion.
D. Coarctation of the aorta.
51. Components in the evaluation of corneal
47. In patients with eyelid ptosis, which of the protective mechanisms prior to ptosis sur-
following is the most important measure in gery include all of the following, except
determining the type of surgery to perform? A. Examination for lagophthalmos
A. Margin reflex distance B. Jones (primary dye) testing
B. Levator function C. Assessment of Bell’s phenomenon
C. Response to phenylephrine testing D. Evaluation of corneal sensation.
D. Palpebral fissure width.
52. The primary abnormality seen in simple
48. If one makes an incision 12 mm above congenital ptosis is in the
the eyelid margin through the full thick- A. Levator muscle
ness of the central upper eyelid, what is B. Levator aponeurosis
the correct order of the anatomic structures C. Levator innervation
encountered? D. Muller’s muscle.
A. Skin, orbicularis oculi muscle, orbital
septum, orbital fat, levator aponeurosis, 53. The primary abnormality seen in ptosis
Muller’s muscle, conjunctiva after cataract surgery is in the
B. Skin, orbital septum, orbicularis oculi A. Levator muscle
muscle, orbital fat, levator aponeurosis, B. Levator aponeurosis
Muller’s muscle, conjunctiva C. Levator innervation
C. Skin, orbicularis oculi muscle, orbital D. Muller’s muscle.
septum, orbital fat, Muller’s muscle,
levator aponeurosis, conjunctiva 54. The procedure of choice in a patient with
D. Skin, orbicularis oculi muscle, orbital ptosis following cataract surgery would be
fat, orbital septum, levator aponeurosis, A. Levator muscle resection
Muller’s muscle, conjunctiva. B. Unilateral frontalis suspension
46 E. A. El Toukhy

C. Muller’s muscle resection 59. A 40-year-old patient is seen 2 months after


D. Reinsertion of levator aponeurosis. blunt trauma to the right orbit. The examina-
tion is normal except for blepharoptosis on
55. Which is a clinical test specifically used in that side. Levator function is normal on both
diagnosing myasthenia gravis (MG)? sides, and the patient states the eyelid posi-
A. Exercise stress test tions were equal on both sides prior to the
B. Ice pack test injury. There is no enophthalmos, and the
C. Thyroid-stimulating hormone (TSH) patient does not complain of diplopia. What
receptor antibody test is the best next step in managing this patient?
D. Three-step test. A. Surgical exploration and repair of the
levator aponeurosis
56. In myogenic congenital ptosis, the levator B. Close observation with no plan for sur-
complex (in the ptotic eye) is: gical correction until 3–6 months after
A. Disinserted from the tarsus initial injury
B. Histologically different from normal C. Computed tomography (CT) scan to
levator complex with decreased muscle rule out an orbital fracture
fibers and fatty infiltrates D. A Tensilon test to rule out new-onset
C. Innervated by cranial nerve VII myasthenia gravis.
D. Absent below Whitnall ligament.
60. A patient with new onset ocular myasthenia
gravis should have a chest CT scan done to
look for what associated condition?
A. Thymoma
B. Sarcoid
C. Apical lung tumor (Pancoast’s tumor)
D. Thyroid disease.

61. Which of the following is least useful in the


evaluation of a patient with acquired ptosis?
A. Interpalpebral fissures
B. Frontalis muscle excursion
C. Levator muscle function
D. Margin-reflex distance.
57. The above complication can occur with all
materials used for frontalis suspension of
except:
A. Silicone
B. Fascia lata
C. Nylon (e.g., Supramid)
D. Goretex.

58. Which of the following is not a component


of Horner’s syndrome?
A. Miosis
B. Anhidrosis
C. Blepharoptosis
D. Decreased stimulation of the levator
muscle.
3 Ptosis 47

65. Recurrent unilateral, or bilateral, eyelid


swelling in a younger patient is suggestive
of which of the following diagnoses?
A. Hemifacial spasm
B. Gorlin’s syndrome
C. Dermatochalasis
D. Blepharochalasis.

66. Which of the following is a contraindication


to Muller’s muscle conjunctival resection?
A. Acquired aponeurogenic ptosis
B. Post-cataract extraction ptosis
C. No eyelid position change following
instillation of topical phenylephrine
D. Mild congenital ptosis.

62. The patient is asked to look from extreme 67. Which of the following tests for myasthenia
downgaze to extreme upgaze. What are you gravis can precipitate respiratory arrest?
measuring? A. Tensilon test
A. Levator muscle function B. Acetylcholine receptor antibody titer
B. Lid lag C. Rest recovery
C. Lagophthalmos D. Ice test.
D. Muller’s muscle function.
68. A patient with congenital ptosis has bilat-
63. What is the most important measurement to eral measurements of margin reflex distance
use when deciding whether a frontalis sling +1 mm, lid fissures of 5 mm, and lid excur-
is the preferred treatment for ptosis? sions of 4 mm. What is the most appropriate
A. Upper eyelid excursion surgical approach to treat the ptosis?
B. Eyelid crease horizontal length A. Bilateral Mullerectomy
C. Palpebral fissure B. Bilateral frontalis suspension
D. Contralateral eyelid retraction. C. Bilateral maximal external levator
resection
64. A 75-year-old woman complains of restric- D. Bilateral Fasanella-Servat.
tion of her upper field of vision and dif-
ficulty reading when looking down. She 69. Which of the following signs is found in
denies any discomfort, epiphora, or diplo- blepharochalasis syndrome?
pia. Her vision is J1 + OU through her well- A. Cicatricial entropion
positioned bifocal segments. A basic tear B. Blepharoptosis
secretion test is normal. Examination shows C. Hypertrophy of orbital fat pads
an eyelid malposition. What is the most D. Thickened eyelid skin.
likely diagnosis?
A. Entropion 70. Regarding congenital myopathic ptosis,
B. Dermatochalasis which is incorrect:
C. Involutional ptosis A. Is less marked in downgaze
D. Ectropion. B. Is associated with an indistinct or absent
upper eyelid crease
48 E. A. El Toukhy

C. Causes occlusive amblyopia in about


20% of patients
D. Is unilateral in about 70% of patients.

71. Regarding blepharophimosis syndrome,


which is incorrect:
A. Is always bilateral
B. Also consists of telecanthus and epican-
thus inversus
C. Is commonly associated with mental
retardation
D. Is seen in 6% of children who have con-
genital ptosis.
74. The above 42 years old myopic patient has
72. Regarding Marcus Gunn jaw-winking, been using contact lenses for 15 years. The
which is incorrect: most probable surgical procedure used to
A. Commonly involves the ipsilateral inter- correct her ptosis was:
nal pterygoid muscle. A. Bilateral levator muscle resection
B. Amplitude is greater in patients who have B. Bilateral levator aponeurosis reinsertion
more severe ptosis. C. Bilateral Muller muscle resection
C. Often becomes less noticeable with D. Bilateral frontalis sling.
increasing age.
D. May require levator ablation as treatment.

73. Regarding dehiscence of the levator


aponeurosis, which is incorrect:
A. Is typically associated with poor levator
function
B. Is associated with an abnormally high or
indistinct upper eyelid crease
C. Occurs in 6% of patients after cataract
surgery
D. May be caused by contact lens wear.

75. Causes of this complication following pto-


sis surgery include all except:
A. Extensive use of diathermy
B. Extensive dissection around the root of
the lashes
C. Extensive dissection on the posterior
surface of the levator
D. Extensive dissection on the anterior sur-
face of the levator.
3 Ptosis 49

78. This complication of ptosis surgery can be


76. This complication following ptosis surgery
prevented by:
occurs due to:
A. Use of local anesthesia with intraopera-
A. Extensive use of diathermy
tive adjustment
B. Extensive dissection around the root of
B. Use of 3 sutures for muscle fixation
the lashes
C. Proper dissection of both muscle horns
C. Extensive dissection on the posterior
D. Complete opening of the orbital septum.
surface of the levator
D. Extensive dissection on the anterior sur-
79. Advantages of frontalis flap procedure
face of the levator.
includes all except:
A. No risk of rejection or severe body
reaction
B. The flap grows with the child
C. The flap develops before fascia lata
D. Presence of a remote but acceptable scar.

80. Frontalis flap procedure results in all


except:
A. Less lid lag on downgaze
B. Less ptosis on upgaze
C. Preservation of lid contour
D. Progressive cheese-wiring effect.

77. This complication of ptosis surgery can be 81. Indications of frontalis flap include all
prevented by: except:
A. Use of local anesthesia with intraopera- A. Acquired ptosis with poor levator
tive adjustment function
B. Use of 3 sutures for muscle fixation B. Congenital ptosis with poor levator
C. Proper dissection of both muscle horns function
D. Complete opening of the orbital septum. C. Recurrent cases after levator surgery
D. Traumatic ptosis with forehead scars.
50 E. A. El Toukhy

82. Regarding the surgical technique for fronta- Answers for this chapter ptosis
lis flap procedure, which is false:
A. Requires a long learning curve 1 B 21 D 41 C 61 A 81 D
B. Is adjustable 2 C 22 C 42 B 62 A 82 A
C. Can be done through a single incision
3 A 23 A 43 B 63 A 83 C
D. Is essentially a rotational flap.
4 C 24 B 44 A 64 C 84 B

83. The following drops can improve the ptosis 5 B 25 A 45 C 65 D


temporarily except: 6 B 26 D 46 B 66 C
A. Apraclonidine 7 B 27 B 47 B 67 A
B. Oxymetazoline 8 C 28 C 48 A 68 B
C. Naphazoline 9 B 29 D 49 B 69 B
D. Phenyepherine. 10 D 30 D 50 D 70 C
11 B 31 A 51 B 71 C
84. Which of the following is associated with
12 C 32 D 52 A 72 A
type 2 blepharophimosis syndrome?
A. Epicanthus tarsalis 13 C 33 C 53 B 73 A
B. Gene mutation in FOXL2 14 B 34 D 54 D 74 C
C. Primary ovarian failure 15 A 35 A 55 B 75 C
D. Increased interpupillary distance. 16 B 36 A 56 B 76 C
17 B 37 A 57 B 77 B
18 A 38 C 58 D 78 A
19 B 39 C 59 B 79 D
20 B 40 B 60 A 80 D
Lid Reconstruction
4
Essam A. El Toukhy

Traumatic and post-surgical eyelid defects vary preoperative evaluation. Anticoagulation should
in size, complexity, and amount of tissue loss. be stopped in the perioperative period whenever
An extensive knowledge of the anatomy of the reasonable with respect to the patients’ systemic
ocular adnexa and potential options for repair risks and with the permission of the prescribing
allows the surgeon to individually tailor the physician.
reconstruction to best suit the patients’ needs. The goals of tumor excision and reconstruc-
This chapter provides a highlight of multiple tion should be outlined in order of importance:
useful approaches for varying degrees of eyelid Removal of the malignancy; restoration of func-
reconstruction. tion; cosmesis.
The pre-operative consultation for eyelid Defects of the anterior lamella of the eye-
reconstruction is central to surgical success and lid can be repaired by direct closure, rotational
centers around managing patient expectations. It flaps, grafts, or a combination of these methods.
should address potential functional and cosmetic The targeted repair of the posterior and anterior
outcomes as well as potential for additional sur- lamellae with careful attention on the amount of
gical interventions. Small lesions can end up tension results in improved post-operative func-
with unexpectedly large defects being ‘tip of tion and cosmesis. This reconstruction serves as
the iceberg’ phenomenon. With proper recon- the backbone for many of the repairs
struction, lid tissues will usually reach excellent
healing over 6–12 months in the vast majority • Reconstruction of both the anterior and pos-
of cases. Procedures of the nasolacrimal system terior lamellae are required
must be addressed including silicone intubation • Either the anterior or posterior lamella must
or the possibility of future conjunctivo-dacryo- have a blood supply
cystorhinostomy if sacrifice of the canaliculi is • A graft on top of a graft will result in failure
required. Similarly, globe prominence, hypo- of both grafts
plasticity of the inferior orbital rim, eyelid lax- • A pedicle flap is required for one of the
ity, and actinic damage should all be addressed. lamellae
Assessment of patient comorbidities, medica- • Minimize vertical tension on the eyelid dur-
tions and allergies is an important portion of the ing closure
• Horizontal tension will typically improve
with healing
E. A. El Toukhy (*) • Vertical tension will not and will cause eyelid
Oculoplasty Service, Cairo University, Cairo, Egypt malposition
e-mail: eeltoukhy@yahoo.com

© The Author(s), under exclusive license to Springer Nature Switzerland AG 2021 51


E. A. El Toukhy (ed.), Oculoplasty for Ophthalmologists, https://doi.org/10.1007/978-3-030-68469-3_4
52 E. A. El Toukhy

• Match tissue color, texture, and quality as


best possible
• Limit cautery to the minimal required amount
• Anatomic re-creation of the canthi is para-
mount to achieve a stable lid
• Use of a frost suture to prevent early post-
operative retraction and to protect the globe
during healing.

A detailed description of the use and steps of


lid reconstruction techniques are covered in this
chapter.

Lid Reconstruction:

2. The above technique is ideal in:


A. Lower Lid coloboma
B. Lower lid retaction
C. Lower lid benign lesions
D. Lower lid malignant lesions.

1. The above technique is ideal in:


A. Lower Lid coloboma
B. Lower lid retaction
C. Lower lid benign lesions
D. Lower lid malignant lesions.

3. This lesion is:


A. Traumatic
B. Congenital
C. Inflammatory
D. Neoplastic.
4 Lid Reconstruction 53

4. When planning reconstruction of an eyelid 10. In lid reconstruction; one can use all except:
defect the surgeon should: A. A flap for the anterior lamella and a flap
A. Replace both anterior and posterior for the posterior lamella
lamella with grafts B. A flap for the anterior lamella and a
B. Avoid undermining adjacent tissue graft for the posterior lamella
C. Minimize vertical tension C. A graft for the anterior lamella and a
D. Allow wound to granulate prior to flap for the posterior lamella
reconstruction. D. A graft for the anterior lamella and a
5. Regarding congenital coloboma the most graft for the posterior lamella.
appropriate statement is: 11. A young male with a history of eyelid
A. An isolated anomaly if present in the trauma was seeking lid reconstruction after
upper medial eyelid primary repair, during surgery we should
B. Eyelid margin is not involved avoid all except;
C. Distichiasis is not a feature of this disease A. Replacing both anterior and posterior
D. Eyelid sharing procedures are recom- lamellae with grafts
mended for children. B. Excising adjacent tissue
6. After surgical excision of a lower lid tumor, C. Vertical tension
the most appropriate procedure for moder- D. Horizontal tension.
ate defect (<50%) is: This 60% defect resulted after a Mohs sur-
A. Semicircular advancement or rotation gical procedure
flaps
B. Advancement of a transconjunctival flap
from the upper eyelid into the posterior
lamellar defect
C. Mustarde procedure
D. Free transconjunctival autografts from
the upper eyelid.
7. Congenital colobomas of the eyelids are
associated with which systemic syndrome?
A. Goldenhar’s syndrome
B. Pierre Robin’s syndrome
C. Hallermann-Streiff syndrome
D. Stickler’s syndrome. 12. What surgical method would be the most
8. A 60-year-old patient underwent full thick- appropriate for reconstruction of the poste-
ness surgical excision of a squamous cell rior lamella?
carcinoma that occupied half of the upper A. Cutler-Beard flap
eyelid. Which of the following procedures B. Bipedicle myocutaneous flap
is best suited for her eyelid reconstruction? C. Full-thickness skin graft
A. Direct closure with lateral canthotomy D. Hughes tarsoconjunctival flap.
B. Tenzel semicircular flap 13. What is the least likely cause that led to the
C. Cutler-Beard procedure eyelid defect?
D. Hughes procedure. A. Basal cell carcinoma
9. Which of the following is not a good option B. Metastatic cancer
for full thickness skin grafting: C. Sebaceous cell carcinoma
A. Upper eyelid skin D. Squamous cell carcinoma.
B. Retroauricular
C. Preauricular
D. Hard palate.
54 E. A. El Toukhy

14. A recurrent squamous cell carcinoma is


excised from the medial canthus. Which
of the following reconstructive techniques
should be avoided to prevent detection of a
deep delayed recurrence?
A. Midforehead rotational flap
B. Full thickness skin graft from the retro-
auricular area
C. Full thickness skin graft from the upper
eyelid
D. Undermining with direct closure. 18. This lid defect requires:
15. Excisional biopsy is a useful treatment A. Hughes flap and a skin graft
modality in which of the following? B. Semicircular flap and a skin graft
A. Lattice corneal dystrophy C. Cutler Beard flap and a skin graft
B. Ocular cicatricial pemphigoid D. Mustarde flap and a skin graft.
C. Nodular scleritis
D. Conjunctival intraepithelial neoplasia.
16. What is the most common complication fol-
lowing repair of total eyelid defects (upper
and lower)?
A. Need for dacryocystorhinostomy
B. Eyelid rigidity manifested by ptosis and
lagophthalmos
C. Proptosis
D. Corneal ulceration.

19. This defect requires:


A. Hughes flap and a skin graft
B. Semicircular flap and a skin graft
C. Cutler Beard flap and a skin graft
D. Mustarde flap and a skin graft.

17. This technique is:


A. Hughes flap
B. Semicircular flap
C. Cutler Beard flap
D. Mustarde flap.
4 Lid Reconstruction 55

20. The best choice for this defect is:


A. Hughes flap with two stage
reconstruction
B. A tarso conjunctival transposition flap
C. A hard palate graft
D. A Mustarde flap and a skin graft.

21. The advantages of this technique include all


except:
A. Maintaining two point fixation
B. Single stage procedure
C. Does not require a skin graft
D. Lined by mucous membrane.
56 E. A. El Toukhy

22. All are true regarding the above lesion


except:
A. Requires staged reconstruction
B. Requires mucous membrane graft from 23. The height of the semicircular defect equals:
the lip or palate A. Up 33% of the lid margin
C. No need for lid sharing B. Up to 50% of the lid margin
D. The brow defect can be repaired C. The width of the defect
simultaneously. D. The length of the defect.
4 Lid Reconstruction 57

24. The next step in the surgical technique is: 27. In the repair of a total eyelid defect from
A. Cutting the capsulopalpebral fascia trauma, what is the preferred use of the
B. Release of the orbital septum avulsed tissue?
C. Cutting the lateral palpebral ligament A. Reimplantation even if the tissue has
D. Creating a periosteal flap. been ischemic for several hours
B. Avoidance of reimplantation because of
infection potential
C. Avoidance of reimplantation because of
graft-versus-host disease
D. Reimplantation with chemotherapy.
28. The below technique is used for:
A. Lower lid Entropion
B. Lower lid Ectropion
C. Lower lid retraction
D. Lower lid Laxity.

25. The next step in the surgical technique is:


A. Lid margin repair
B. Creating two point fixation
C. Closure of deep tissues
D. Reforming the lateral canthus. 29. In lid reconstruction; “Two Point Fixation”
means:
A. Suturing both the orbicularis muscle
and levator muscle
B. Recreation of the medial and lateral
canthal tendons
C. Suturing both lids together and creating
a temporary tarsorraphy at the end of
the procedure
D. Adjusting the eyelid contour medially
and laterally.

26. The advantages of this technique include all


except:
A. Maintaining two point fixation
B. Single stage procedure
C. Leaving a noticeable scar
D. Lined by mucous membrane.
58 E. A. El Toukhy

Answers of Lid Reconstruction

1 B 11 D 21 C
2 D 12 D 22 B
3 B 13 B 23 C
4 C 14 A 24 C
5 A 15 D 25 B
6 A 16 B 26 C
7 A 17 C 27 A
8 C 18 A 28 C
9 D 19 A 29 B
10 D 20 B
Cosmetics
and Injectables 5
Noha El Toukhy

Facial rejuvenation encompasses correction Cosmetic upper blepharoplasty involves the


of facial rhytides and volume attrition by cos- removal or skin, orbicularis muscle, and/or fat of
metic surgical or non-surgical procedures. Facial the upper lid. Patients’ complaints usually involve
ageing is an ongoing natural process which an extra fold of skin or puffiness of the upper
is influenced by intrinsic and extrinsic fac- eyelid. Lateral lacrimal gland prolapse should be
tors. The synergistic effects of reduction of tis- noted, and if present, should be addressed during
sue elasticity, collagen loss, soft tissue atrophy, the surgery with repositioning of the gland. The
persistent use of facial muscles and the effects brow and upper lid constitute a continuum which
of gravity are the main factors for the intrinsic must be evaluated in any patient undergoing an
tissue changes. Photo degradation of tissues, upper lid blepharoplasty.
smoking, lifestyle habits are some of the extrin- As in the upper eyelid, the skin of the lower
sic contributors. The fundamental principle of eyelid is the thinnest in the body and is devoid
facial rejuvenation is to reverse or hault these of subcutaneous fat. Aging changes in the lower
changes with surgical or non surgical methods to eyelid occur variably in each patient: tear trough
improve the tissue descent, restore the lost vol- deformity, orbital fat prolapse, loss of skin elas-
ume, and improve the quality of the skin. ticity, or orbicularis prominence. A custom-
Cosmetic rejuvenation of the periocular area ized approach considering each patient’s lower
can include incisional and non-incisional pro- eyelid configuration is important. Surgical pro-
cedures. Although the use of non-incisional cedures for lower blepharoplasty include the
procedures, including botulinum toxin, dermal transcutaneous approach and the transconjunc-
fillers, and skin resurfacing, has gained in popu- tival approach. Fat can be removed or reposi-
larity, the oculoplastic surgeon should continue tioned into depressed region. The lower eyelids
to be proficient in incisional procedures. The have several particularly subtle anatomical and
most common periocular incisional procedures functional considerations that the careful sur-
include upper blepharoplasty, lower blepharo- geon must consider to select the appropriate
plasty, mid-face lift and browplasty. Careful pre- approach. Overlooking small variations in anat-
operative evaluation is critical, and the choice of omy and function can lead to suboptimal out-
the appropriate surgery is paramount. comes or complications in lower eyelid surgery.
Proper planning and exact execution are essen-
tial to have a good surgical outcome in perform-
N. El Toukhy (*) ing lower blepharoplasty.
Haverford College, Haverford, PA, USA
e-mail: nohaelt@yahoo.com

© The Author(s), under exclusive license to Springer Nature Switzerland AG 2021 59


E. A. El Toukhy (ed.), Oculoplasty for Ophthalmologists, https://doi.org/10.1007/978-3-030-68469-3_5
60 N. El Toukhy

As the lower eyelid transitions to the cheek skin to achieve a lasting improvement. So, in
inferiorly, the suborbicularis oculi fat pad addition to decreasing wrinkles that are present,
(SOOF) is deep to the orbital orbicularis oculi ultimately, prolonged use of BTX prevents fur-
over the inferior orbital rim. Inferior to the ther deepening of the crease and truly prevents
rim, the superficial musculoaponeurotic system signs of aging. Best response is seen in ages
(SMAS) overlies the SOOF. Superficial to the between 30–50 years. The effects of Botox are
SMAS lies an additional malar fat pad. The cor- cumulative, and results improve on repeated
rect position of both the SOOF and the malar fat treatment.
pad lead to a high, smooth cheek characteristic As a cosmetic agent, Botulinum toxin has
of the youthful midface. been used in the management of: forehead wrin-
For the lower lids; Preoperative clinical kles, glabellar folds (frown lines), lower lid
evaluations must address all of the following: wrinkles, crow‘s feet, orbicularis hypertrophy
Skin laxity and wrinkles, Fat prolapse, Tear (sausage roll orbicularis), brow lift and reposi-
trough deformity, Eyelid position, Eyelid lax- tioning, upper gum show (gummy smile), verti-
ity. Techniques include: Transcutaneous lower cal lip lines (smoking lines), mental crease and
blepharoplasty, Lateral tarsal strip, lateral notch, masseter injection (Texas jaw line), verti-
SMAS lifting, Transconjunctival lower blepha- cal platysmal bands. And recently, the introduc-
roplasty, Subtractive blepharoplasty, tissue tion of the mesobotox technique and the Baby
redraping (fat repositioning) blepharoplasty and Botox technique.
Pinch technique skin excision. Fillers are substances used for augmentation
Injectables offer an excellent non-surgical of soft tissues or fill up the volume attrition due
method of facial rejuvenation. Among which, to subcutaneous fat loss associated with aging.
botulinum toxin (BTX) and soft tissue fillers are With increasing age, the body’s natural poten-
one of the most common and favorite tools for tial to produce hyaluronic acid as well as its
non-surgical rejuvenation. inherent hygroscopy decreases thereby playing
BTX injections, frequently applied in treating an ­important role in facial soft tissue atrophy.
spastic facial dystonias have been used for dec- Fillers are primarily indicated for volume aug-
ades and are still the most preferable treatment mentation and correction of static rhytides. They
methods today due to undesired effects of alter- restore symmetry and the volume loss on the
native treatment methods. face. Soft tissue fillers help in re-augmentation
In addition to being used to reduce wrin- of the depleting collagen, and support and lift
kles, BTX is successfully used in the temporary the fat pads and ligaments. The role of dermal
treatment of idiopathic and thyroid dysfunction fillers for facial aesthetics has been revolution-
induced upper eyelid retraction, inoperable lac- ised with the introduction of Hyaluronic acid
rimal duct blockage and temporary induction (HA) fillers. The newer Hyaluronic Acid (HA)
of ptosis in facial paralysis, as well as in other based agents have restored the interest in dermal
areas including extremity hyperhidrosis, brux- fillers as they promise better outcomes with a
ism, migraine, tension-type headaches, and para- lesser side effect.
lytic spasticity. BTX injections to minimize scar Fillers are an excellent adjunct to botuli-
formation have also been reported. num toxin and in many cases, the combination
Its effect on wrinkles has an early temporary is superior to surgery. An ideal combination
reversible phase with relaxation of the muscle is the administration of the neurotoxin to relax
tone and decreased force of contraction giv- the muscles of facial expression and maximally
ing a better appearance during animation. With reduce the dynamic lines, and subsequently, a
repeated injections, the late permanent stable filler is injected to further reduce any remaining
phase results in remodeling of the dermis and static lines.
5 Cosmetics and Injectables 61

Some of the common indications for its use 4. Complications of Botulinum toxin injec-
are: tions include all except:
A. Ectropion
1. Upper face: correction of glabellar lines, B. Ptosis
superior sulcus deformity, temporal fossa C. Epiphora
hollowing and forehead contouring D. Lid lag.
2. Mid- face: midface lift, correction of tear
trough deformity, cheek augmentation, nose 5. Complications of Botulinum toxin injec-
augmentation and contouring tions include all except:
3. Lower face: lip augmentation, marionette A. Diplopia
lines, perioral rhytides, downturned oral B. Dermatochalasis
commissures, and irregular chin lines, pre- C. Lagophthalmos
jowl sulcus, redefining of jaw line and chin D. Gum show.
augmentation.
6. Ptosis caused by Botulinum injection can be
The number of patients with body dysmorphic partly reversed by
disorder, a mental disorder where patients spend A. Carbonic anhydrase inhibitors
the majority of their time worrying about slight B. Prostaglandin analogues
or un-noticeable flaws in their appearances, has C. B- blockers
greatly increased over the past decades. It is D. Alpha-agonists.
important to understand that patients struggling
with BDD are in need of psychological assis- 7. Brow Ptosis can be treated by all except :
tance, not surgical help. These patients do not A. Surgery
need surgeries, but instead, should be referred to B. Endoscopy
psychologists, who would work on the patients’ C. Injections
sense of self perception and self-esteem. D. Laser.

Cosmetics & injectables 8. The upper lid has how many pre-aponeu-
rotic fat pads?:
1. Botulinum toxin is indicated in all the fol-
A. One
lowing except:
B. Two
A. Lid retraction
C. Three
B. Entropion
D. Four.
C. Ectropion
D. Aberrant regeneration of cranial nerves.
9. The lower lid has how many pre-aponeu-
2. Botulinum toxin is indicated in all the fol-
rotic fat pads?:
lowing except:
A. One
A. Irreparable lacrimal obstruction
B. Two
B. Crocodile tears syndroms
C. Three
C. Tension headache
D. Four.
D. Migraine headache.
3. Botulinum toxin is indicated in all the fol-
10. All the following drugs should be stopped
lowing except:
before blepharoplasty except:
A. Healing of facial wounds
A. Herbal supplements
B. Lagophthalmos due to facial palsy
B. Aspirin
C. Ptosis
C. Antihypertensives
D. Wrinkle improvement.
D. Steroids.
62 N. El Toukhy

11. Regarding periocular Botulinum toxin C. IV steroids for compressive optic


injections , one is false: neuropathy.
A. Onabotulinum toxin A was the first D. Observation and pain management.
toxin available for aesthetic indications
B. Is the treatment of choice for benign 16. Regarding botulinum toxin, one of the fol-
essential blepharospasm lowing statement is true:
C. Duration of effect is typically 12 months A. Used in blepharospasm.
D. Complications includes ptosis and cor- B. Must be avoided in patient with
neal exposures. hemifacial spam due to risk of brain
aneurysm.
12 Soft tissue dermal fillers , one is false: C. Lid retraction is a known side effect.
A. Bovine collagen filler was the first filler D. Botulinum toxin B used in cosmetic
B. The hyaluronic acid fillers were derived wrinkle reduction.
from bacteria
C. The hyaluronic acid fillers requires skin 17. Hemifacial spasm is usually associated with
testing for allergic reaction before use all of the following except:
D. Complication of periocular fillers A. Bilaterality
injection includes central retinal artery B. Age of onset over 50 years
occlusion. C. Vascular etiology
D. Involuntary spasm of the orbicularis
13. True statement regarding benign essential muscle.
blepharospasm:
A. Unilateral focal dystonia 18. Few hours following bilateral blepharo-
B. More common in males plasty a patient complain of sudden pain
C. Muscles of the face are also involved near the left eye. Removal of the dressing
D. Caused by vascular compression of the and local examination reveals tense and
facial nerve. ecchymotic left eyelids. The first step would
be in the management of this case:
14. Regarding laser lid skin resurfacing, one is A. Treatment with ice packs
false B. Visual acuity measurement and check
A. Ultrapulsed CO2 and Erbium YAG laser pupillary responses
are the most widely used C. Check corneal sensation for a possibil-
B. Ultrapulsed CO2 laser has a small ther- ity of a cavernous sinus thrombosis
mal injury into the tissue than Erbium D. Open the wound to release a possible
YAG laser retrobulbar hemorrhage.
C. The darker the skin pigmentation ,
the greater the risk of postoperative 19. When performing mid-face rejuvenation
inflammation which structure is elevated to its previous
D. Herpes simplex virus infection after laser anatomic position?
resurfacing is associated with scaring. A. Preseptal orbicularis
B. Retro-orbicularis oculi fat
15. Few hours following blepharoplasty, the C. Lateral canthal tendon
patient complains of pain and reduced vision D. Suborbicularis oculi fat (SOOF).
noticed to have proptosis and abnormal pupil-
lary reaction, the most appropriate step is: 20. Botulinum toxin is indicated in all the fol-
A. CT scan. lowing except:
B. Canthotomy and cantholysis to release A. Essential blepharospasm
retro orbital hemorrhage. B. Postoperative residual squint
5 Cosmetics and Injectables 63

C. A-pattern exotropia 27. Preoperative evaluation of the lower lids


D. Hemifacial spasm. before blepharoplasty should include all
except:
21. Complications of Upper blepharoplasty A. Skin laxity
include all except: B. Tear trough
A. Ptosis C. Eyelid laxity
B. Lid retraction D. Eyelid excursion.
C. Lagophthalmos
D. Ectropion. 28. Preoperative evaluation of the lower lids
before blepharoplasty should include all
22. Complications of Upper blepharoplasty except:
include all except: A. Snap back test
A. Orbital hemorrhage B. Distraction test
B. Asymmetry C. Jones test
C. Brow ptosis D. Fat distribution.
D. Lid gangrene.
29. The pinch technique is used to determine:
23. Complications of Upper blepharoplasty A. Site of incision
include all except: B. Amount of skin resected
A. Dry eyes C. Amount of orbicularis resected
B. Scar formation D. Amount of fat resected.
C. Infection
D. Entropion. 30. Regarding persistant conjunctival chemo-
sis following blepharoplasty, all are true
24. Regarding lacrimal gland prolapse: except:
A. Is a frequent finding in dermatochalasis A. Requires pressure bandage
B. Will disappear after upper B. Is caused mainly by lymphatic
blepharoplasty obstruction
C. Requires excision C. May need surgical excision
D. Requires a separate procedure later. D. Occurs more in diabetics.

25. Regarding tear trough deformity, all are true


except:
A. Is due to descent of the SOOF and
malar fat pads
B. Requires volume addition
C. Results in a double convexity
appearance
D. Can only be corrected surgically.

26. Preoperative evaluation of the lower lids


before blepharoplasty should include all
except: 31. Cosmetic correction of the above lid can be
A. SOOF position done by all except:
B. SMAS position A. Botox injections
C. Puntal position B. Transcutaneous blepharoplasty
D. Lacrimal gland position. C. Fat repositioning
D. Lower lid tightening procedure.
64 N. El Toukhy

34. Treatment of this cosmetic complaint


requires Botox injection in which muscle:
A. Orbicularis Oris
B. Buccinator
C. Levator labii superioris
D. Levator anguli oris.

35. In the midface, botox has all the following


effects except:
A. Eliminate gummy smile
B. Eliminate smoking lines
C. Induce Texas jaw line
D. Induce mental crease.
32. Botox injection in this area can result in all
except:
36. The “ baby Botox “ technique entails all
A. Ptosis
except:
B. Diplopia
A. Injection in younger patients
C. Brow elevation
B. Injection of smaller doses
D. Epiphora.
C. Injection more frequently
D. Less stability of results.

37. Fillers are used in the cosmetic correction


of all except:
A. Superior sulcus deformity
B. Tear trough deformity
C. Depressed scars
D. Sausage shaped orbicularis.

38. Fillers are used in the cosmetic correction


33. Botox injection in this area can result in all of all except:
except: A. Lower lid wrinkles
A. Ptosis B. Upper lid wrinkles
B. Central retinal artery occlusion C. Cheek augmentation
C. Brow elevation D. Periorbital hyperpigmentation.
D. Diplopia.
5 Cosmetics and Injectables 65

39. Patients with body dysmorphic disorder B. TED is regarded as a risk factor for
(BDD) have: post blepharoplasty surgery orbital
a. Type A personality hemorrhage
b. Obvious cosmetic body defect C. Pressure dressing should be conducted
c. History of multiple surgeries directly after surgery
d. Unrealistic expectations. D. Urgent orbital decompression then
high dose of IV steroid is the manage-
40. Management of body dysmorphic disordere ment of choice for postoperative orbital
requires: hemorrhage.
A. Proper preoperative identification
B. Use of special assessment tools and 44. Essential blepharospasm is usually charac-
questionnaires terized by all of the following except:
C. Referal to psychologists A. Unilaterality.
D. A clear plan for surgery. B. Age of onset usually over 50 years.
C. Obscure etiology
41. Regarding blepharoplasty , one is false: D. Involuntary spasm of the orbicularis
A. Lower lid blepharoplasty is most muscle.
commonly performed for cosmetic
indications 45. Rhytidectomy refers to:
B. Upper lid blepharoplasty is most com- A. Face lift
monly performed for functional reasons B. Brow lift
C. Difficulty in reading is an indication for C. Laser skin resurfacing
functional lower lid blepharoplasty D. Injection of botox.
D. Cosmetic upper lid blepharoplasty
often requires skin rejuvenation and 46. A patient calls to report pain, sudden swell-
chemical peals. ing, and decreased vision the night after a
blepharoplasty procedure. What should be
42. Regarding blepharoplasty techniques , one done?
is false: A. Advise the patient to use ice packs to
A. Transconjunctival incision is preferred decrease the swelling.
more than subciliary incision in lower B. Set up an appointment for the patient to
lid blepharoplasty see you the next day.
B. At least 20 mm of skin should remain C. Make arrangements to see the patient
between the inferior border of the eye as soon as possible.
brow and the lower eye lid margin in D. Reassure the patient that discomfort,
upper lid blepharoplasty swelling, and blurry vision are normal
C. During blepharoplasty surgery in a postoperative findings.
patient with dry eye syndrome ,the sur-
geon should preserve orbicularis oculi 47. Which one of the following statements
muscle regarding blepharoplasty is FALSE?
D. The amount of excess skin to be excised A. Repair of lower eyelid dermatochalasis
is determined by pinch technique. and / or steatoblepharon may be fol-
lowed by lower eyelid retraction.
43. Complication of blepharoplasty surgery , B. A transconjunctival blepharoparoplasty
one is false: is a procedure primarily used to per-
A. Loss of vision usually associated with form upper eyelid surgery when trying
lower lid blepharoplasty more than to avoid an anterior incision.
upper lid
66 N. El Toukhy

C. The advantage of eyelid crease fixation C. Diphenhydramine.


in conjunction with blepharoplasty is D. Mitomycin-C.
that it aligns the eyelid crease and the
postoperative scar. 52. Laser skin resurfacing is contraindicated in
D. Damage to the inferior oblique muscle patients taking which one of the following
is a potential complication of both ante- medications?
rior and posterior approaches to lower A. Isotretinoin.
eyelid blepharoplasty. B. Hydrochlorothiazide.
C. Docetaxel.
48. The following are true about the pre- D. Sildenafil citrate.
aponeurotic fat EXCEPT:
A. It is situated between the orbital septum 53. The most likely outcome following inad-
and the levator vertent suturing of the orbital septum into
B. The trochlea divides the fat into a subcutaneous tissues while performing
medial and a lateral pad blepharoplasty is
C. The medial pad has more fibrous tissue A. Ectropion.
than the lateral pad B. Lid retraction in downgaze.
D. The medial pad has a yellowish appear- C. Entropion.
ance whereas the lateral pad is white. D. Blepharoptosis.

49. The following are true about the orbital sep- 54. All of the following are characteristic of
tum EXCEPT: features of blepharochalasis, except
A. Is attached to the arcus marginalis of A. Lacrimal gland atrophy.
the orbital rim B. Excess eyelid skin.
B. Is inserted on the levator at point where C. Blepharoptosis.
the levator muscle becomes aponeurosis D. Blepharophimosis.
C. Is attached to the retractors 4mm below
the inferior tarsal border 55. Which of the following extraocular muscles
D. Limits the spread of cellulitis into the is least likely to be injured during upper or
orbit. lower blepharoplasty?
A. Inferior oblique.
50. All of the following favor the diagnosis of B. Inferior rectus.
benign essential blepharospasm over hemi- C. Superior oblique.
facial spasm, except D. Superior rectus.
A. Absence of abnormal movements dur-
ing sleep. 56. The most significant complication of
B. No involvement of lower facial muscles. blepharoplasty is
C. Synchronous contractures of involved A. Orbital hemorrhage.
muscles. B. Diplopia.
D. Lack of response to neurosurgical C. Overcorrection.
decompression of the facial nerve. D. Cellulitis.

51. What preoperative medication is most 57. The carbon dioxide laser has all of the fol-
appropriate for reducing the chance of irre- lowing characteristics except:
versible scarring in patients prior to under- A. Wavelength in the infrared spectrum
going laser skin resurfacing? B. Able to be seen by the human eye
A. Valacyclovir. C. Utilized for orbital tumor excision
B. Prednisone. D. Operates at 10.6 µm
5 Cosmetics and Injectables 67

58. The site of action of botulinum toxin type C. Direct eyebrow elevation
A (Botox), when used to treat facial move- D. Midforehead lift.
ment disorders, is the:
A. Motor nerve terminal, inhibiting acetyl- 63. For what type of facial spasm is magnetic
choline release resonance imaging useful?
B. Motor nerve terminal, promoting cho- A. Hemifacial spasm
linesterase release B. Benign essential blepharospasm
C. Plasma membrane (sarcolemma) of the C. Acute facial nerve palsy followed by
striated muscle, inhibiting acetylcho- aberrant regeneration
line release D. Blepharospasm associated with dry
D. Plasma membrane (sarcolemma) of the eyes.
striated muscle, promoting cholinester-
ase release 64. Complications of blepharoplasty include all
except:
59. What is the most common complication of A. Superior rectus muscle weakness.
external lower eyelid blepharoplasty? B. Epiphora.
A. Lower eyelid retraction C. Lower lid retraction.
B. Pyogenic granuloma at the incision site D. Ptosis.
C. Lash loss
D. Bacterial infection at the incision site 65. Regarding laser skin resurfacing, which is
incorrect:
60. A patient is evaluated preoperatively for A. Should be avoided in keloid-forming
blepharoplasty surgery. Which of the fol- patients.
lowing is not a relative contraindication to B. Can eliminate wrinkles and skin
surgery? imperfections.
A. Poorly controlled hypertension C. Gives best results in patients who have
B. Severe keratoconjunctivitis sicca fair complexions.
C. Insulin-dependent diabetes D. May cause reactivation of herpes simplex.
D. Atrial fibrillation with anticoagulation.

61. A young girl has had transitory bilateral,


painless eyelid edema which lasts over
years. There is no history of erythema, pru-
ritus, or atopy. Examination shows baggy
upper eyelid skin with a crepe paper-like
appearance. For this patient, what is the
most likely diagnosis?
A. Contact dermatitis
B. Thyroid eye disease
C. Dermatochalasis 66. Regarding the above disorder, which is
D. Blepharochalasis. incorrect:
A. Usually occurs in patients older than 50
62. A young patient presents with a high fore- years.
head and brow ptosis. Which of the follow- B. May be associated with phenothiazine
ing is the best surgical approach to treat the use.
brow ptosis? C. Is not associated with familial dystonia.
A. Pretrichial endoscopic forehead lift D. Is usually bilateral.
B. Coronal forehead lift
68 N. El Toukhy

67. In severe cases of essential blepharospasm, 72. Deoxycholic acid injections in the lids
one may see all except: results in :
A. Decreased tear production. A. Flattening of wrinkling
B. Brow ptosis. B. Reversal of filler action
C. Ectropion. C. Dissolving fat pads
D. Oromandibular dystonia. D. Reversal of botulinum action.

68. Methods of treating essential blepharos-


Answers for this chapter Cosmetics and
pasm include all except:
Injectables
A. Surgical myectomy of affected areas.
B. Chemical injection with doxorubicin.
1 C 19 D 37 D 55 D
C. Oral benzodiazepines.
D. Lid crutches. 2 D 20 A 38 B 56 A
3 C 21 D 39 B 57 B
69. Arterial occlusion following filler injection 4 D 22 D 40 D 58 A
most commonly occur during the injection 5 D 23 D 41 D 59 A
of : 6 D 24 A 42 B 60 C
A. Upper eyelid
7 D 25 D 43 C 61 D
B. Tear trough
8 B 26 D 44 A 62 A
C. Glabella
D. Crow’s feet. 9 C 27 D 45 A 63 A
10 C 28 C 46 C 64 A
70. Reversal of filler injection can be done 11 C 29 B 47 B 65 B
using : 12 C 30 D 48 D 66 C
A. Deoxycholic acid 13 C 31 A 49 B 67 C
B. Hyaluronidase 14 B 32 D 50 C 68 D
C. Choline esterase
15 B 33 D 51 A 69 C
D. Carbonic anhydrase.
16 A 34 C 52 A 70 B

71. In the lower lid, the medial fat pocket is sep- 17 A 35 D 53 B 71 B


arated from the middle fat pocket by the : 18 B 36 D 54 A 72 C
A. Inferior rectus muscle
B. Inferior oblique muscle
C. Capsulopapebral fascia
D. Lockwood ligament.
The Lacrimal System
6
Nadeen El Toukhy

Tearing, a common complaint in the daily corneal and conjunctival staining with dyes as
oculoplastic clinic can result from dry eyes, fluorescein.
hypersecretion of tears, eyelids or eyelash mal- Treatment of dry eyes include tear
positioning, or more commonly from stenosis Supplementation, anti-inflammatory ther-
or obstruction in the lacrimal drainage system. apy, meibomian glands heat therapy, lacrimal
Disorders of the lacrimal drainage system, ­occlusive devices and neurostimulation
which cause tearing, discharge, or medial can-
thal swelling, are common ophthalmic com-
plaints and account for about 3% of visits to Lacrimal Obstruction
general ophthalmology clinics. Accurate evalua-
tion and localization of the pathology is essen- Congenital nasolacrimal duct obstruction
tial for proper treatment and management. (CNLDO) is the most common ocular abnor-
Dry eyes disease is one of the most com- mality in children, aged less than 1year.
monly encountered ocular surface diseases Noncanalization of the inferior caudal end of the
affecting millions of people. The severity var- duct is the most common cause. Spontaneous
ies over a wide spectrum and there are multiple resolution of the obstruction occurs in 96% of
diagnostic and treatment options available. Most the children in the first year of life. Conservative
of the cases are managed by conservative treat- management including lacrimal sac massage and
ment. Newer treatment modalities help improve antibiotics, is the mainstay in this age group. For
patient compliance. older children nasolacrimal probing efficiently
Meibomian gland dysfunction (MGD) deals with most of the obstructions, however,
defined as a chronic, diffuse abnormality of the the timing for probing remains controversial.
meibomian glands, is commonly characterized The other invasive treatments like silicon tube
by terminal duct obstruction and/or qualitative/ intubation, balloon catheter dilation or dacryo-
quantitative changes in the glandular secretion cystorhinostomy may be considered in cases
Tests for dry eyes and MGD include Ocular refractory to probing.
surface disease index (OSDI), Schirmer’s test, Lacrimal punctal stenosis, despite being a
tear break-up time, tear osmolarity, Meibography, common cause of epiphora, is frequently missed
and often misdiagnosed. Moreover, available
management guidelines are inconsistent and
N. El Toukhy (*) lack integrity. More understanding of the patho-
University of Pennsylvania, Philadelphia, USA physiology, etiology, clinical grading and diag-
e-mail: nadineeltoukhy@yahoo.com nosis of this punctal disorder ie required.

© The Author(s), under exclusive license to Springer Nature Switzerland AG 2021 69


E. A. El Toukhy (ed.), Oculoplasty for Ophthalmologists, https://doi.org/10.1007/978-3-030-68469-3_6
70 N. El Toukhy

Chronic inflammation and subsequent fibro- Conjunctivodacryocystorhinostomy: Retrograde


sis is the basic ultra-structure response to intubation dacryocystorhinostomy, or botuli-
various noxious stimuli and appears to be the num toxin injection into the lacrimal gland.
current proposed mechanism for acquired punc- Acquired NLD obstruction is a relatively
tal stenosis. Longstanding treatment with several benign condition that can result from inflam-
topical anti glaucoma agents, such as timolol, mation of unknown causes and eventually lead
latanoprost, betaxolol and pilocarpine have been to secondary acquired stenosis and occlusive
associated with punctal stenosis. Other topical fibrosis of the lumen of the NLD. It is more
agents have also been suggested as causes, and common in elderly patients and affects women
are often administered simultaneously. They twice as frequently as men. It is most commonly
include prednisolone acetate, dexamethasone, an idiopathic involutional stenosis. Diagnosis
chloramphenicol, tobramycin, adrenaline, nap- can be easily reached by a combination of fluo-
hazoline, tropicamide, indomethacin, and mito- rescein dye disappearance test (DDT), lacrimal
mycin C. Punctal affection may be related to drainage system irrigation and diagnostic prob-
the medication themselves, the preservatives ing. Classically; in patients with acquired NLD
in the commercial preparations or the duration obstruction, DCR is the treatment of choice.
of treatment. The term “Dacryotoxicity” was External DCR has a higher success rate than
introduced by the editor (ElToukhy) in 1999 Endonasal DCR, can be done under local anes-
to describe this effect. The use of these topical thesia, requires no expensive instrumentation,
medications results in changes in the chemi- has a shorter learning curve, and when properly
cal and physical properties of the tears drained done, leaves no scar. Stenting may be used in
by the punctum (e.g. Ph, solutes’ concentration, certain patients with DCR.
suspensions, ….) to which the delicate punc- Treatment of acute dacryocystitis includes
tual epithelium reacts by inflammation and later pain relief, warm compresses, topical and sys-
fibrosis. Treatment includes the 3 snip procedure temic antibiotics. Once the acute dacryocystitis
which is the most popular and has been advo- settles, most patients require a dacryocystorhi-
cated by many as the most successful treatment. nostomy (within 2–3 weeks) as a result of block-
Lacrimal stenting procedures can also be used. age within the nasolacrimal duct.
Canalicular obstruction and stenosis can be
either congenital or more commonly acquired. Lacrimal
With inflammation, there is usually swelling
in the area of the canaliculus, and the punc-
tum is erythematous and raised (the so-called
“pouting punctum” sign. Anatomically, the
canalicular obstructions can be proximal cana-
licular (first 2-3 mm of canaliculus), mid cana-
licular (3-6 mm) or distal canalicular (beyond
6-8mm of normal canaliculus). Diagnosis
can be easily made with syringing and a care-
fully done gentle probing which shows a typi-
cal ‘soft’ stop as the probe encounters soft
tissue obstruction within the canaliculus, com-
pared to a ‘hard’ stop seen in nasolacrimal
duct obstructions. A thin probe, preferably 1. The above figure is :
not larger than 00 should be used gently and A. Flurophotometry
never forced inside the canaliculus to avoid B. Meibography
creating a false passage. Treatment is diffi- C. Interferometry
cult and includes: Canalicular Trephination, D. Meniscometry.
6 The Lacrimal System 71

3. The nasolacrimal duct opens at the level of


A. Spheno-ethmoidal recess
B. Superior meatus
C. Middle meatus
D. Inferior meatus.

4. The anterior nasal opening ( nares ) leads to :


A. Spheno-ethmoidal recess
B. Superior meatus
C. Middle meatus
D. Inferior meatus.

2. The above figure represent which grade of 5. The length of the lacrimal system in a 1
gland loss : year old is ;
A. Grade 0 : normal A. 12–15 mm
B. Grade 1 : <33% B. 18–20 mm
C. Grade 2 : <50% C. 24–28 mm
D. Grade 3 : >50%. D. 28–32 mm.

6. The above patient needs :


A. Antibiotics
B. Intubation
C. DCR
D. CDCR.
72 N. El Toukhy

7. The above patient needs :


A. Antibiotics
B. Extubation
C. Reinsertion of the tubes
D. DCR.

8. The above is a complication of :


A. Canaliculitis
B. NLD obstruction
C. Silicone intubation
D. Jones tube placement.
6 The Lacrimal System 73

9. The above is a complication of :


A. Canaliculitis
B. NLD obstruction
C. Silicone intubation
D. Jones tube placement.
74 N. El Toukhy

10. The above is a :


A. Monocanalicular tube
B. Mini Monoka
C. Metereau tube
D. Lacrimal trephine.

11. The above is a :


A. Monocanalicular tube
B. Mini Monoka
C. Metereau tube
D. Lacrimal trephine.
6 The Lacrimal System 75

12. The above is a :


A. Monocanalicular tube
B. Mini Monoka
C. Metereau tube
D. Jones tube.

13. The above is a :


A. Monocanalicular tube
B. Mini Monoka
C. Metereau tube
D. Jones tube.
76 N. El Toukhy

14. Obstruction at P requires : 17. Obstruction at 3 requires :


A. Plug a. DCR
B. Three snip procedure b. Lacrimal trephine
C. Intubation c. Intubation
D. Dilatation. d. Jones tube.

15. Obstruction at 1 requires : 18. Obstruction at D requires :


A. Mini Monoka A. DCR
B. Three snip procedure B. Lacrimal trephine
C. Intubation C. Intubation
D. Dilatation. D. Jones tube Plug.

16. Obstruction at 2 requires : 19. Obstruction at 4 requires


A. Lacrimal trephine A. DCR
B. Mini Monoka B. Lacrimal trephine
C. Intubation C. Intubation
D. DCR. D. Jones tube.
6 The Lacrimal System 77

20. The above is used in the treatment of :


A. Puntal obstruction
B. Canalicular obstruction
C. Pouting punctum
D. Canaliculitis.

22. Treatment of this condition is ;


A. Antibiotics
B. Incision
C. Intubation
D. DCR.

21. Treatment of this condition is ;


A. Antibiotics
B. Massage
C. Probing
D. Intubation.

23. The above patient will develop


A. Chronic dacyocystitis
B. Lacrimal fistula
C. Preseptal cellulitis
D. Orbital cellulitis.
78 N. El Toukhy

29. Regarding nasolacrimal canal one of the


following statement is true:
A. The nasolacrimal fossa is formed by
maxillary and lacrimal bones.
B. The bony nasolacrimal duct is located
at the posterior medial wall.
C. It empties into the middle meatus.
D. Nasolacrimal duct (NLD) is 19mm in
adults.

24. The Above patient has : 30. The most physiological test for assessment
A. Chronic dacyocystitis of lacrimal drainage system will be:
B. Lacrimal fistula A. Syringing
C. Preseptal cellulitis B. Dacryocystography
D. Orbital cellulitis. C. Lacrimal scintigraphy
D. Jones II.
25. A 6 months old child present with history of
epiphora and recent swelling over lacrimal 31. A 15-month-old girl presents with tearing
sac area with discharge, the most appropri- and discharge from the left eye since birth.
ate management is: Which of the following is true regarding
A. Treatment with topical antibiotics. this condition?
B. Treatment with systemic antibiotics. A. The condition is likely to resolve
C. Immediate probing spontaneously
D. Massaging and observation. B. The appropriate treatment is nasolacri-
mal duct probing
26. Osteotomy of which bone is done for mak- C. Dye disappearance testing is likely to
ing opening in DCR: show no asymmetry
A. Ethmoid D. Punctual abnormalities are likely to be
B. Lacrimal the cause.
C. Frontal process of maxilla
D. Inferior meatus. 32. A 65-year-old man presents with tearing
and discharge. On examination, irrigation
27. Regarding the canaliculi: of the lower canaliculus produces mucopu-
A. Papillary conjunctivitis is present. rulent reflux. Which one of the following is
B. Actinomyces Israeli is a frequently iso- true about this condition?
lated pathogen. A. Jones testing will reveal dye in the nose
C. It is a self-limiting disease. B. The condition is likely to resolve with a
D. Canaliculotomy when performed course of antibiotics
should involve both the horizontal and C. There is probably a common canalicu-
vertical canaliculi. lar block
D. The correct treatment is
28. Regarding DCR, one of the following is an dacryocystorhinostomy.
indication for surgery:
A. Incomplete nasolacrimal duct obstruction. 33. During dacryocystorhinostomy the osteot-
B. Persistence of epiphora. omy site is located in which one of the fol-
C. Resolved single episode of dacryocystitis. lowing locations?
D. Painful distension of the lacrimal sac. A. Is within 10 mm of the cribriform plate
6 The Lacrimal System 79

B. Enlarges the opening of the common C. Long-term topical antibiotics have no


duct rule in the management
C. Is adjacent to the valve of Hasner D. Probing should be performed before
D. Is adjacent to the superior turbinate. the age of 6 months.

34. A 50-year-old patient has a 2-day history 37. A newborn presented with swelling over
of left-sided medial canthal swelling, pain, the lacrimal sac , above the medial canthal
redness, and tearing. Few weeks before, he tendon , the child has no breathing problem
noted intermittent epiphora and swelling and there is no evidence of infection , the
that could be relieved with digital massage. most appropriate next step is:
On examination, his visual acuity was 6/6 A. Observation
OU. Medial canthal and lower-eyelid edema B. MRI to rule out meningoencephalcele
and erythema were present. In regards to the C. Initiation of antibiotics
treatment of this condition: D. Massaging of the sac.
A. Cool compresses are applied to the
medial canthus 38. A 65 years old woman complains of tearing
B. Most adults will need a DCR for cor- and discharge. Irrigation of the lower cana-
rection of outflow obstruction liculus produces mucopurulent reflux from
C. Topical antibiotics without systemic both puncti. All are true except;
antibiotics should be prescribed A. Jones testing will not reveal dye in the
D. Diagnostic probing may be therapeutic nose .
in adults. B. There is probably a common canalicu-
lar block.
35. A patient presenting with acute, severe pain C. Most likely diagnosis is lacrimal duct
in the medial canthal region with minimal obstruction.
enlargement of the lacrimal sac with no D. Correct treatment is
inflammation is most likely to have which dacryocystorhinostomy.
of the following condition?
A. Acute dacryocystitis 39. A 5-month-old boy is presented with epi-
B. Chronic dacryocystitis phora due to nasolacrimal duct obstruction.
C. Actinomyces canaliculitis What is the preferred initial treatment?
D. Impacted dacryolith. A. Digital massage
B. Warm compresses
36. With regards to congenital nasolacrimal C. Oral antibiotics
duct obstruction : D. Urgent surgery.
A. 50% resolve in the first year of life
B. Imperforate membrane at the valve of
Hasner is present in half of newborns
80 N. El Toukhy

40. What is the most appropriate management C. Recurrent infection of the lacrimal sac.
option ? D. Dacryoliths (lacrimal stones).
A. Aspiration of the mass with a large
bore needle for biopsy 44. Which one of the following is an indication
B. Curettage with possible incision of for probing of the nasolacrimal system?
punctum A. Acute episode of acquired
C. Dacryocystorhinostomy DCR dacryocystitis.
D. Curettage and incision with irrigation B. Intermittent acquired dacryocystitis.
of canaliculus with povidone iodine or C. Congenital nasolacrimal duct obstruc-
fortified penicillin. tion unresponsive to massage.
D. Work—up of all patients with epiphora.
41. A 12 months old child has had tearing and
discharge from the right eye since birth. 45. In regard to canalicular trauma, all of the
Which of the following statements is true? following are true EXCEPT:
A. Dye disappearance test is likely to A. One may wait 24 to 48 hours after
show symmetry. injury to allow soft tissue swelling to
B. This condition is likely to resolve decrease.
spontaneously. B. Upper canalicular trauma alone should
C. The appropriate treatment is nasolacri- never be surgically repaired so as not to
mal duct probing. risk damage to the remaining nasolacri-
D. Punctual abnormalities are likely to be mal system.
the cause. C. Silicone stents should be left in place
for 3 to 6 months.
42. Which gland does NOT contribute to the D. Surgical microanastomosis of the cut
aqueous layer of the tear film? canalicular ends with silicone stent
A. Krause. intubation offers the best possibility of
B. Main lacrimal successful repair.
C. Zeis.
D. Wolfring. 46. Chronic use of the following medications
has been reported to cause canalicular ste-
43. What is the most common reason for failure nosis EXCEPT:
of a DCR? A. Echothiophate.
A. Obstruction at the level of the common B. Idoxuridine.
canaliculus or bony ostomy site. C. Epinephrine.
B. Unsuspected lacrimal sac tumor. D. Atropine.
6 The Lacrimal System 81

47. The parasympathetic nerve fibers to the lower medial canthus with surrounding
lacrimal gland travel through the following erythema and an elevated tear lake with
nerves EXCEPT: mucoid debris. The puncti appeared normal
A. Deep petrosal nerve in both eyes. What should be done for fur-
B. Greater petrosal nerve ther management of this case?
C. Zygomatic branch of the maxillary A. Urgent DCR
nerve B. DCR after a 2 week course of antibiotics
D. Zygomaticotemporal nerve. C. Lacrimal probing and intubation
D. Oral Antibiotics and warm compresses.
48. The following are true about the lacrimal
gland EXCEPT: 52. Which of the following is the treatment of
A. Receives its blood supply chiefly from canaliculitis?
a branch of the ophthalmic artery A.. Probing.
B. Contains capsule derived from the B. Syringing.
orbital septum C. Dilatation.
C. Is divided into two lobes by the lateral D. Canaliculotomy.
horn of the levator aponeurosis
D. Receives sensory supply from the 53. Mucin layer deficiency of tear film is seen
trigeminal nerve. in:
A. Keratoconjunctivitis sicca
49. The following are true about the nasolacri- B. Lacrimal gland removal
mal system, EXCEPT: C. Canalicular block
A. The canaliculi are found within the D. Herpes zoster.
medial canthus along their full lengths
B. The lower canaliculus is longer than 54. Epiphora means:
the upper canaliculus A. Cerebrospinal fluid running from nose
C. The canaliculus can be dilated to after fracture of anterior cranial fossa
three times its size without affecting B. A presenting feature of a cerebral tumour
its integrity C. An abnormal flow of tears due to
D. Sinus of Maier is found in the common obstruction of the lacrimal duct
canaliculus. D. Eversion of lower eyelid following
injury.
50. The following forms the lacrimal sac fossa:
A. Lacrimal bone and orbital plate of 55. A 2 month-old child presents with epiphora
maxilla. and regurgitation of mucopurulent material:
B. Lacrimal bone and ethmoid bone. The likely diagnosis is:
C. Lacrimal bone and frontal process of A. Mucopurulent conjunctivitis
maxilla. B. Congenital dacryocystitis
D. Lacrimal bone and nasal bone. C. Buphthalmos
D. Encysted mucocoele.
51. A 50 year old woman presented with an
enlarging, tender, red mass in the right 56. Most common site of obstruction in con-
medial canthal area for 2 weeks. She has no genital NLD obstruction:
history of facial trauma or surgery, but had A. Upper canaliculus
a similar episode of medial canthal swell- B. Lower canaliculus
ing and pain 8 months earlier that resolved C. Common canaliculus
with oral antibiotics and warm compresses. D. Valve of Hasner.
Examination revealed a mass in the right
82 N. El Toukhy

57. Initial treatment of congenital dacryocystitis 63. What percentage of infants are born with an
is: imperforate valve of Hasner?
A. Massage A. 10%
B. Probing B. 20%
C. DCR C. 50%
D. Antibiotics. D. 80%.

58. Treatment of chronic dacryocystitis is: 64. Approximately how many days after birth
A. Dacryocystorhinostomy do infants gain full tear production?
B. Antibiotics A. 14 days
C. Probing B. 21 days
D. Massage. C. 35 days
D. 42 days.
59. A 65-year-old woman presents with
65. The most common neoplasm of the lacrimal
watering from her left eye since 2 years.
gland is the
Syringing revealed a patent drainage sys-
A. Adenoid cystic carcinoma
tem. Rest of the examination was normal.
B. Mucoepidermoid carcinoma
A diagnosis of lacrimal pump failure was
C. Benign mixed tumor
made. Confirmation of diagnosis is done by:
D. Adenocarcinoma.
A. Dacryoscintigraphy
B. Dacryocystography 66. The most common malignant neoplasm of
C. Pressure syringing the lacrimal gland is the
D. Canaliculus irrigation test. A. Adenoid cystic carcinoma
B. Mucoepidermoid carcinoma
60. Phenol red test for dry eye: True statement C. Maligant mixed tumor
is: D. Adenocarcinoma.
A. It requires topical anaesthesia
B. It measures the volume of tears as it 67. What anatomic structure divides the lacri-
changes colour on contact with tears mal gland anteriorly into orbital and palpe-
C. If colour changes to blue, it depicts bral lobes?
mucin deficiency A. Orbital septum
D. It requires a pH meter. B. Periorbita
C. Superior transverse ligament
61. Distention of the lacrimal sac superior to D. Levator aponeurosis.
the medial canthal tendon occurs in?
A. Primary acquired nasolacrimal duct 68. A 40-year-old man presents with a one year
obstruction history of gradually progressive painless
B. Canaliculitis proptosis of the right eye. CT reveals globu-
C. Lacrimal sac tumor lar enlargement of the lacrimal gland with
D. Dacryolithiasis. no extension anterior to the orbital rim. All
of the following are true, except
62. As the bony nasolacrimal canal runs inferi- A. Initial approach to the patient should
orly it initially curves include an incisional biopsy
A. Medial and anterior B. Histopathology will show the tumor
B. Medial and posterior has a pseudocapsule
C. Lateral and anterior C. Definitive treatment will necessitate lat-
D. Lateral and posterior. eral orbitotomy
D. The most likely diagnosis is more fre-
quently encountered in men.
6 The Lacrimal System 83

69. In evaluating a child with tearing, all of the Starting from the punctum, what distance
following are causes of reflex hypersecre- will the probe travel before reaching the
tion, except inferior meatus?
A. TORCH infection A. 12 mm
B. Congenital glaucoma B. 20 mm
C. Distichiasis C. 24 mm
D. Epiblepharon. D. 30 mm.

70. The average distance from the lacrimal 75. In adults, the average distance from lacri-
punctum to the nasolacrimal sac is mal punctum to inferior nasal meatus is
A. 2 mm A. 12 mm
B. 6 mm B. 18 mm
C. 10 mm C. 25 mm
D. 12 mm. D. 30 mm.

71. What is the organism that most commonly 76. What is the most commonly performed
causes canaliculitis? clinical test in the evaluation of the adult
A. Nocardia asteroides patient with epiphora?
B. Staphylococcus A. Jones I test
C. Candida a/bicans B. Jones II test
D. Actinomyces israelii. C. Lacrimal irrigation
D. Dye disappearance test.
72. All of the following medications are known
to potentially cause canalicular obstruction, 77. Which one of the following functional tests
except of lacrimal drainage is most likely to yield a
A. Phospholine iodine false-positive result?
B. 5-fluorouracil A. Lacrimal scintigraphy
C. Doxorubicin B. Secondary dye test (Jones II test)
D. Idoxuridine. C. Dye disappearance test
D. Primary dye test (Jones I test).
73. What type of epithelium are the lacrimal
canaliculi lined by? 78. Which one of the following functional tests
A. Stratified cuboidal of lacrimal drainage allows identification of
B. Pseudostratified ciliated columnar a failure of the lacrimal pump mechanism?
C. Stratified squamous A. Schirmer's test
D. Bilayered cuboidal. B. Primary dye test (Jones I test)
C. Dye disappearance test
74. An infant with congenital nasolacrimal duct D. Secondary dye test (Jones II test).
obstruction undergoes lacrimal probing.
84 N. El Toukhy

79. Which is the most appropriate initial step in C. Chronic dacryocystitis


the treatment of this patient? D. Wegener granulomatosis.
A. Dacryocystorhinostomy
B. Broad-spectrum antibiotics 83. The most common site of organic obstruc-
C. Nasolacrimal duct probing tion in acquired nasolacrimal obstruction is
D. Canalicular irrigation. A. Punctum
B. Canaliculus
80. The most important predisposing factor for C. Intraosseous nasolacrimal duct
acute dacryocystitis is D. Valve of Hasner.
A. Chronic blepharitis
B. Acute bacterial conjunctivitis 84. All of the following conditions may present
C. Dry eye with epiphora secondary to impaired blink
D. Tear stasis. function except
A. Sjogren's syndrome
81. When performing an endoscopic dacryo- B. Parkinson's disease
cyst-rhinostomy, part of the following bones C. Scleroderma
are often removed, except D. Progressive supranuclear palsy.
A. Nasal
B. Maxilla 85. Which of the following glands are matched
C. Lacrimal with their correct types of secretions?
D. Ethmoid. A. Moll-apocrine, main lacrimal gland-
eccrine, meibomian glands-apocrine
82. Acute, lancinating pain in the medial canthal B. Glands of Krause-holocrine, gland of
region with minimal noninflamed enlarge- Zeis-apocrine, goblet cells-holocrine
ment of the lacrimal sac is most suggestive of C. Glands of Wolfring-eccrine, gland of
A. Impacted dacryolith Moll-apocrine, goblet cells-holocrine
B. Acute dacryocystitis D. Main lacrimal gland-eccrine, meibomian
glands-holocrine, gland of Zeis-apocrine.
6 The Lacrimal System 85

86. The (OSDI) is used to diagnose: C. Congenital nasolacrimal duct obstruc-


A. Severity of epiphora tion unresponsive to massage
B. Severity of dry eyes D. Workup of all patients with epiphora.
C. Level of lacrimal obstruction
D. Sjogren syndrome. 92. Adult patients presenting with epiphora
with a complete obstruction at the sac-duct
87. All of the following statements are true in junction would be expected to have:
describing the lacrimal gland except: A. Negative dye disappearance test/posi-
A. The lateral horn of the levator separates tive Jones III
the orbital and palpebral lobes B. Positive dye disappearance test/positive
B. The orbital and palpebral lobes have Jones I
separate excretory glands that empty C. Positive dye disappearance test/nega-
into the conjunctival fornix approxi- tive Jones I
mately 5 mm above the superior margin D. Negative dye disappearance test/nega-
of the tarsus tive Jones II.
C. The lacrimal glands are exocrine glands
D. Blood supply is provided by the lacri- 93. Which one of the following statements
mal artery, a branch of the ophthalmic regarding dacryocystograms is true?
artery A. They are a required part of the workup in
acquired nasolacrimal system obstruction
88. The osteotomy site fashioned at the time of B. They are an excellent test for nasolacri-
a dacryocystorhinostomy (DCR): mal function
A. Is adjacent to the valve of Hasner C. They demonstrate canaliculi well
B. Enlarges the opening of the common duct D. They demonstrate the nasolacrimal sac
C. Is adjacent to the superior turbinate well.
D. Is within 10 mm of the cribriform plate.
94. All of the following statements regard-
89. The Jones I test (primary dye test): ing tumors of the nasolacrimal sac are true
A. Accurately defines the location of a except:
nasolacrimal system obstruction A. They may produce painless irreducible
B. Involves irrigating the lacrimal sac with swelling of the lacrimal sac
fluid B. They may produce bleeding on
C. Has a high false-negative rate attempted probing
D. Is a reliable indicator of nasolacrimal C. They do not usually produce secondary
duct obstruction. dacryocystitis
D. They may produce epiphora.
90. What is the most frequently seen primary
malignant tumor of the lacrimal sac? 95. Regarding the canalicular system, which
A. Fibrous histiocytoma statement is false?
B. Hemangiopericytoma A. The ampulla has the largest diameter of
C. Squamous cell carcinoma the canalicular system
D. Lymphoma. B. A common canaliculus is present in
approximately 30% of the population
91. Which one of the following is an indication C. The canaliculus has a diameter of
for probing of the nasolacrimal system? approximately 1.0 mm
A. Acute episode of acquired dacryocystitis D. The average distance from the punc-
B. Intermittent acquired inflammatory tum to the nasolacrimal sac is approxi-
nasolacrimal system obstruction mately 10 mm.
86 N. El Toukhy

96. Regarding irrigation of lacrimal outflow 99. Appropriate initial workup of this patient
system, which statement is false? includes all of the following except:
A. Syringing saline into the lower canali- A. A complete ophthalmic exam
culus that irrigates into the nose indi- B. CT scan of the orbits and sinuses
cates that no obstruction exists and that C. Probing and irrigation of the left nasol-
the system is functioning normally acrimal system
B. Irrigation of the upper punctum with D. Culture and Gram stain of the medial
regurgitation through the upper punc- canthal discharge.
tum suggests an upper canalicular
obstruction 100. Appropriate initial therapy of this patient
C. Irrigation of the lower canaliculus into would include all of the following except:
the sac with complete regurgitation A. DCR
through the upper punctum suggests B. IV antibiotics
obstruction of the nasolacrimal sac or C. Topical antibiotic drops
duct D. Incision and drainage of any pointing
D. It may be helpful to recover fluid from abscess.
the nose to examine for casts
101. In acquired nasolacrimal system obstruc-
97. All of the following are indications for tion, where is the blockage most frequently
a conjunctivodacryocystorhinostomy or located?
CDCR (Jones tube procedure) except: A. Canaliculi
A. Lacrimal canaliculi have been B. Nasolacrimal sac
destroyed C. Nasolacrimal duct
B. Canalicular remnants cannot be anasto- D. Inferior turbinate.
mosed with the intranasal cavity
C. Common canalicular obstruction 102. What is the most common bacterial etiol-
combined with nasolacrimal duct ogy in acute dacryocystitis?
obstruction A. Actinomyces israelii
D. Paralytic or scarred eyelids with absent B. Pseudomonas aeruginosa
canalicular pumping mechanism. C. Streptococcus pneumoniae
D. Staphylococcal species.
98. In acute dacryocystitis: A 12-month-old child has right-sided epi-
A. Topical antibiotics without systemic phora since birth. The mother has been
antibiotics should be prescribed massaging the right nasolacrimal sac for
B. Cold compresses are applied to the the past 6 months with no improvement:
medial canthus
C. Diagnostic probing may be therapeutic 103. In congenital nasolacrimal system obstruc-
in adults tion, where is the level of the obstruction?
D. Most adults will need a DCR for cor- A. Common canaliculus
rection of outflow obstruction. B. Lacrimal sac
A 10-year-old boy involved in an accident C. Valve of Rosenmiiller
few months ago has left-sided epiphora D. Valve of Hasner.
since the accident, along with a 1-week
history of fever and progressive swelling, 104. The next therapeutic recommendation
redness, and pain in the left medial canthal would include:
region with mucopurulent discharge from A. Continuing massage
the medial canthus: B. Nasolacrimal system probing
C. DCR
6 The Lacrimal System 87

D. Cbservation, as most congenital 106. All of the following organisms are associ-
obstructions resolve without therapy. ated with canaliculitis except:
A. Actinomyces
105. Silicone stent intubation (with possible B. Candida
inferior turbinate infracture) is indicated in C. Acanthamoeba
this patient when: D. Streptomyces.
A. Massage therapy has proven
unsuccessful 107. Which one of the following suggests a
B. Dacryocystography (DCG) shows diagnosis of canaliculitis?
obstruction at the level of the nasolacri- A. Mucopurulent reflux from punctum
mal duct with compression of the lacrimal sac
C. Nasolacrimal system probing has B. Gritty sensation on probing with yel-
proven unsuccessful low-tinged concretions
D. The patient is older than 12 months. C. A palpable subcutaneous mass above
The shown patient reports a 3-month his- the medial canthal tendon
tory of intermittent tearing and mattering D. Palpable masses in the lacrimal sac.
in her right medial canthus. Additionally,
she has noted focal swelling and tenderness
near her lid margins.

108. Treatment of canaliculitis includes all of 109. Adenoid cystic carcinoma of the lacrimal
the following except: gland is best treated by:
A. Canalicular curettage A. Exenteration and removal of involved
B. Canalicular incision and debridement bone
C. Canalicular irrigation B. Radiation therapy
D. DCR. C. Chemotherapy
D. All of the above.
88 N. El Toukhy

110. What is the most common organism impli- 115. When copious mucous refluxes from
cated in dacryocystitis? the superior canaliculus while irrigating
A. Non-septate fungi through the inferior canaliculus, what is the
B. Gram-positive bacteria most likely site of obstruction?
C. Septate fungi A. Inferior canaliculus
D. Gram-negative bacteria. B. Superior canaliculus
C. Nasolacrimal duct
111. Blood-tinged tears should prompt what D. Common internal punctum.
treatment?
A. Balloon dacryoplasty 116. A 1-week-old infant is having difficulty
B. Biopsy of lacrimal sac breathing due to bilateral congenital dacry-
C. Probing and tube placement ocystocele. What management is needed?
D. Dacryocystorhinostomy. A. Urgent decompression in the operating
room
112. A patient with acute dacryocystitis, reflux B. Systemic antibiotics
of pus from the canaliculi, and preseptal C. Topical antibiotics and massage
cellulitis should be treated with which of D. Bedside probing of the nasolacrimal
the following? duct.
A. Immediate dacryocystorhinostomy
B. Massage 117. Regarding dacryocystography, which is
C. Systemic antibiotics incorrect:
D. Probing and irrigation for diagnosis A. Can define the site of complete lacrimal
confirmation. system obstruction
B. Can visualize a filling defect in patients
113. What is the appropriate treatment for acute who have lacrimal sac tumo
dacryocystitis with localized abscess? C. May evaluate lacrimal system physi-
A. Irrigation and probing of the lacrimal ologic function
sytem followed by application of warm D. Can image compression or deflection
compresses of the lacrimal sac or duct.
B. Oral antibiotics and drainage of abscess
or immediate dacryocystorhinostomy 118. Regarding congenital nasolacrimal obstruc-
C. Topical antibiotics tion, which is incorrect :
D. Surgical creation of a permanent dacry- A. Should usually be treated by about age
ocutaneous fistula. 1 year with irrigation and probing
B. Should be treated with silicone intuba-
114. What would be the preferred management tion after two failed probing attempts
to treat a patient with membranous, con- C. Associated with amnioceles requires
genital, nasolacrimal duct obstruction and probing at an early age
stenosis of both upper and lower canaliculi? D. Spontaneously resolves in more than
A. Bicanalicular intubation of the nasolac- 90% of patients by age 1 year.
rimal duct
B. Bicanalicular intubation with 119. Regarding dacryocystorhinostomy, which
dacrocystorhinostomy is incorrect:
C. Monocanalicular intubation A. Has a success rate of 90%
D. Bicanalicular ring intubation with pig- B. Requires a skin incision below the
tail probe. medial canthal tendon
6 The Lacrimal System 89

C. May require incision of the anterior 126. Punctal plugs are used in the management
limb of the medial canthal tendon of all except:
D. Usually requires silicone tube A. Dry eyes
placement. B. Punctal occlusion
C. Glaucoma patients on chronic
120. A normal OSDI is : medications
A. Below 12 D. Post LASIK as a temporary measure.
B. 13-22
C. 23-32 Answers for this chapter Lacrimal System
D. Above 33.
1 B 33 A 65 C 97 C
121. An increase of tear osmolarity above which 2 C 34 B 66 A 98 D
value is seen in dry eyes?:
3 D 35 D 67 D 99 C
A. 290 mOsm/L
4 C 36 B 68 A 100 A
B. 300 mOsm/L
C. 310 mOsm/L 5 B 37 B 69 A 101 C
D. 320 mOsm/L. 6 C 38 B 70 C 102 C
7 B 39 A 71 D 103 D
122. An inflammatory component of dry eyes is 8 C 40 D 72 C 104 B
confirmed by detection of which substance 9 D 41 C 73 C 105 C
in tears : 10 D 42 C 74 B 106 C
A. Lysozyme
11 B 43 A 75 D 107 B
B. Lactoferrin
12 D 44 C 76 C 108 D
C. MMP-9
D. Cytokine. 13 D 45 A 77 D 109 D
14 B 46 D 78 D 110 B
123. Meibomian glands heat therapy entails 15 A 47 A 79 B 111 B
temperatures of : 16 A 48 B 80 D 112 C
A. 40 C 17 A 49 A 81 A 113 D
B. 42 C 18 A 50 C 82 A 114 A
C. 44 C
19 A 51 B 83 C 115 C
D. 46 C.
20 A 52 D 84 A 116 A
124. Neurostimulation is used in treating : 21 B 53 D 85 C 117 C
A. MGD 22 B 54 C 86 B 118 C
B. Aqueous deficient dry eyes 23 B 55 B 87 B 119 D
C. Corneal epithelial defects 24 B 56 D 88 D 120 A
D. Goblet cell deficiency. 25 B 57 A 89 C 121 C
26 C 58 A 90 C 122 C
125. Dacrotoxicity entails :
27 B 59 A 91 C 123 B
A. Lacrimal obstruction caused by alkali
injury 28 C 60 B 92 C 124 B
B. Lacrimal obstruction caused by eye 29 A 61 C 93 D 125 B
drops 30 C 62 D 94 C 126 C
C. Lacrimal obstruction caused by 31 B 63 C 95 B
trachoma 32 D 64 D 96 A
D. Lacrimal obstruction caused by iatro-
genic injuries.
Trauma
7
Essam A. El Toukhy

Eyelid, adnexal and orbital injuries can be a part under anesthesia can be done to avoid further
of multisystem trauma. The basic ABCs of the globe injuries during manipulation of the eyelid.
trauma management should be considered and The eyelid is examined for the extent of the
applied in every trauma patient. This includes wound and if it involves the septum, the mus-
securing a patent airway and stabilization of the cle, lid margin or canaliculus. Canalicular injury
circulation. Ophthalmic evaluation and manage- is suspected when the injury lies medial to the
ment are deferred until more serious problems punctum. Medial or lateral canthal injuries as
are addressed. well as tissue loss should be ruled out
Once the patient is stable, attention could Most lid wounds could be repaired under
be directed to the eye and orbital injuries. The local anesthesia using lidocaine1% with epi-
patient should be evaluated for any globe or nephrine 1:100,000. This can be done in the
optic nerve injuries. This may be difficult emergency room if minor or in the operative
especially in patients who are unconscious or theatre in most injuries. General anesthesia is
uncooperative. The eyelid may be swollen and reserved for extensive injuries, associated cana-
difficult to open, so care should be taken to licular injuries or poorly cooperative patients.
avoid forceful opening of the eyelid as this may Nerve blocks are ideal in such situations.
worsen the already traumatized globe. It should be remembered to reestablish the
Circumstances of the injury can help deter- integrity of the basic lid parts; anterior lamella,
mine the type and extent of the trauma. The posterior lamella, the lid retractors mainly the
mechanism of injury can give an idea about the levator, the canaliculi and the canthal tendons.
depth of the wound as well as the possibility of Tissue loss may be in anterior lamella or it
foreign body presence. can be full thickness involving the lid margin.
This should include evaluation of the globe, In such conditions, it should be remembered to
adnexal tissue, orbit and face. If the patient is avoid undue tension on the wound margins. This
conscious and cooperative, visual acuity, pupil- situation can be dealt with in a manner similar to
lary responses, intraocular pressure measure- lid reconstruction after tumor excision.
ment as well as dilated fundus examination Canalicular lesions may be missed. They
should be performed. Sometimes examination should be suspected in injuries medial to the
punctum that may be laterally displaced. The
diagnosis is confirmed by direct visualiza-
E. A. El Toukhy (*) tion of the cut edge or passing a probe into the
Oculoplasty Service, Cairo University, Cairo, Egypt canaliculus.
e-mail: eeltoukhy@yahoo.com

© The Author(s), under exclusive license to Springer Nature Switzerland AG 2021 91


E. A. El Toukhy (ed.), Oculoplasty for Ophthalmologists, https://doi.org/10.1007/978-3-030-68469-3_7
92 E. A. El Toukhy

Early repair of the canalicular injury is much wall fracture or presence of foreign body is
easier and more successful than late repair suspected.
or conjunctivo dacryocystorhinostomy with A significant orbital trauma can result in a
Jones tube. This must be done under the micro- range of manifestations from orbital contusion
scope with high magnification. It can also be to an orbital wall fracture. Orbital wall fractures
identified using injection of a fluorescein dye may also less commonly occur with a penetrat-
or vesicoelastic material or air. A stent should ing injury. Whenever a penetrating orbital injury
be placed through the transected canaliculus. is present, the patient must be evaluated for the
Bicanalicular silicone tube is commonly used presence of an intraorbital foreign body.
however, some surgeons use monocanalicular Although most orbital injuries are self-limit-
tubes. Destruction of the upper lacrimal system ing, orbital trauma may result in serious seque-
especially with chemical injuries and oblitera- lae that may require emergent intervention such
tion of the canaliculi usually necessitates con- as orbital hemorrhage, traumatic optic neuropa-
junctiveodacryocystorhinostomy (CDCR) with thy and oculocardiac reflex secondary to an
insertion of Lister Jones tube. Chronic dacryo- impinged rectus muscle.
cystitis or complete NLD obstruction are treated Orbital fractures usually present in the set-
by conventional DCR. ting of a blunt trauma to the orbit and face. The
Lacrimal passage injuries associated with term orbital blow out fracture implies increase
orbital or nasal fractures may be overlooked in the orbital volume secondary to an outward
especially with the edema or ecchymosis. deformity in the inferior and/or medial orbital
However, associated nasal bone fractures as well wall(s) which occurs following an impact to
as traumatic telecanthus should raise the index the orbit by an object that is equal to or larger
of suspicion. than the dimensions of the orbital aperture. This
A nasoethmoidal fracture usually results from deformity may be accompanied by herniation
a force delivered across the nasal bridge and of the orbital contents into the adjacent cavi-
it is very common in automobile accidents in ties; namely the maxillary or ethmoid paranasal
which the face strikes the dashboard. The nasal sinuses.
bones become fractured and displaced. The lac- The infraorbital groove, which is located
rimal and sphenoidal bones are usually crushed. medially in the floor, is an area of weakness,
They are associated with surgical emphysema. thus orbital floor fractures are usually located
Traumatic telecanthus is usually present in asso- in its vicinity. This is an important landmark in
ciation with lacrimal passage injury. relation to orbital floor fractures as damage to
Wounds of the eye brow should be meticu- the infraorbital nerve presents with loss of sen-
lously sutured with proper alignment of the sation in the cheek, side of nose and upper lip.
upper and lower border of the brows. If the The floor of the orbit is the roof of the underly-
wound is deep it should be closed in layers ing maxillary sinus.
to minimize scar stretching. However many Nausea, vomiting, palpitations and sweating
wounds of the eye brow will show few weeks may be symptoms of oculocardiac reflex sec-
after healing as a hairless scar. This could be ondary to entrapment of an extraocular muscle,
managed by scar revision and follicular hair which more commonly occurs in children.
transplantation from the opposite or the same Orbital floor fractures in children tend to dif-
brow. fer from those in adults. This is due to the elas-
Evaluation of the orbit includes searching tic nature of bones in children, which result in
for ocular motility deficit, surgical emphysema, greenstick fractures and ‘trap-door’ phenom-
hyposthesia of the check, nose or upper lip in enon. Similarly the Oculocardiac reflex and the
addition to palpable orbital rim fractures. Orbital white-eyed blow out fractures are conditions
imaging with CT is requested when orbital that are seen in mostly only children.
7 Trauma 93

The preferred imaging technique is comput- 3. Typical finding of blowout fracture of the
erized tomography (CT) without intravenous orbital, one is false:
contrast. This is the most sensitive imaging tech- A. Ecchymosis and edema of the eyelids
nique delineating orbital wall fractures. It out- B. Diplopia
lines the location, extent and comminution of C. Exophthalmos
fractures, as well as the presence of extraocular D. Emphysema of the orbit and eyelids.
muscles and orbital soft tissue entrapment. 4. The most serious danger to vision is:
In children with extraocular muscle impinge- A. A blow to the eye ball
ment, release of the impinged muscle and orbital B. Fracture through sphenoid bone
wall fracture repair should be performed within C. Monocular proptosis
24 to 48 hours of the injury. Patients with ocu- D. Orbital cellulitis.
locardiac reflex must be operated immedi- 5. The most common organism in trauma-
ately. Repairing fractures in the first 8 days associated preseptal cellulitis is:
after trauma has a better long term prognosis as A. Haemophilus influenzae
regards motility and enophthalmos than repair- B. Streptococcus pneumoniae
ing after 8 days. C. Bacillus cereus
Orbital floor fractures can be approached D. Staphylococcus aureus.
transcutaneously through a subciliary or lower 6. A man presented with injury of the left
eyelid crease incision. However, transconjuncti- brow with a stick. visual acuity is 20/20 OD
val approach through the inferior tarsal conjunc- and 20/200 OS. Examination revealed prop-
tiva is preferred for better cosmetic outcome. tosis of the left eye with a large tense eyelid
The fracture is identified, prolapsed tissues are hematoma and subconjunctival hemorrhage.
restored back and an implant material is fash- Pupils showed left RAPD. Fundus showed
ioned to cover the defect completely and over- pulsating central retinal artery of the left
lapping the surrounding intact bone by 3–4 mm eye. What would be the most appropriate
circumferentially. immediate management?
Trauma to the orbit can also result in other A. Paracentesis
types of injuries including: orbital roof fractures, B. Begin intravenous corticosteroids
mid-facial fractures, traumatic orbital hemor- C. Intravenous mannitol 20%
rhage, surgical emphysema, carotid cavernous D. Lateral canthotomy and cantholysis.
fistula or septic cavernous sinus thrombosis. 7. The most likely pathophysiology for diplo-
pia development in the setting of traumatic
Trauma carotid cavernous fistula is:
A. Damage to the third cranial nerve from
1. The lacrimal drainage system is usually
elevated intracranial pressure
injured in which type of Le Forte fractures?
B. Compression of the sixth cranial nerve
A. Le Forte I
within the cavernous sinus
B. Le Forte II
C. Compression of the fourth cranial nerve
C. Le Forte III
D. Disturbed eye movements due to orbital
D. All 3 types.
edema.
2. Numbness due to an orbital blowout frac-
8. Which of the following findings is Not
ture is due to:
associated with orbital floor fractures?
A. Entrapment of the infraorbital nerve dis-
A. Late enophthalmos few weeks after the
tal to the foramen
fracture
B. Fracture of the body of the zygoma
B. Tear drop sign on CT scan
C. Fracture of the infraorbital rim
C. Unilateral mid-facial sensory loss
D. Injury of the infraorbital nerve within
D. Rapid improvement in traumatic diplo-
the orbital floor.
pia over a 24-hour period.
94 E. A. El Toukhy

9. First choice in the evaluation of acute B. Within the zygoma medial to the infra-
orbital trauma is: robital canal
A. Orbital ultrasound C. Within the zygoma medial to the infra-
B. Palpation robital fissure
C. CT scan D. Within the maxilla medial to the infra-
D. MRI. robital canal.
10. Indications for repair of orbital blow out 14. In exploring upper eyelid trauma with a
fracture include all of the following except; full-thickness laceration involving the eye-
A. Fracture involving more than half of the lid margin, the physician must be aware of
orbital floor the order in which the anatomical structures
B. Inferior rectus weakness are normally encountered. The correct order
C. Pain and oculocardiac reflex on upgaze is:
D. Significant inferior rectus entrapment. A. Skin, orbicularis muscle, preaponeurotic
11. A patient is struck on the right eye. fat, Muller’s muscle, levator aponeuro-
Radiography shows a fracture of the right sis, conjunctiva
orbital floor, forced duction test cannot be B. Skin, preaponeurotic Fat, orbicularis
done due to poor cooperation. 2 days after muscle, septum, levator aponeurosis,
the injury, 3 mm of right exophthalmos is Muller’s muscle, conjunctiva
present, movement of the eye is restricted C. Skin, orbicularis muscle, septum,
in up gaze, down gaze and horizontal gaze. preaponeurotic fat, levator aponeurosis
Treatment should be; Muller’s muscle, conjunctiva
A. Urgent lateral canthotomy D. Skin, orbicularis muscle, preaponeu-
B. Caldwell–luc incision and packing of rotic fat, septum, levator aponeurosis,
the maxillary sinus Muller’s muscle, conjunctiva.
C. Skin incision over the inferior orbital 15. The most common site of blow out fracture
rim and covering the fracture defect is:
with a plastic plate A. Floor
D. Skin incision beneath the eye lash and B. Lateral wall
covering of the fracture defect with a C. Medial wall
plastic plate. D. Roof.
12. In regard to canalicular trauma, all of the 16. Blow out fracture of the orbit involves:
following are true EXCEPT: A. Superior wall
A. One may wait 24 to 48 hours after injury B. Postero-medial part of the orbital floor
to allow soft tissue swelling to decrease C. Medial wall
B. Upper canalicular trauma alone should D. Lateral wall.
never be surgically repaired so as not to 17. True about blow out fracture of the orbit are
risk damage to the remaining nasolacri- except:
mal system A. Herniates into maxillary antrum
C. Silicone stents should be left in place B. Extraocular movements are restricted
for 3 to 6 months C. Looking down is easy
D. Surgical microanastomosis of the cut D. Orbital floor reconstruction is the
canalicular ends with silicone stent intu- treatment.
bation offers the best possibility of suc- 18. Most common cause of fracture of roof of
cessful repair. orbit is:
13. Where do orbital floor fractures most com- A. Blow on back of head
monly occur? B. Blow on the forehead
A. Along the infraorbital canal C. Blow on the parietal bone
D. Blow on upper jaw.
7 Trauma 95

19. A characterestic finding in direct nasa- 25. A patient is brought to the emergency
orbital-ethmoid fracture is department following an automobile acci-
A. Telecanthus dent. Examination shows a periorbital hema-
B. Hypoglobus toma, ophthalmoplegia, ptosis of the upper
C. Infraorbital hypesthesia eyelid, and a fixed dilated pupil on the left.
D. Epistaxis. Consensual light reflex is intact. Which of
20. All of the following findings are consistent the following is the most likely diagnosis?
with an isolated inferior orbital wall frac- A. Orbital apex syndrome
ture and soft issue entrapment except B. Retrobulbar hematoma
A. Subcutaneous emphysema C. Superior orbital fissure syndrome
B. Infraorbital hypesthesia D. Traumatic mydriasis.
C. Horizontal limitation in ocular motility 26. A patient sustains a Le Fort I fracture on the
D. Hypoglobus. left and a Le Fort III fracture on the right
21. Orbital roof fractures are characterized by: in a motor vehicle collision. In this patient,
A. Fractures of the orbital floor or medial which of the following bones is most likely
wall usually occur as well to be fractured on both sides of the face?
B. Tend to occur in adults A. Ethmoid
C. The patient must fall from a height B. Palate
greater than 3 m C. Pterygoid plate
D. An upper eyelid hematoma is a common D. Zygoma.
association. 27. A young boy presents with a history of fall
22. A patient sustains blunt trauma to the face. from a bicycle. A CT scan showed a pure
Examination shows enophthalmos, periorbi- blowout fracture of the left orbital floor
tal ecchymosis, subcutaneous emphysema, with a slight dislocation of the orbital con-
and ipsilateral epistaxis. These findings are tents. The indication to repair this orbital
most consistent with a fracture of the: blowout fracture includes which one of the
A. Anterior wall of the maxillary sinus following:
B. Medial wall of the orbit A. Fracture involving less than half of the
C. Nasal bones orbital floor
D. Zygomatic arch. B. No inferior rectus entrapment
23. A patient is undergoing repair of a commi- C. Inferior rectus weakness
nuted displaced fracture of the left zygoma. D. Pain and oculocardiac reflex on upgaze.
Which of the following landmarks will be 28. Surgical incisions to repair orbital floor
most useful in restoring the zygoma to its fractures include all except:
anatomically correct position? A. Lower eyelid crease incision
A. Frontozygomatic suture B. Lower fornix incision
B. Lateral orbital wall C. Subciliary incision
C. Lateral buttress D. Grey line incision.
D. Medial buttress. 29. Implants used for orbital fracture repair
24. A patient sustains fractures of the right orbit include all except:
and zygoma in a motor vehicle accident. A. Supramid
Which of the following is an indication for B. Porous polyethylene
immediate ophthalmologic consultation? C. Titanium
A. Diplopia D. PMMA.
B. Eyelid ptosis 30. Implants used for orbital fracture repair
C. Hyphema include all except:
D. Subconjunctival hemorrhage. A. Silicone
B. PTFE
96 E. A. El Toukhy

C. Bone graft A. Inferior rectus entrapment


D. Gelfoam. B. Trismus
31. Complete release of entrapped muscle is C. Globe ptosis
confirmed by all except: D. Lateral canthal dystopia.
A. Hess screen 38. Which of the following findings is most com-
B. Diplopia chart monly associated with orbital floor fractures?
C. Forced duction test A. Orbital cellulitis
D. Electromyography. B. Enophthalmos within 24 hours of the
32. By definition, all Le Fort fractures must fracture
extend posteriorly through the C. Rapid improvement in traumatic diplo-
A. Pterygoid plates pia over a 24-hour period
B. Maxillary bone D. Unilateral midfacial sensory loss (V2
C. Zygomaticomaxillary complex distribution unilaterally).
D. Naso-orbital-ethmoidal. 39. A patient is seen 2 months after blunt
33. The optimal time for surgical repair of trauma to the right orbit. The examination
orbital floor fractures is generally consid- is normal except for blepharoptosis on the
ered to be right side. Levator function is normal on
A. Within 24 hours of injury both sides, and the patient states the eye-
B. 1–3 days following injury lid positions were equal on both sides prior
C. 7–14 days following injury to the injury. There is no enophthalmos or
D. 2–6 weeks following injury. diplopia. What is the best next step in man-
34. Regarding surgical repair of a cut aging this patient?
canaliculus: A. Surgical exploration and repair of the
A. After surgical repair, stents are left in levator aponeurosis
place for 3 weeks B. Close observation with no plan for sur-
B. Direct suturing of the cut ends is required gical correction until 3 to 6 months after
C. Irrigation using methylene blue may initial injury
facilitate intraoperative visualization C. Computed tomography (CT) scan to
D. Repair of all such injuries is rule out an orbital fracture
recommended. D. A Tensilon test to rule out new-onset
35. In Traumatic telecanthus, reattachment of myasthenia gravis.
the medial canthal tendon to which of the 40. You are asked to see a patient with nonin-
following structures is necessary for recon- fected second-degree burns to the eyelids.
struction of the normal eyelid anatomy? The emergency room physicians ask what
A. Medial orbital tubercle topical medicine should be placed over the
B. Anterior process of the maxilla burns to prevent scarring. What would the
C. Anterior lacrimal crest most appropriate response be?
D. Posterior lacrimal crest. A. Gentamicin in a water-miscible base
36. The most common cause of indirect trau- B. Silver sulfadiazine 1% cream
matic optic neuropathy is blunt trauma to C. No topical medications
which bone? D. Hydrocortisone 1% ointment.
A. Maxillary 41. A patient is evaluated for trauma to the
B. Zygomatic right orbit. He has marked proptosis and an
C. Frontal intraocular pressure of 40 mm Hg on the
D. Temporal. affected side. A CT scan shows intraorbital
37. Which of the following is least likely to hemorrhage. Which of the following actions
be associated with a zygomaticomaxillary would be the least effective in acutely
complex fracture? reducing intraocular pressure?
7 Trauma 97

A. Lateral canthotomy and cantholysis B. Entrapped muscle


B. Administration of high-dose oral C. Sinusitis
corticosteroids D. Enophthalmos.
C. Administration of topical aqueous 47. If a patient has a ruptured globe in addition
suppressants to a symptomatic blowout fracture, what
D. Administration of intravenous mannitol. should the surgeon do?
42. A young boy sustained a right upper eye- A. Repair both injuries at the same time
lid laceration after falling from his bicycle. B. Repair the ruptured globe immediately
The laceration measures 15 mm and extends and delay the floor exploration for 2 to 4
from the eyelid margin to above the eyelid weeks
crease. There is an avulsed avascular sec- C. Repair the ruptured globe immediately
tion of the laceration superiorly. The results and delay the floor exploration for 2 days
of his ocular examination are normal except D. Wait 48 hours and then repair both inju-
for marked swelling of the eyelid. What ries at the same time.
would the least appropriate action in the 48. A patient has sustained blunt facial and
management of this case be? orbital trauma in a motor vehicle accident.
A. Check the tetanus status The patient has limited supraduction and
B. Discard the avulsed tissue marked upper eyelid edema and ptosis.
C. Repair the eyelid margin A CT scan shows a frontal sinus fracture,
D. Check the status of the levator muscle. orbital roof fracture, and pneumocepha-
43. What is the best way to distinguish a lus. What would the appropriate course of
restrictive motility disorder from a paretic action be?
disorder after a blowout fracture? A. Neurosurgical consultation to rule out
A. Alternate prism cover testing intracranial injury
B. Goldmann single binocular visual fields B. Valsalva to evaluate brain-orbit interface
C. Motility examination of the cardinal C. Forced-duction testing to rule out supe-
positions of gaze rior rectus muscle entrapment
D. Forced-duction testing. D. Superior orbitotomy to rule out a levator
44. Which of the following is not a common laceration.
sign of an orbital blowout fracture? 49. Arteriovenous fistulas that affect the orbit
A. Tenderness over the zygomaticofrontal most commonly develop following what
suture type of trauma?
B. Infraorbital hypoesthesia A. Orbital blowout fracture
C. Restriction of supraduction B. Frontal sinus fracture
D. Enophthalmos. C. Penetrating intacranial trauma
45. What type of orbital blow-out fracture D. Basal skull fracture.
requires urgent referral? 50. Limited downgaze is noted in a patient with
A. Medial wall fractures a known floor fracture. What is the most
B. Fractures involving more than 4 mm of likely cause of the downgaze deficit?
displacement A. Disinsertion of the inferior rectus mus-
C. Small symptomatic fractures in patients cle from the globe
younger than age 18 years B. Orbital edema or hemorrhage affecting
D. Fractures associated with a blood-filled the inferior rectus muscle
maxillary sinus. C. Entrapment of the inferior oblique mus-
46. Which of the following is the most common cle in the floor fracture
indication for repair of medial orbital wall D. Concomitant orbital roof fracture
fractures? with entrapment of the superior rectus
A. Epiphora muscle.
98 E. A. El Toukhy

51. Regarding orbital floor fractures, which is


incorrect:
A. Associated with enophthalmos demand
immediate repair
B. Of the medial wall are associated with
orbital emphysema
C. Associated with hypesthesia of the
cheek and upper gum suggest infraorbi-
tal nerve damage
D. Associated with a positive forced duc-
tion test always have extraocular muscle
entrapped in the fracture.
52. Naso-orbital-ethmoidal fracture is com-
monly associated with:
A. Epiphora
B. Infraorbital nerve hyposthesia
C. Facial nerve paralysis
D. Trismus.

54. Indications for performing the above proce-


dure include all except:
A. Hyphema
B. RAPD
C. Pulsating retinal vessels
D. Elevated IOP.
55. The most important step in repair of a cut
53. Management of the above picture may
canaliculus is:
require all except:
A. Suturing both ends together
A. Transnasal wiring
B. Insertion of a stent
B. Y miniplate
C. Identification of the distal segment
C. DCR
D. Using general anesthesia.
D. Lacrimal stenting.
7 Trauma 99

56. The above injury may require the use of all 60. In adults: the preferred anesthesia used in
except: repair of lid injuries is:
A. Dye A. Topical
B. Local anesthesia B. Local
C. Stent C. Regional
D. Vesicoelastic. D. General.
61. Regarding eyebrow injuries:
A. Are repaired after lid injuries
B. Are repaired before forehead injuries
C. Usually heal well without scarring
D. Always require tattooing later.

57. Repair of the above post traumatic lesion


may require all except:
A. Wiring
B. Bony fixation
C. Lacrimal stent
D. Using a flap.

62. Management of this injury is;


A. Observation
B. Immediate repair
C. Repair within 48 hours
D. Repair within 2 weeks.

58. The three sutures used in the diagram lies at


the level of all except:
A. Lash line
B. Grey line
C. Meibomian orifices
D. Muco-cutaneous junction.
59. Bad repair of the above injury can result in
all except:
A. Lagophthalmos
63. Features of this lesion includes all except:
B. Ectropion
A. Subcutaneous emphysema
C. Conjunctival shortening
B. Epistaxis
D. Lid notching.
100 E. A. El Toukhy

C. Hypesthesia Answers of Trauma


D. Diplopia.
64. Indications to remove an orbital FB include 1 B 21 D 41 B 61 B
all except: 2 D 22 B 42 B 62 A
A. Organic nature
3 C 23 B 43 D 63 C
B. Close to the optic nerve
4 B 24 C 44 A 64 B
C. Associated with diplopia
D. Anterior. 5 D 25 C 45 C 65 D
65. In acute severe traumatic optic neuropathy 6 D 26 C 46 B 66 D
(TON) associated with major head trauma, 7 B 27 D 47 B
which of the following statements is most 8 D 28 D 48 A
likely to be true? 9 C 29 D 49 D
A. Computed tomography (CT) of the 10 B 30 D 50 B
optic canal is commonly abnormal
11 A 31 D 51 D
B. Fundoscopy is commonly abnormal
12 B 32 A 52 A
C. High-dose steroids are indicated
D. Intervention is unlikely to result in 13 D 33 C 53 D
improvement. 14 C 34 D 54 A
66. Which individual variable at presentation 15 A 35 D 55 C
is the strongest predictor of visual outcome 16 B 36 C 56 B
using the ocular trauma score (OTS)? 17 C 37 A 57 D
A. Endophthalmitis
18 B 38 D 58 D
B. Globe rupture
19 A 39 B 59 C
C. Presence of relative afferent pupillary
defect 20 C 40 C 60 C
D. Visual acuity.
The Orbit
8
Essam A. El Toukhy

The orbit is defined by the four bony walls sur- Pain is another common manifestation. The
rounding the eye and all of the contents con- level, character, onset and progression of the
tained within that space. Understanding the pain are helpful in determining the etiology
anatomy of the relevant nerves, extraocular mus- of the orbital process. Pain is most commonly
cles, vascular system, adipose tissue, lacrimal caused by inflammation, either autoimmune or
system, lymphatics, and anterior segment struc- infectious. Orbital inflammatory disease and
tures in addition to the adjacent nasal sinuses orbital cellulitis are examples of these processes.
and cranial cavity is a must. Orbital pathology Both processes cause pain via the inflamma-
can be caused by any process that involves these tory cascade. This is why orbital inflammatory
structures, as well as distant disease processes syndrome is sometimes difficult to differentiate
including metastasis or causing inflammation of from orbital cellulitis when both are unilateral
the orbit. The examination will allow the oculo- and causing pain. Cellulitis will be associated
plastic surgeon to form a differential diagnosis with malaise, and a leukocytosis in most cases.
that will often be refined by imaging and labora- Benign processes generally do not cause pain.
tory studies. Categories include vascular, inflam- Large slow growing benign masses typically
matory, traumatic, autoimmune, metabolic, do not cause severe pain, but a large mass may
infectious, neoplastic, congenital, and endocrine. cause a pressure type or discomfort. Chronic
For many general ophthalmologists, the orbital processes include thyroid eye disease (by
orbital evaluation may seem to involve struc- far the most common cause of proptosis), sar-
tures that are away from a general ophthalmolo- coidosis and granulomatosis with polyangiitis.
gist’s usual structures. However, an organized Thyroid eye disease typically has an indefinite
approach to the exam will make this area feel onset and is slowly progressive over weeks and
familiar. months. Malignant tumors, late in the course,
Proptosis is the commonest orbital manifesta- can invade sensory nerves and cause pain associ-
tion. The ophthalmologist should always check ated with hypesthesia. In some cases the prop-
vision, pupils, pressures, confrontation visual tosis may be present for years, progress very
fields, and eye movements. slowly, and cause no pain (benign mixed tumor
of the lacrimal gland).
Past medical history can be helpful. A his-
tory of any cancer should alert the physician
E. A. El Toukhy (*) to the possibility of a metastasis with lung,
Oculoplasty Service, Cairo University, Cairo, Egypt breast, colon, and prostate cancer being the
e-mail: eeltoukhy@yahoo.com

© The Author(s), under exclusive license to Springer Nature Switzerland AG 2021 101
E. A. El Toukhy (ed.), Oculoplasty for Ophthalmologists, https://doi.org/10.1007/978-3-030-68469-3_8
102 E. A. El Toukhy

most common. Previous surgery and trauma Most periorbital changes can be identified
also notably affect the orbital structures allow- using a pen light. The pen light can help illumi-
ing physicians to contextualize any proptosis nate suspicious skin lesions, check the pupils,
measured. and assess the eye movements including the eye-
An appropriate physical exam includes slit lid excursion. This quick exam would allow to
lamp evaluation of the anterior segment and all identify retraction of the upper eyelids showing
the usual components of a complete eye exam. exposed sclera above the limbus, lid lagophthal-
Unique to the orbital exam is the evaluation of mos in downgaze and temporal flare, all caused
proptosis, periorbital changes, palpation, and by thyroid eye disease. Other pathognomonic
detection of globe pulsation. findings include fullness of the temple in sphe-
Gross proptosis or prominent eyes will be noid wing meningioma, an S-shaped eyelid in
immediately obvious and probably indicates a patient with neurofibromatosis type 1 with a
that the individual has asymmetry outside of the plexiform neurofibroma, or a diabetic patient
range of normal. Quantification of this finding is with routine orbital cellulitis that have a necrotic
performed using the Hertel exophthalmometer. black lesion in the nasopharynx indicating a
A normal range is about 18 mm. phycomycosis. During slit lamp examination
The most important component of meas- we may identify a salmon colored patch of the
urement is asymmetry between the two eyes. conjunctiva confirming diagnosis of lymphoma
A measurement of the tip of the cornea to the of the orbit. During fundus examination we may
lateral orbital rim differing more than 2 mm see ciliary shunt vessels of the optic nerve, con-
between the two eyes should be considered sistent with an orbital meningioma, or the “can-
abnormal. Even in the absence of pain, progres- dle wax dripping” periphlebitis associated with
sion, or relevant past medical history, incidental sarcoidosis.
asymmetry of the orbits that is greater than 2 Palpation of the orbit may identify a specific
mm should be investigated further with imaging mass or a general fullness of the orbit (resistance
if progression is observed. Asymmetry and pro- to retropulsion). Note the position and character
gression are the greatest red flags for proptosis. of any masses. A child presenting with a slow
The types of globe displacement also add growing superotemporal, discrete, smooth mass
additional clues to the etiology of an orbital likely has a dermoid cyst. Upon palpation of the
process. A slow-growing tumor within the mus- skin overlying the lacrimal gland, the patient
cle cone will cause axial proptosis, meaning may note some diminished sensation. This
the eye will be pushed directly forward out of hypesthesia is associated with lacrimal gland
the socket. Masses within the extraconal spaces malignancies deriving from the lacrimal gland
result in displacement of the globe away from a epithelium. The ophthalmologist may identify
mass. A superior mass will result in inferior dis- the heat and pain due to orbital inflammation or
placement of the globe. Lacrimal gland tumors infection, associated with the redness that was
result in inferior and possibly medial displace- noted on inspection. Any possible restriction of
ment of the globe. An exception to this rule is extraocular muscles identified on exam can be
the scirrhous carcinoma breast cancer metas- further tested with forced ductions.
tasis. The sclerosing tumor can cause enoph- Pulsation is a sign suggesting a pulsatile vas-
thalmos. Lateral displacement of the globe is cular lesion such as an AVM or high flow carotid
typically seen in sinus disease including carci- cavernous fistula. It may also be seen after the
noma or a mucocele. Superior displacement of removal of the orbital roof or lateral wall fol-
the globe is relatively rare, but may occur from lowing a sphenoid wing meningioma excision.
maxillary sinus tumor. Interestingly, the most Auscultation can also be applied to the globe
common inferior orbital mass is lymphoma, and orbit where a high-flow fistula may produce
despite the fact that most lymphomas arise in the a bruit, typically accompanied by dilated epis-
superior orbit. cleral vessels.
8 The Orbit 103

The Various Orbital Diagnoses Include improvement with high-dose oral steroids. In
contrast to this immune related process, thyroid
Vascular eye disease presents less acutely, over weeks
Many vascular pathologies affect the orbit. In with slowly progressive signs of inflammation,
children, the infantile hemangioma usually eyelid retraction and possible motility distur-
appears in the first few weeks of life. These bance and proptosis.
hemangiomas grow over a period of several
weeks and involute to a degree over 5–10 years. Metabolic
In contrast to this childhood vascular lesion, In rare instances, fluid shifts in burn victims
adults may present with slowly progressive axial can result in orbital compartment syndrome
proptosis due to a cavernous vascular malfor- following aggressive IV rehydration, result-
mation in the muscle cone. Following trauma, ing in a similar picture as that of a retrobulbar
a direct high flow carotid cavernous fistula can hematoma.
occur, typically of acute or subacute onset asso-
ciated with pain. Accompanying symptoms usu- Iatrogenic
ally include high intraocular pressure, unilateral Surprisingly some proptosis can be purpose-
proptosis, and acute progression. Indirect fis- ful and beneficial cosmetically. Orbital volume
tulas can occur spontaneously in older adults. augmentation with implants or fillers can restore
These have low flow and are more likely to have ocular prominence. Intraorbital implants are
normal intraocular pressure and slower progres- commonly used to restore symmetry. Filler is
sion without a bruit. rarely used to improve enophthalmos.

Infection Neoplasm
Orbital bacterial cellulitis presents acutely with Benign and metastatic tumors vary significantly
pain, unilateral proptosis, and rapid progres- in their presentation. Pleomorphic adenoma of
sion. There is tenderness on palpation with nota- the lacrimal gland may slowly progress over
ble edema and erythema, often with induration. many months to result in non-axial inferonasal
Fungal cellulitis occurs in immunosuppressed displacement of the globe while rhabdomyosar-
patients. The inadequate immune response coma in a child could have an aggressive orbital
allows the normally docile fungus to infect cellulitis-type presentation with onset over days.
the orbital tissues. In this case, typical signs of Differentiation within the category of neoplasm
orbital inflammation are not present due to the typically requires imaging.
lack of normal immunocompetency.
Congenital
Trauma A dermoid cyst can be differentiated from an
Proptosis resulting from trauma is caused by a encephalocele by location and progression.
retrobulbar hematoma resulting in pain of rapid Most dermoid cysts arise laterally from within
onset with periorbital ecchymosis, edema, che- the frontozygomatic suture. Encephaloceles are
mosis, and vision loss due to high intraocu- present medially and often increase with the
lar pressure or stretching of the optic nerve if Valsalva maneuver.
severe.
Endocrine
Autoimmune Thyroid eye disease (TED) is an autoimmune
This etiology includes the continuum of idi- and endocrine related disease due to its antibody
opathic orbital inflammatory syndrome to IgG acting directly on fibroblasts that result in fibro-
antibody-mediated inflammatory disease. It sis of muscles and differentiation into adipo-
occurs and typically results in unilateral prop- cytes in the orbit. It is the most common cause
tosis with pain, acute onset, chemosis and injec- of bilateral and unilateral proptosis in adults,
tion, a lack of response to antibiotics, and rapid which typically presents bilaterally with many
104 E. A. El Toukhy

periorbital changes including eyelid retraction, phase. Contrast-enhanced CT examination


eyelid flare, proptosis, and has both an acute and is indicated if an orbital mass is suspected to
chronic form. allow differentiation of different orbital lesions
Imaging of the orbit is often indicated for according to their enhancement pattern. Non-
further evaluation of proptosis. Radiological contrast CT examination is performed in cases
imaging is an essential diagnostic tool in the for orbital trauma or thyroid orbitopathy as
oculoplastic specialty, particularly in orbital and these pathologies are very well delineated by
lacrimal lesions as the pathology cannot be vis- hypodense orbital fat.
ualized from outside. Moreover, it can be used It is crucial to note that, CT of the orbits is
in the management plan and follow up of the not performed both without and with contrast
patients. administration; as it does not yield significant
diagnostic improvement, with the risk of dou-
bling the radiation dose to the lens.
Imaging Modalities The eye is a sensitive organ to radiation, and
exposure to higher doses of radiation may lead
1. Ultrasound (US) to the development of early cataract. The risk of
Ultrasound (US) with Doppler can be used in radiation exposure is especially important in the
the diagnosis and follow up of various globe and pediatric patients with special attention in chil-
orbital lesions. It is the first imaging modality dren having malignant lesions who require long-
in children with superficial lesions. The tech- term follow-up imaging to assess response after
nique is non-invasive, cost-effective and easy to therapy and to diagnose early recurrence.
perform, and has a high accuracy for the char- CT angiography (CTA) and CT venography
acterization of vascular lesions. However, it is (CTV) are helpful for assessment of orbital vas-
operator dependent and cannot visualize the cular lesions. Also, it can be useful in planning
orbit completely. Also, it is contraindicated if further evaluation and management of these
rupture globe is suspected.
lesions.

2. Computed Tomography
3. Magnetic Resonance Imaging
CT is the initial imaging modality for the evalu-
MRI is the modality of choice for evaluating
ation of orbital trauma; infection and detection
most of the orbital lesions, particularly in non-
of a foreign body. It is superior to MRI in the
emergency settings, with imaging of patients
detection of calcification, or acute hemorrhage;
presenting with subacute or chronic symptoms.
evaluation of orbital osseous lesions; as well as
MRI has higher soft tissue resolution and tis-
the assessment of orbital soft-tissue lesion with
sue characterization compared to CT and pro-
suspicion of bony erosion. CT is preferred in the
vides more precise delineation of the different
setting of an emergency or if there is a contrain-
dication for MR examination. orbital compartments. Thus, MRI is preferred
Thin slice multi-detector CT scan of the orbit in the evaluation of suspected orbit neoplasms,
provides rapid volumetric image acquisitions. orbit inflammatory disorders, orbit vascular
Coronal and sagittal reconstructed images are malformations, and optic nerve sheath complex
routinely obtained in bone and soft tissue win- lesions. Moreover, MR is ideal for visualiza-
dows. 3D reconstructed images are beneficial tion of intracranial extension of orbital lesions,
in the assessment of complex orbital fractures, as well as lesions at the orbital apex, optic canal
orbito-cranial masses, fibrous dysplasia or and cavernous sinus.
neurofibromatosis. However, MR is more expensive with
CT scan of the orbits is usually performed longer examination time compared to CT. It
following the intravenous (IV) administration requires sedation in some patients; additionally,
of an iodinated contrast medium in the venous it is contraindicated in patients with a cardiac
8 The Orbit 105

pacemaker, aneurysmal clip, or metallic foreign However, because of its invasive nature, it is
bodies. only indicated in selected cases. It is used as a
The standard protocol for MRI orbit exami- problem- solving tool when the findings of ini-
nation is to acquire both unenhanced and tial CT or MR angiography examination are
enhanced imaging after IV administration of unclear. Also, it provides intra-procedural guid-
gadolinium contrast medium. Unenhanced ance of endovascular treatment.
MRI examination is performed alone, if there Before we review any type of scan, con-
is a contraindication to gadolinium IV admin- sider your original exam. When the patient has
istration like renal failure, contrast allergy or globe displacement, investigate the area of the
pregnancy. scan where we could imagine a mass pushing
MRA and MRV examinations can be used the globe. If we have identified a mass, then the
in imaging of orbital vascular lesions; however, next consideration is if it is discrete or infiltra-
they are more susceptible to artifacts and lower tive. Erosion of the bone is more indicative of a
spatial resolution than CTA and CTV studies. malignant process. A fossa that is formed from
However, they can be done without contrast a mass or from the pressure of long-standing
medium, with no risk of ionizing radiation, thus thyroid eye disease is indicative of a chronic
they can be used in patients who cannot tolerate process and usually represents a benign pro-
iodinated contrast material. cess. Generally smaller lesions are better lesions
Diffusion-weighted imaging (DWI) is based but are not necessarily prognostic. Therefore, a
upon assessing the random Brownian motion of small, well circumscribed, homogeneous, mass
water molecules within the tissue. A lesion with indenting but not eroding the bone is likely a
high cellularity demonstrates restricted diffusion benign process. Most other tumors and masses
and low apparent diffusion coefficient (ADC) are diagnosed with incisional biopsy except
value. The use of DWI has been reported to fur- where removal of the entire lesion is easier such
ther increase the diagnostic utility of MRI in as a cavernous hemangioma or a dermoid cyst.
the characterization of orbital masses. Tumors A “biopsy” in those cases would be curative
composed of tightly packed cells with a high for both lesions. The incisional biopsy allows
nuclear-to-cytoplasmic ratio like lymphoma for minimal damage to surrounding structures
show restricted diffusion. Additionally, DWI is while providing specific information or further
valuable in differentiating abscess from other non-surgical intervention including radiation,
inflammatory processes as the thick purulent chemotherapy, and the more recent treatment of
material in an abscess demonstrates diffusion checkpoint inhibitors. A growing list of targets
restriction. allows the native host immune system to attack
cancer cells with a high degree of specificity
4. Conventional Angiography based on tissue DNA testing. This has resulted
Conventional angiography has advantages over in multiple cases that in previous decades would
CTA and MRA examinations, as it is real-time have been treated only with exenteration but can
imaging that provides better temporal resolution now be approached differently.
with evaluation of blood flow dynamics.
106 E. A. El Toukhy

The Orbit

1. This lesion is characteristic for:


A. Leukemia
B. Lymphoma
C. Sarcoidosis
D. Steven Johnson syndrome.
8 The Orbit 107

2. The child has proptosis directed down and


out. Possibilities include all except:
A. Mucocele
B. Meningocele
C. Dacryocele
D. Subperiosteal abscess.
108 E. A. El Toukhy

3. The above lesion is: C. Hemangioma


A. Rhabdomyosarcoma D. Mucocele.
B. Dermoid cyst

4. The above lesion is: C. Hemangioma


A. Rhabdomyosarcoma D. Mucocele.
B. Dermoid cyst
8 The Orbit 109

5. The above lesion is: C. Cavernous hemangioma


A. Rhabdomyosarcoma D. Optic nerve meningioma.
B. Optic nerve glioma

6. The above lesion enlarges during upper res- B. Lymphangioma


piratory tract infections. The most probable C. Hemangiopericytoma
diagnosis is: D. Lymphoma.
A. Hemangioma
110 E. A. El Toukhy

7. The hyperdense lesion in the CT points to B. Lymphangioma


the diagnosis of: C. Hemangiopericytoma
A. Hemangioma D. Venous malformation.

8. The above lesion is characteristic of: C. Orbital cellulitis


A. Thyroid ophthalmopathy D. Cavernous sinus thrombosis.
B. Idiopathic orbital inflammation
8 The Orbit 111

9. The above lesion is characteristic of: C. IgG4 orbital inflammation


A. Thyroid ophthalmopathy D. Cavernous sinus thrombosis.
B. Idiopathic orbital inflammation

10. The enlarged superior ophthalmic vein B. Idiopathic orbital inflammation


points to the diagnosis of: C. IgG4 orbital inflammation
A. Thyroid ophthalmopathy D. Cavernous sinus thrombosis.
112 E. A. El Toukhy

11. The above lesion is characteristic of: 14. Orbital tuberculosis, one is false:
A. Frontal sinus mucocele A. Usually unilateral
B. Lacrimal gland tumor B. Mimic orbital malignancy
C. Fibrous dysplasia C. Chronic drainage fistula may be the pre-
D. Eosinophlic granuloma. senting sign
12. Regarding microphthalmia, one is false D. Treated surgically.
A. All children with micophthalmia have 15. Regarding orbital decompression in thyroid
hypoplastic orbit eye disease, one is false:
B. Most microphthalmic eyes have no A. Indicated to restore globe position even
potential for vision if there is no sight threatening conditions
C. Enucleation is nessesary for fitting of B. Indicated if radiotherapy was not
ocular prosthesis effective
D. Dermis fat graft may grow resulting in C. Fat versus bone decompression is based
progressive socket expansion. on patient’s age
13. A patient with subperiosteal abscess can be D. Orbital surgery should precede strabis-
managed by observation in the following mus surgery.
situation except: 16. During decompression of orbital floor,
A. Patient younger than 9 years diplopia and dystopia can be minimized by
B. Medial location of subperiosteal preserving:
abscess A. The palatine bone
C. Presence of gas in the abscess on CT B. The bone between the medial wall and
scan the floor
D. Associated with isolated ethimoidal C. The zygomatic bone
sinusitis. D. The ethmoidal bone.
8 The Orbit 113

17. Nonspecific orbital inflammation, one is C. There is a strong correlation between


false: mitotic rates and the clinical behaviors
A. The muscle insertion is involved in 50% D. It appears as a bluish encapsulated mass
of cases during surgery.
B. The ring sign in B-scan ultrasound indi- 23. True statement regarding lymphatic
cates presence of posterior scleritis malformation(lymphangioma) is:
C. Rapid and favorable response to steroid A. Present since birth
indicates a cell mediated pathology B. Contain both arterial and lymphatic
D. It is a diagnosis of exclusion. component
18. Clinical features of nonspecific orbital C. Does not proliferate
inflammation in children, one is false: D. Intralesional sclerosing agents have no
A. One third of cases are bilateral role in management.
B. Mostly associated with systemic 24. The best treatment for a patient with
disorders decreased visual acuity due to optic sheath
C. Headache and abdominal pain are a meningioma that does not extend outside
common finding the orbit is:
D. Uveitis maybe the presenting sign. A. Systemic corticosteroid
19. Clinical features of infantile capillary B. Fractional stereotactic radiation
hemangioma, one is false: C. Proton pump radiation
A. Start to involute after the first year of D. Exenteration.
life 25. The best approach to an intraconal orbital
B. More common in females tumor is:
C. MRI finding show intralesional vascular A. Trans caruncular orbitotomy
channels with low blood flow B. Vertical eye lid splitting orbitotomy
D. Can be a part of Kasabach–Merritt C. Medial orbitotomy
syndrome. D. Lateral orbitotomy.
20. The management paradigram for infantile 26. Regarding exenteration, one is false:
capillary hemangioma include, one is false: A. Considered in management of recurrent
A. Beta blockers non responding rhabdomyosarcoma to
B. Systemic steroids radio and chemotherapy
C. Sclerosing agents B. Total exentration is the removal of all
D. Surgery. intraorbital soft tissue with or without
21. Regarding cavernous hemangioma, one is the skin of eye lids
false: C. Fixating of ossoointegrated implant is
A. The most common benign neoplasm of achieved with glue
the orbit in adult females D. The exenteration prosthesis usually do
B. May present with increased intraocular not blink or move.
pressure 27. Regarding exophthalmometer the most
C. Arteriography and venography are not appropriate statement is:
useful in the diagnosis A. Is not useful in pulsatile proptosis
D. Radiodense phelebliths are present in B. Normal distance between the apex of
early growing lesions. the cornea and lateral orbital wall rim is
22. Features of hemangiopericytoma, one is <20 mm
false: C. A difference between the two eyes of <4
A. It is a hypercellular and hypervascular mm is considered normal
mass lesions D. Most common cause of bilateral propto-
B. Benign lesions may recur and metastasize sis is orbital pseudotumor.
114 E. A. El Toukhy

28. Regarding the anatomy of the optic canal 34. Mucosa associated lymphoid tissue (MALT):
one of the following statement is true: A. Accounts for less than 10% of orbital
A. Entrance to the optic canal lies inside lymphoma
annulus of Zinn. B. Possible association with chronic chla-
B. Medial wall of the canal is the medial mydia infection
wall of the sphenoid. C. Has no malignant potential
C. Optic nerve leave the optic canal to D. Systemic association is rare.
enter the anterior cranial fossa 35. Regarding the orbital wall structure the
D. Optic nerve injury is due to absence of most appropriate statement is:
periorbita in the canal. A. The orbital roof is made of the frontal
29. Good reflectivity and dancing spikes along bone and ethmoidal bone
with good sound transmission is seen in B. Separates the anterior cranial fossa from
orbital B-scan in which tumor? the orbit
A. Cavernous haemangioma C. The superior orbital fissure is bounded
B. Pleomorphic adenoma superiorly by the lesser wing of the
C. Adenoid cystic carcinoma sphenoid
D. Lipodermoid. D. The optic canal transmits the ophthal-
30. What is the volume of the orbit? mic vein.
A. 10 ml 36. Regarding orbital metastatic tumors in
B. 20 ml adults, the least likely statement is:
C. 30 ml A. Breast and lung cancer account for the
D. 50 ml. majority of orbital metastases
31. One of the following statements is incorrect B. Pain is a frequent presentation
regarding the orbital septum: C. 75% of patients have a history of known
A. Is separated from the levator aponeuro- primary tumor
sis by orbital fat D. Extraocular muscle involvement is rare.
B. Is firmly attached to Whitnall’s ligament 37. Which is the most common orbital encapsu-
C. Fuses with the capsulopalpebral fascia lated tumor?
in the lower lid A. Lacrimal gland tumor
D. Inserts on the levator aponeurosis about B. Cavernous haemangioma
3–5 mm above the tarsal plate. C. Intraorbital dermoid cyst
32. In the management of malignant lacrimal D. Lymphangioma.
gland tumor the least appropriate statement 38. Management of Rhabdomyosarcoma in a 6
is: years old boy usually involves which one of
A. Percutaneous biopsy is contraindicated the following?
B. Perineural extension into the cavernous A. Enucleation and orbital radiation
sinus is common B. Lumbar puncture to rule out central
C. Surgical debulking with intracarotid nervous system metastasis
chemotherapy is an option C. Systemic chemotherapy and orbital
D. Exenteration is required. radiation
33. One of the following is true regarding D. Exenteration of the orbit.
osteoma: 39. Which one of the following statements is
A. Premalignant tumor true in regards to unilateral exophthalmos in
B. Originates from mesodermal tissue a child?
C. Rapidly growing tumor A. Capillary hemangiomas are the most
D. Surgical excision is the first line of common benign primary orbital tumors
management. in children
8 The Orbit 115

B. Optic nerve meningiomas are more 44. Presence of sudden enophthalmos without
common than gliomas in children history of a previous injury involving the
C. Neurofibroma is the malignant tumor orbit in adults is suspicious for:
that most commonly produces exoph- A. Cavernous hemangioma
thalmos in children B. Orbital cellulitis
D. Thyroid ophthalmopathy is the most C. Thyroid-related orbitopathy
common cause of unilateral exophthal- D. Metastatic breast cancer in a woman.
mos in children. 45. A 63-year-old diabetic patient presented
40. Examination of a 70-year-old patient with with pain in the right eye, redness, and
a progressively enlarging mass in the left swelling one month earlier. Examination
inferior orbit reveals a “salmon patch” revealed severe right proptosis, eyelid
appearance of the inferior fornix. The most edema, limited movement in all directions
likely diagnosis is: and necrotic crust and lesions on the hard
A. Melanoma palate and nasal passages. The proper treat-
B. Sebaceous carcinoma ment of this condition includes all of the
C. Lymphoma following EXCEPT:
D. Reactive lymphoid hyperplasia. A. Radiation of the orbits
41. A 1-year-old girl with a round, well-demar- B. Amphotericin B for 6 weeks
cated mass at the superotemporal rim that C. Stabilizing the underlying disease process
has been there since birth. The most likely D. Debridement of all devitalized tissue,
diagnosis is: including Exenteration if necessary.
A. Rhabdomyosarcoma 46. A 56-year-old woman presents with bilat-
B. Capillary hemangiomas eral proptosis, double vision, and chemosis.
C. Neurofibroma She has bilateral lid retraction and lid lag.
D. Dermoid cyst. The most common recommended surgical
42. A 63-year-old patient has a 3 years history order of therapy is:
of steadily progressive bilateral painless A. Orbital decompression, strabismus sur-
proptosis and visual loss. Ct scan showed gery, and eyelid retraction surgical repair
bilateral orbital infiltrates. Biopsy of the B. Eyelid retraction surgery, orbital
orbital infiltrates was reported as “reactive decompression, and strabismus surgery
lymphoid hyperplasia.” The most appropri- C. Orbital decompression and eyelid
ate treatment is which one of the following? retraction surgery repair
A. Radiation with a dose of 1500–2000 D. Eyelid retraction surgery, strabismus
ncGy surgery, and orbital decompression.
B. Systemic steroids 47. A 55-year-old patient presents with gradual
C. Complete surgical excision painless proptosis in the left eye. On exami-
D. Systemic chemotherapy. nation, visual acuity is normal; the left
43. A 56-year-old man presents with bilat- globe is displaced inferiorly and medially
eral proptosis, double vision and chemo- and a firm lobular mass is palpated near the
sis. Which one of the following features superior lateral orbital rim. CT of the left
on CT orbits distinguishes thyroid related orbit showed a lacrimal gland mass with no
orbitopathy from orbital inflammatory bony erosion. The next step in the manage-
syndrome? ment of this patient would be:
A. Periorbital soft tissue edema of the lids A. Incisional biopsy
B. Enlarged extraocular muscle B. Metastatic work-up
C. Absence of a thickened extraocular C. A 2-week course of systemic
muscle tendon corticosteroids
D. Enlarged lacrimal glands. D. Excisional biopsy.
116 E. A. El Toukhy

48. A 10-year-old patient with painless out- examination, visual acuity is normal OU;
ward protruding of the right eye combined there is 3 mm of proptosis on the left and
with loss of vision in this eye for 3 months. arterialized conjunctival and episcleral ves-
Which one of the following radiological sels. IOP is 30 mmHg in OS and 12 mmHg
features is considered pathognomonic for in OD. Arterialization of vessels is most
optic nerve glioma? likely caused by disturbance in which of the
A. Multiple cystic cavities within optic following structure?
nerve A. Intracranial ophthalmic artery
B. Kinking of the optic nerve B. Intra-orbital central retinal artery
C. “Tram-track” enlargement of the optic C. Meningeal branches of Internal Carotid
nerve Artery
D. Adjacent bony erosion. D. Cervical common carotid artery.
49. A 36-year-old patient with a presumed diag- 53. Which of the following is not a major crite-
nosis of Idiopathic Orbital Inflammatory rion for the diagnostic of Neurofibromatosis
Syndrome (IOIS) was treated with 60 mg type I?
of oral prednisone for 2 weeks with no A. Optic nerve glioma
improvement. What would be the next step B. Lisch nodules
in the management of this case? C. Posterior subcapsular cataract
A. Orbital irradiation D. Plexiform neurofibromas.
B. Induction with intravenous 54. Surgical spaces of the orbit include all
methyl­prednisolone except one:
C. Oral cyclophosphamide A. The sub-periorbital surgical space,
D. Orbital biopsy. which is the potential space between the
50. A 6-year-old boy presents with redness of bone and the periorbita
his left eye. On examination, the skin of the B. The extraconal surgical space, which
upper and lower eyelid is red and inflamed, between the periorbita and the muscle
no orbital tenderness, visual acuity is nor- cone
mal in both eyes, he is orthotropic in primary C. The intraconal surgical space, which
position, and Hertel exophthalmometry reads lies within the muscle cone
16 mm in the right eye and 18 mm in the left D. Sub-Conjunctival surgical space, which
eye. Which of the following examination lies between the conjunctiva and the
findings that best differentiates between the Tenon.
diagnoses of preseptal and orbital cellulitis? 55. Orbital cellulitis, one is false:
A. External examination findings (ery- A. Implies active infection of the orbital
thema, warmth) soft tissue anterior to the orbital septum
B. Ocular motility findings B. 90% of cases of orbital cellulitis occurs
C. Exophthalmometry as a secondary extension of acute or
D. Fever. chronic bacterial sinusitis
51. Which one of the following is NOT a typi- C. Delay in treatment may result in devel-
cal manifestation of Idiopathic Orbital opment of cavernous sinus thrombosis
Inflammatory Syndrome? D. Decreased vision and pupillary abnor-
A. Peripheral ulcerative keratitis malities suggest involvement of orbital
B. Dacryoadenitis apex.
C. Orbital myositis 56. Optic nerve gliomas, one is false:
D. Optic perineuritis. A. Occur predominantly in children in the
52. A 70-year-old female, with no history of second decade of life
trauma, complains of mild redness and B. The chief clinical feature is gradual,
irritation of her left eye for 2 months. On painless unilateral and axial Proptosis
8 The Orbit 117

C. In the majority of cases are self- limited C. The classic radiographic appearance
and show minimal growth of the lesion on computed tomography
D. Diagnosis of these tumors can usually (CT) scanning is a fusiform enlarge-
be established by orbital CT scan. ment of the optic nerve
57. Regarding Rhabdomyosarcoma, one is true: D. Tumors arising from the optic nerve
A. The most common secondary orbital have a poorer prognosis than those aris-
malignancy in childhood ing from the optic chiasm.
B. The average age of onset is 3 years 63. Which one of the following statements
C. The tumour is usually retrobulabar about ocular adnexal dermoid cysts is false?
D. A biopsy must be done immediately, A. They are choristomatous arrests of epi-
usually via an anterior orbitotomy. thelial tissue
58. Lacrimal gland tumour, one is false: B. The most common location is the supe-
A. Clinically, the large majority of lacrimal ronasal orbital rim
gland tumors will represent idiopathic C. Generally, they do not enlarge after the
inflammatory disease first year of life
B. Usually are not responding to anti- D. Radiography of orbital lesions generally
inflammatory medication demonstrates bony excavation.
C. Majority of them do not require surgical 64. Regarding MRI in orbital disease, one is
intervention false:
D. Majority of cases do not require biopsy. A. No view of bone or calcium
59. Which one of the following statements B. Less soft tissue detail
about capillary hemangiomas is false? C. Multiple planes can be imaged at once
A. Systemic interferon may lead to D. Better for orbitocranial junction or
involution intracranial problem.
B. They are more common in girls than in 65. Capillary hemangioma, one is false:
boys A. Are common primary benign tumours
C. They characteristically blanch with of the orbit in children
pressure B. Manifest primarily in the second year of
D. MRI findings show intralesional vascu- life
lar channels with low blood flow. C. The majority of capillary hemangiomas
60. The epibulbar lesion most commonly seen are superficial
in children younger than 15 years is: D. Ophthalmic indications for treatment are
A. Dermoid anismetropia, strabismus and amblyopia.
B. Pyogenic granuloma 66. Clinical feature of infantile capillary
C. Nevus hemangioma, one is true:
D. Epithelial inclusion cyst. A. Start to involute about the 3rd year of
61. In childhood orbital cellulitis, the least life
important thing is: B. More common in males
A. Orbital imaging C. MRI finding show intralesional vascular
B. Hospitalization channels with high blood flow
C. Isolation of the infectious organism D. Sclerosing agents are included in the
D. Pediatric ENT consultation. management paradigram.
62. Which of the following statements about 67. The first line of treatment of optic nerve gli-
optic nerve glioma is false? oma is:
A. The age range with the highest inci- A. Observation
dence is 2–6 years B. Surgical excision
B. Two common means of presentation C. Radiotherapy
include visual loss and proptosis D. Chemotherapy.
118 E. A. El Toukhy

68. In the evaluation of a child with unilateral C. Post radiation


exophthalmos, which assumption is correct: D. Severe vasculitis.
A. Cavernous hemangiomas are the most 74. Regarding optic nerve glioma one of the
common benign orbital tumour in following statements is true:
children A. Predominantly seen in adults
B. Thyroid ophthalmopathy is the most B. 90% associated with neurofibromatosis
common cause of unilateral exophthal- C. MRI can show cystic degeneration
mos among children D. Expands within the optic nerve with no
C. Optic nerve meningiomas are more dural expansion.
common than gliomas in children 75. 45 years old diabetic presented with one day
D. Proptosis is an unusual presentation of history of right upper lid swelling, examina-
nonspecific orbital inflammatory disease tion reveals normal visual acuity with promi-
in children. nent upper lid swelling, mild restriction of
69. Cavernous sinus thrombosis is often charac- adduction and elevation with 2mm proptosis,
trized by, one is false: the step that can be deferred to later would be:
A. Rapid progression of the Proptosis A. Admission and ordering of CT orbits
B. Development of neurological signs B. Starting IV antibiotics
C. Meningitis C. Blood cultures
D. Lumbar puncture may show chronic D. Orthoptic evaluation of III nerve palsy.
inflammatory cells. 76. The least possible complication of orbital
cellulitis is:
70. Which of the following is NOT a known
A. Ethmoidal sinusitis
cause of pulsatile proptosis?
B. Cavernous sinus thrombosis
A. Neurofibromatosis type 1
C. Brain abscess
B. Orbital roof fracture
D. Neurotrophic keratitis.
C. Orbital meningoencephalocele
77. One of the followings is not a surgical space
D. Thyroid-associated orbitopathy.
of the orbit
71. The most common neoplasm of the lacrimal
A. The Extraconal surgical space
gland is:
B. The Subperiosteal space
A. Lymphoproliferative disorders
C. The supra-arachnoid space
B. Adenoid cystic carcinoma
D. The episcleral surgical space.
C. Pleomorphic adenoma
78. Meningioma of the optic nerve is character-
D. Malignant mixed tumour.
ized by the following Except:
72. With regards to vascular tumours of the eye-
A. Typically compresses the junction of
lids and orbit, one of the following is TRUE:
chiasm with optic nerve
A. Cavernous haemangioma causes grad-
B. Early visual field affection
ual myopic shift
C. May cause loss of smell
B. Capillary hemagioma blanches with
D. Early limitation of eye movement.
pressure
79. The majority of orbital lymphomas are:
C. Orbital varix causes proptosis at
A. Polyclonal proliferation
rest which is reduced by Valsalva
B. T-cell tumors
manoeuvre
C. Well differentiated
D. Dural-cavernous fistula has a high
D. Bilateral on presentation.
blood-flow rate.
80. Pain and perineural invasion are associated
73. A high flow carotid cavernous fistula in a
with;
young man is likely to be due to:
A. Rhabdomyosarcoma
A. Congenital arteriovenous malformation
B. Adenoid cystic carcinoma of the lacri-
B. Head trauma
mal gland
8 The Orbit 119

C. Esthisioneuroblastoma C. Rapid reduction of systemic steroids


D. Optic sheath meningioma. may cause recurrence of inflammatory
81. Corticosteroids are used in treatment of all signs and symptoms
except; D. It is usually bilateral in adults.
A. Orbital lymphoma 87. Capillary hemangioma
B. Orbital mucocele A. Enlarges dramatically during second
C. Non specific orbital inflammation year of life
D. Wegener’s granulomatosis. B. More common in the medial upper eyelid
82. What is the optimal imaging technique for a C. All should be treated as early as possible
posterior optic nerve glioma? D. Seclerosing solution is highly
A. Plain X-ray films recommended.
B. Computerized tomography 88. Orbital computed tomography scanning of a
C. Magnetic resonance imaging patient with a dural cavernous sinus fistula
D. Ultrasound. is likely to show enlargement of which one
83. A patient with an optic-nerve sheath menin- of the following blood vessels?
gioma confined to the orbit has visual acuity A. Central retinal vein
of 20/30. What would be the best treatment B. Pteregopalatine venous plexus
at this stage? C. Superior ophthalmic vein
A. Radiation therapy D. Inferior ophthalmic vein.
B. Systemic steroid therapy 89. A pink-colored mass in the conjunctival for-
C. Chemotherapy nix which appears as a salmon patch is typi-
D. Observation. cal for;
84. In management of dermoid cysts of the A. Systemic lupus erythematouses
orbit the most important to be considered is: B. Orbital lymphoma
A. Filling in any bony defects with bone C. Lymphangioma
substitute D. Xanthogranuloma.
B. Removal of all cyst wall and cyst 90. Orbital lymphoma is most commonly
content located in;
C. All dermoids even if asymptomatic A. The extra ocular muscles
should be removed as early as possible. B. Retro orbital fat
D. Deffering surgery till age 18 to avoid C. Lacrimal fossa
bony deformity. D. Orbital apex.
85. Regarding dermoid cysts; 91. Subperiosteal abscess of the orbit in adults
A. Contain no dermal appendages is more likely than in children to;
B. Preseptal orbital dermoids are most com- A. Drain spontaneously
monly found in the medial upper eye lid B. Respond to single antibiotic therapy
C. Are lined by keratinizing epidermis C. Be Polymicrobial
D. Are always fixed to periosteum. D. Arise from ethmoidal sinus.
86. Regarding non specific orbital inflamma- 92. Compared to CT scanning, MRI provide
tion, which is false; better;
A. It is a diagnosis of exclusion A. View of bone and calcium
B. It should be differentiated from thyroid B. View of orbital apex and orbito-cranial
eye disease junction
120 E. A. El Toukhy

C. Elimination of motion artifact C. It passes backward in the orbit between


D. Safety to patient with prosthetic the levator and the superior rectus muscle
implants. D. It does not receives the central retinal
93. A healthy 6 years old child presents with vein.
proptosis of the left eye, family phtographs 98. The most common predisposing factor of
reveal same prominence of the eye for the orbital cellulitis is:
past year. One week prior to presentation A. Dacryocystitis
the child had seizures. Fundus examination B. Dental infection
reveals choroidal folds in the left eye, which C. Sinusitis
one of the following diagnostic test is least D. Trauma.
useful to be done; 99. Limitation of ocular motility occurs in all
A. Orbital ultrasound except
B. MRI A. Orbital margin fracture
C. Fluorescen angiography B. Cavernous sinus thrombosis
D. CT scan. C. Orbital cellulitis
94. On a T2 weighted MRI, which would D. Thyroid orbitopathy.
appear hyperintense? 100. In orbital cystic lesions only one of the
A. Fat following statements is true;
B. Blood in the carotid A. Dermoid cyst is a hamartoma
C. Bone B. Deep dermoid cysts typically present
D. Vitreous. in infancy
95. Which structure and its bony framework are C. Frontal mucocele may invade the orbit
paired INCORRECTLY? D. Posterior encephalocele is character-
A. Lacrimal sac fossa—lacrimal and max- ized by pulsating proptosis and a bruit.
illary bones 101. Medial wall of the orbit is formed by all
B. Optic canal—greater and lesser wings except:
of the sphenoid bone A. Ethmoid bone
C. Inferior orbital fissure—maxilla, B. Sphenoid bone
zygomtic bone, palatine bone, and C. Frontal bone
greater wing of the sphenoid bone D. Lacrimal bone.
D. Anterior and posterior ethmoidal 102. Site of entry of inferior division of oculo-
­foramen—ethmoid and frontal bones. motor nerve into the orbit is:
96. Which one of the following is the most A. Inferior orbital fissure
common primary malignancy of the orbit in B. Superior orbital fissure
children? C. Foramen lacerum
A. Neuroblasroma D. Foramen rotundum.
B. Rhabdomyosarcoma 103. Structures passing through superior orbital
C. Ewing’s sarcoma fissure include all except:
D. None of the above. A. II nd cranial nerve
97. The following is TRUE about the superior B. III rd cranial nerve
ophthalmic vein: C. IV th cranial nerve
A. It is the main venous channel of the D. VI th cranial nerve.
orbit 104. Purulent inflammation of the tissues of the
B. It is formed by the union between the orbit is called:
facial vein and the temporal vein A. Orbital cellulitis
B. Endophthalmitis
8 The Orbit 121

C. Panophthalmitis 112. A 19 years old young girl with previous


D. Dacryocystitis. history of repeated pain over medial can-
105. Which of the following conditions causes thus and chronic use of nasal decongest-
pseudoproptosis? ants presented with abrupt onset of fever
A. Hyperthyroidism and chills and rigors, diplopia on lateral
B. Orbital pseudotumour gaze, moderate proptosis and chemosis.
C. High myopia On examination, optic disc is congested.
D. Optic nerve glioma. Most likely diagnosis is:
106. Most common cause of unilateral propto- A. Cavernous sinus thrombosis
sis in adults: B. Orbital cellulitis
A. Thyroid ophthalmopathy C. Acute ethmoidal sinusitis
B. Rhabdomyosarcoma D. Orbital apex syndrome.
C. Orbital cellulitis 113. All of the following could result from
D. Orbital blow out fracture. infection of the right cavernous sinus
107. Most common cause of unilateral propto- except:
sis in all age groups: A. Constricted pupil in response to light
A. Thyrotoxicosis B. Engorgement of retinal veins seen on
B. Retinoblastoma ophthalmological examination
C. Intraocular haemorrhage C. Ptosis of right eyelid
D. Raised lOP. D. Right ophthalmoplegia.
108. Most common cause of bilateral proptosis 114. A retrobulbar intraconal mass with well
in children: defined capsule presenting with slowly
A. Rhabdomyosarcoma progressive proptosis in the 2nd to 4th
B. Lymphoma decade is most likely:
C. Retinoblastoma A. Capillary haemangioma
D. Neuroblastoma. B. Cavernous haemangioma
109. A 45 years old female presents with uni- C. Lymphangioma
lateral mild axial proptosis. There is no D. Haemangiopericytoma.
redness or pain. Investigation of choice is: 115. A patient presents with unilateral propto-
A. T3 and T4 to rule out thyrotoxicosis sis which is compressible and increases
B. CT scan to rule out meningioma on bending forward. No thrill or bruit is
C. Doppler to rule out haemangioma present. MRI shows retrobulbar mass with
D. USG to rule out orbital pseudotumour. enhancement. The likely diagnosis is:
110. Infection from the dangerous area of the A. A V malformation
face spreads to the cavernous sinus via B. Orbital varix
which of the following veins? C. Orbital encephalocoele
A. Maxillary veins D. Neurofibromatosis.
B. Retromandibular veins 116. An 8-year-old boy presents with propto-
C. Superficial temporal vein sis in the left eye for 3 months. CT scan
D. Ophthalmic veins. reveals intraorbital extraconal mass lesion.
111. Paralysis of 3rd, 4th and 6th cranial nerves Biopsy shows embryonal rhabdomyosar-
with involvement of ophthalmic division of coma. Metastatic workup is normal. The
the 5th cranial nerve localizes the lesion to: standard line of management is:
A. Cavernous sinus A. Chemotherapy
B. Apex of the orbit B. Wide local excision
C. Brainstem C. Chemotherapy and radiotherapy
D. Base of the skull. D. Enucleation.
122 E. A. El Toukhy

117. In which side is the globe displaced in lac- C. Embryonal


rimal gland tumour? D. Mixed cellularity.
A. Superior 125. The least common variety of
B. Inferonasal Rhabdomyosarcoma is:
C. Inferotemporal A. Alveolar
D. Nasal. B. Pleomorphic
118. Most common type of optic nerve glioma is: C. Embryonal
A. Gemiocytic D. Mixed cellularity.
B. Fibrous 126. The variety of Rhabdomyosarcoma with
C. Protoplasmic the best prognosis is:
D. Pilocytic. A. Alveolar
119. Optic nerve glioma is associated with: B. Pleomorphic
A. Neurofibromatosis I C. Embryonal
B. Neurofibromatosis II D. Mixed cellularity.
C. Von Hippel Lindau disease 127. The variety of Rhabdomyosarcoma with
D. Sturge Weber syndrome. the worst prognosis is:
120. All of the following types of lymphoma A. Alveolar
may be seen in the orbit except: B. Pleomorphic
A. Non-Hodgkin’s lymphoma, mixed C. Embryonal
lymphocytic and histiocytic D. Mixed cellularity.
B. Non-Hodgkin’s lymphoma, poorly 128. Chocolate cyst is due to bleeding in a
differentiated A. Capillary hemangioma
C. Burkitt’s lymphoma B. Cavernous hemangioma
D. Hodgkin’s lymphoma. C. Lymphangioma
121. The most common benign orbital tumor in D. Orbital varix.
children is: 129. Exentration is indicated in all the follow-
A. Capillary Hemangioma ing conditions, except:
B. Dermoid cyst A. Eyelid sebaceous carcinoma with deep
C. Lymphangioma orbital invasion
D. Glioma. B. Primary orbital rhabdomyosarcoma
122. The most common primary malignant C. Extraocular extension of melanoma
orbital tumor in children is: without systemic metastasis
A. Leukemia D. Orbital phycomycosis.
B. Rhabdomyosarcoma 130. The following clinical features are charac-
C. Neuroblastoma teristic of adenoid cystic carcinoma of the
D. Lymphoma. lacrimal gland except:
123. The most common orbital malignant A. Occurrence in old age group
tumor in adults is: B. Rapid progression over weeks to months
A. Lymphoma C. Presence of bone erosion
B. Metastasis D. Hypesthesia in the temporal region.
C. Lacrimal gland tumors 131. All of the following clinical features are
D. Invasion from choroidal malignant characteristic of lymphoid tumors of the
melanoma. lacrimal gland, except:
124. The most common variety of A. Bony remodeling
Rhabdomyosarcoma is: B. Absence of globe ptosis
A. Alveolar C. More anterior location
B. Pleomorphic D. Soft to palpation.
8 The Orbit 123

132. The following disorders typically pro- B. Thrombocytopenia


duces downward and lateral displacement C. Leukocytosis
of the globe? D. Leukocytopenia.
A. Malignant mixed tumor of the lacrimal 139. A patient with suspected orbital cellulitis
gland develops loss of ocular motility with intact
B. Squamous cell carcinoma of the max- vision. The most likely diagnosis is
illary sinus A. Cavernous sinus thrombosis
C. Frontal sinus mucocele B. Orbital compartment syndrome
D. Optic nerve sheath meningioma. C. Meningitis secondary to orbital
133. All of the following disorders typically cellulitis
present with a rapid onset, except D. Orbital apex syndrome.
A. Ruptured dermoid cyst 140. Enophthalmos can occur in all of the fol-
B. Benign mixed tumor of the lacrimal lowing except
gland A. Silent sinus syndrome
C. Rhabdomyosarcoma B. Orbital varix
D. Orbital cellulitis. C. Medial wall orbital fracture
134. An orbital meningocele usually presents in D. Malar hypoplasia.
the: 141. Nevus flammeus occurs in:
A. Supraorbital notch A. Neurofibromatosis
B. Lateral canthus B. Treacher Collins’ syndrome
C. Lacrimal gland fossa C. Von Hippel’s disease
D. Medial canthus. D. Sturge-Weber syndrome.
135. A traumatic carotid cavernous fistula 142. Most periocular capillary hemangiomas
results in all of the following, except manifest by
A. Elevated orbital arterial pressure A. 4 to 8 weeks of age
B. Audible bruit B. 6 to 8 months of age
C. Tortous conjunctival vessels extending C. 12 to 18 months of age
to the limbus D. 3 to 4 years of age.
D. Extraocular muscle enlargement. 143. Most capillary hemangiomas reach their
136. The most common orbital or eyelid find- peak size at:
ing in the craniosynostosis syndromes is A. 2 to 3 months
A. Ankyloblepharon B. 9 to 12 months
B. Ptosis C. 18 to 24 months
C. Exotropia D. 3 to 5 years.
D. Hypertelorism and proptosis. 144. A characteristic feature distinguishing
137. Which of the following is true of breast between nevus flammeus and capillary
cancer metastatic to the orbit? hemangioma is
A. The majority of orbital metastases in A. Lesion color
women originate from breast cancer B. Presence or absence of blanching with
B. Patients will likely benefit from hor- pressure
mone therapy C. Area of skin affected
C. Bone metastasis is more common than D. Extent of skin thickening.
extraocular muscle involvement 145. Regarding orbital rhabdomyosarcoma,
D. Enophthalmos is more common than which is incorrect?
proptosis. A. Metastatic workup includes lumbar
138. Which of the following can be associated puncture and bone marrow biopsy
with multiple large capillary hemangiomas? B. Alveolar rhabdomyosarcoma, occurs
A. Thrombocytosis most frequently in the superior orbit
124 E. A. El Toukhy

C. Electron microscopic studies may be D. The lesion is lined by nonkeratiniz-


used in facilitating the diagnosis, as ing squamous epithelium with dermal
cross-striations are more apparent on appendages.
electron microscopy 151. A 45-year-old woman presents with an
D. The embryonal pattern is the most encapsulated well-circumscribed retrobul-
common pathologic variant, account- bar mass. All of the following are potential
ing for more than 80% of total cases. diagnoses, except
146. A patient with known neurofibromatosis A. Neurilemoma
presents with pulsating proptosis of long B. Hemangiopericytoma
duration. CT scan of the orbit will most C. Lymphangioma
likely reveal D. Fibrous histiocytoma.
A. Orbital neurofibroma 152. Which one of the following regarding der-
B. Abnormality of the sphenoid bone moid and epidermoid cysts is false?
C. Optic nerve glioma A. They share a common
D. Carotid-cavernous fistula. pathophysiology
147. A patient with diabetes mellitus B. The key distinguishing feature
­presenting with orbital cellulitis requires between the two is the nature of the
immediate: wall of the cystic cavity
A. Complete blood count C. In adults, nearly all of these lesions are
B. Oral temperature anterior to the orbital septum
C. Blood glucose measurement D. Superficial cysts present more often
D. Careful ear, nose, and throat evaluation. during childhood.
148. Compared with adults, all of the follow- 153. A patient presents with slowly progres-
ing findings are more common in pediat- sive proptosis of the right eye. Excisional
ric idiopathic orbital inflammation (orbital biopsy confirms large cavernous spaces
pseudotumor), except containing erythrocytes. All of the follow-
A. Uveitis ing may be seen in this condition, except
B. Optic disc edema A. Accelerated growth of the lesion in a
C. Unilateral presentation pregnant patient
D. Eosinophilia. B. Anterior displacement of the far point
149. All of the following disorders may be plane of the eye
associated with a clinical presentation C. More than one type of optic neuropathy
indistinguishable from typical inflamma- D. Radiodense phleboliths present on CT
tory orbital pseudotumor, except scan tmagmg.
A. Wegener’s granulomatosis 154. What is the expected reflectivity on
B. Churg-Strauss syndrome A-scan ultrasonography of the capsule of
C. Sarcoidosis cavernous hemangioma?
D. Polyarteritis nodosa. A. None
150. A 3-year-old boy presents with a painless B. Low
lesion of the lateral left upper eyelid and C. Medium
brow. The lesion is smooth and fixed to D. High.
the underlying bone. Which of the follow- 155. A 2-year-old child presents with bilateral
ing statements is true: proptosis. Which of the following is the
A. An eyelid crease incision usually does least likely diagnosis?
not provide sufficient exposure A. Metastatic neuroblastoma
B. The lesion is classified as a hamartoma B. Leukemia
C. The condition may occur along the C. Capillary hemangioma
superomedial orbital rim D. Orbital pseudotumor.
8 The Orbit 125

156. The most common sinus lesion that 162. The most common site of a primary tumor
invades the orbit is the metastatic to the orbit in men is
A. Osteoma A. Lung
B. Inverted papilloma B. Colon
C. Mucocele C. Prostate
D. Squamous cell carcinoma. D. Melanoma.
157. What is the treatment of choice for a 163. In a 10 years old patient with optic nerve
patient with an optic nerve sheath menin- glioma, which one of the following clini-
gioma confined to the orbit and with pro- cal features would be considered incon-
gressive visual loss? sistent with the diagnosis?
A. Observation A. Unilaterality
B. Surgical excision B. Insidious onset
C. Chemotherapy C. Afferent pupillary defect
D. Radiation therapy. D. Pain.
158. An 86-year-old patient presents with left 164. Which of the following answers would be
eye proptosis, diplopia, and a subconjunc- the best treatment option for a localized
tival salmon patch lesion. Which of the fol- orbital lymphoproliferative lesion?
lowing is the most appropriate next step? A. Radiation and systemic corticosteroids
A. Biopsy B. Radiation therapy alone
B. B-scan ultrasonography C. Surgical excision combined with
C. Corticosteroids chemotherapy
D. Orbital imaging. D. Surgical excision combined with
159. A 7-year-old boy presents with a 3-day radiation.
history of progressive proptosis, injection, 165. Hyperostotic lesions of the orbit can occur
and pain of the left eye. He is systemi- in all, except
cally well with normal temperature. White A. Metastatic prostate carcinoma
blood cell count is normal, and orbital CT B. Sphenoid wing meningioma
scanning reveals superonasal orbital infil- C. Fibrous dysplasia
tration with bony erosion. The most likely D. Metastatic melanoma.
diagnosis at this point is 166. A 11-year-old patient presents with acute,
A. Frontal sinus mucocele unilateral, left-sided periocular pain, prop-
B. Rhabdomyosarcoma tosis, and double vision. Which condition
C. Bacterial orbital cellulitis would not be included in the differential
D. Optic nerve glioma. diagnosis?
160. Which one of the following histiocytic A. Cavernous hemangioma
disorders is most likely to involve orbital B. Sinusitis with orbital abscess
bone? C. Traumatic retrobulbar hemorrhage
A. Sinus histiocytosis D. Orbital lymphangioma.
B. Hand-Schtiller-Christian disease 167. Twenty-four hours later (and without
C. Letterer-Siwe disease any treatment), the pain has resolved.
D. Eosinophilic granuloma. Periocular ecchymosis has developed,
161. Which of the following is one of the most and the double vision has stabilized. The
common mesenchymal tumor of the orbit? most likely diagnosis based on the clinical
A. Hemangiopericytoma history:
B. Fibrous histiocytoma A. Rhabdomyosarcoma
C. Osteogenic sarcoma B. Capillary hemangioma
D. Ossifying fibroma. C. Orbital abscess
D. Lymphangioma.
126 E. A. El Toukhy

168. If the patient was losing vision because of D. Benign mixed cell tumor of the lacri-
this process, you would consider: mal gland.
A. Open surgery to excise the lesion in its 173. The orbital ultrasound would show:
entirety A. Tissue of homogenous character
B. CT-directed drainage of the encysted B. High internal reflectivity
blood C. B scan identifying tumor in the ante-
C. Injection of sclerosing agents rior inferior orbit
D. Radiotherapy. D. Low amplitude internal echoes.
169. This disease process is an example of: 174. The natural history of such lesion is:
A. The most common cause of proptosis A. Slow growth over several years
in children B. Erosion of surrounding bony structure
B. The most common primary orbital C. Displacement of the globe downward
malignancy in children and medially
C. A tumor that may enlarge with upper D. Potential for malignant conversion of
respiratory infections the presently benign lesion.
D. An orbital vascular lesion that will invo- 175. Surgical removal of the lesion would best
lute after intralesional corticosteroids. be approached by:
170. In orbital infectious disease: A. A lateral orbitotomy with en bloc
A. The presence of a subperiosteal col- removal of the mass
lection of fluid is an indication for B. Incisional biopsy followed by radia-
surgery tion or chemotherapy
B. The onset of decreased vision and an C. An anterior approach through the infe-
afferent pupillary defect in the pres- rior fornix
ence of an orbital abscess is an indica- D. A medial orbitotomy with reflection of
tion for surgery the medial rectus muscle.
C. Proptosis and limitation of motility A 60-years-old woman presents with
differentiate an orbital abscess from painless swelling of the lacrimal gland
orbital cellulitis and anterior orbit for 2 months. There
D. The maxillary sinus is the most com- is no significant history.
mon sinus involved when orbital cellu- 176. What is the most likely diagnosis?
litis occurs as a result of sinusitis. A. Primary lacrimal gland lymphoma
171. A 6 years old patient has a 2-week history B. Pleomorphic adenoma
of rapidly progressing superonasal mass C. Adenoid cystic carcinoma
that does not affect vision. Examination D. Malignant pleomorphic adenoma.
shows proptosis pushing the eye down and 177. What is the most accurate description of
out. The best management includes all of the pathology specimen taken from this
the following except: lesion?
A. CT scan A. Spindle cells with both ductal epithe-
B. Anterior orbitotomy with biopsy lium and a mixed stromal pattern
C. MRI scan B. “Swiss cheese” pattern-hyperchromatic
D. Observation. small cells proliferating around nerves
172. A 48 years old female who is otherwise C. Ductal epithelium in a tubular forma-
healthy has a 2 years old slowly progress- tion with malignant degeneration
ing painless proptosis with normal vision. D. Mixture of both B and T cells, with
what is the most likely diagnosis? predominance of B cells.
A. Optic nerve glioma 178. The most common cause of bilateral
B. Cavernous hemangioma exophthalmos in adults is:
C. Metastatic breast cancer A. Cavernous hemangioma
8 The Orbit 127

B. Pseudotumor 185. A 61-year-old patient, with a chronic


C. Thyroid-related orbitopathy sinus problem presents with a 1-week his-
D. Metastatic disease. tory of redness and pain of the right eye.
179. What is the most common cause of unilat- His visual acuity is 20/20. His right upper
eral childhood exophthalmos? eyelid is swollen. The right conjunctiva
A. Capillary hemangioma is injected with dilated episcleral vessels
B. Thyroid-related orbitopathy inferiorly. The underlying sclera appears
C. Orbital hemorrhage inflamed. His ocular motility is limited,
D. Orbital cellulitis. and there is 2 mm of proptosis. A CT scan
180. Predisposing conditions for mucormycosis shows a diffuse infiltrate in the right infe-
include: rior orbit. There is also thickening of the
A. Diabetes left nasal mucosa. Which of the following
B. Renal disease tests would be most beneficial in diagnos-
C. Dehydration ing this patient’s condition?
D. All of the above. A. Serum c-ANCA (antineutrophil cyto-
181. Proper treatment of orbital mucormycosis plasmic antibodies)
includes all of the following except: B. Serum p-ANCA (antineutrophil cyto-
A. Stabilizing the underlying disease plasmic antibodies)
process C. Serum ESR (erythrocyte sedimenta-
B. Debridement of all devitalized tissue, tion rate) and C-reactive protein
including exenteration if necessary D. Conjunctival culture for bacterial and
C. Amphotericin B for 6 weeks viral pathogens.
D. Radiation to the orbit. 186. A 50-year-old patient presents with a
182. Indications for closure of carotid cavern- 4-day history of right eyelid swelling,
ous sinus fistulas does not include which conjunctival injection, and pain and dou-
of the following? ble vision. Visual acuity is normal. There
A. Ophthalmoplegia is normal pupillary reactions. There is
B. Severe headache a small abduction deficit 0D. The right
C. Persistent bruit upper eyelid is erythematous and swollen.
D. Progressive proptosis. The conjunctiva is injected and chemotic.
183. Which of the following sequelae of a Deeper episcleral vessels are injected. The
carotid cavernous sinus fistula is the most cornea is clear and the anterior chamber
common cause of visual disability? is quiet. The left eye is normal. Fundus
A. Strabismus examination is normal. There is 2 mm of
B. Proptosis with corneal exposure proptosis 0D. CT scan of the orbits shows
C. Spontaneous choroidal detachment diffuse soft tissue infiltrate involving the
D. Elevated intraocular pressure with pro- anterior portion of the right orbit. The
gressive optic nerve damage. nasal mucosa appears thickened on the left
184. A non-painful, well-circumscribed side. The patient has no fever, and labora-
homogenous spherical mass in the inferior tory studies are normal with no evidence
orbit that moderately enhances with con- of diabetes. After 7 days on oral corti-
trast in an otherwise healthy patient who costeroids, the patient shows moderate
complains of diplopia is best treated by: improvement. He has less pain, and red-
A. Chemotherapy ness and eyelid swelling have decreased.
B. Radiation therapy However, the proptosis and diplopia are
C. Surgical excision unchanged. Which of the following is the
D. Observation. most appropriate next step?
A. Start intravenous corticosteroids
128 E. A. El Toukhy

B. Continue the present treatment for 1 191. Computerized tomography has demon-
more week strated an orbital bone mass to have a
C. Remove tissue from the right orbit for “ground glass” appearance. What systemic
biopsy involvement should be ruled out?
D. Repeat the CT scan of the orbits. A. Generalized muscle weakness
187. A 2-year-old girl has left lower eyelid B. Visceral cancer
ecchymosis. There is 3 mm of proptosis C. Thyroid disease
of the left eye. Her medical history is sig- D. Endocrine abnormality.
nificant for treatment of some unknown 192. During routine examination of a patient’s
tumor. Which of the following childhood inferior cul-de-sac, a subconjunctival
tumors is the most likely diagnosis? lympho-proliferative lesion is observed.
A. Rhabdomyosarcoma The patient is unaware of this lesion and is
B. Retinoblastoma reportedly in good health. The results of the
C. Neuroblastoma remainder of the ocular examination are nor-
D. Leukemia. mal. A biopsy is done. What would the least
188. A 56-year-old man complains of an aching useful test performed on this biopsy be?
sensation around his left eye for 6 weeks. A. Permanent sections
The discomfort increases on upgaze. One B. Culture and sensitivity
week ago, he noted blurred vision in the C. Cell-surface markers
left eye and a low-grade fever. His visual D. Electron microscopy.
acuity is 20/20 OD and 20/40 OS. The 193. What is the best study to rule out organic
patient has 3 mm of proptosis in the left orbital foreign bodies?
eye and mild erythema and tenderness A. Magnetic resonance imaging
around the left eyelid. What is the most B. A dowsing rod
helpful diagnostic test for this patient? C. Plain films
A. CT scan of the orbits D. Computerized tomography.
B. Complete blood count 194. What is the study of choice for the evalua-
C. Thyroid function tests tion of fractures in acute orbital trauma?
D. Skull films. A. Orbital ultrasound
189. Which of the following orbital diseases is B. Computerized tomography
least likely to improve with corticosteroids? C. Magnetic resonance imaging
A. Orbital mucocele D. Nerve conduction.
B. Thyroid-related orbitopathy 195. The myositic form of idiopathic orbital
C. Orbital pseudotumor inflammation is associated with which of
D. Orbital lymphoma. the following conditions?
190. Which of the following is not a potential A. Efficacy of systemic steroid therapy
advantage of MRI over CT scanning? B. S-shaped deformity of the eyelid
A. MRI does not expose the patient to C. Fusiform enlargement of extraocular
radiation muscle involving the tendon
B. MRI is unaffected by motion artifact D. Nodular enlargement of the extraocu-
C. MRI can generate high quality axial, lar muscle belly.
coronal, and sagittal image without 196. What is a common sign of a malignant
repositioning the patient lymphoproliferative lesion?
D. MRI allows for better evaluation of A. Firm nodular anterior orbital mass
lesions that extend from the orbit to B. Painful proptosis
the cranium. C. Vision loss
D. Madarosis.
8 The Orbit 129

197. What is the optimal imaging technique for B. Attempted aspiration of the mass with
a posterior optic nerve glioma? empiric antibiotics if aspiration is
A. Magnetic resonance imaging unsuccessful
B. Plain X-ray films C. Discharge home on oral antibiotics
C. Computerized tomography D. Treat with intravenous antibiotics for
D. Orbital ultrasound. 10 days and reevaluate.
198. When diplopia develops in the setting of 203. What is the preferred management of
traumatic carotid cavernous fistula, what is hemangiopericytoma involving the orbit?
the most likely pathophysiology? A. Incisional biopsy followed by external
A. Compression of the fourth cranial radiation
nerve as it exits the brainstem B. Intralesional steroid injection
B. Compression of the superior rectus C. Observation
muscle within the muscle cone D. Complete local excision.
C. Damage to the third cranial nerve from 204. A 35-year-old woman has decreased vis-
elevated intracranial pressure ual acuity in the right eye over 3 years.
D. Compression of the sixth cranial nerve The visual acuity in the left eye is 20/20.
within the cavernous sinus. Examination of the right eye shows visual
199. What is the test of choice when consid- acuity of 20/70, a right afferent pupillary
ering treatment for a carotid cavernous defect, 3 mm axial proptosis and bilater-
fistula? ally normal optic discs. What is the most
A. Computed tomography likely diagnosis?
B. Magnetic resonance imaging A. Adenoid cystic carcinoma of lacrimal
C. Conventional angiography gland
D. Computed tomographic angiography. B. Orbital lymphoma
200. What would you expect to find on com- C. Optic nerve glioma
puterized axial tomography (CT) of a D. Optic nerve sheath meningioma.
dural sinus fistula? 205. What tissue provides such a bright signal
A. Extraocular muscle enlargement on a T1-weighted, magnetic resonance
B. Phleboliths image (MRI) that it can obscure important
C. Enlargement of the internal carotid structures?
artery A. Bone
D. Orbital expansion with Valsalva B. Fat
maneuver. C. Vitreous
201. For which orbital disease can increased D. Lens.
orbital fat volume be a primary radio- 206. What pathologic finding is found in idi-
graphic finding? opathic orbital inflammation?
A. Orbital myositis A. Monoclonal hypercellular lymphoid
B. Thyroid orbitopathy proliferation
C. Sarcoidosis B. Polyclonal hypercellular lymphoid
D. Wegner’s granulomatosis. proliferation
202. A 6-year-old presents with proptosis and C. Granulomatous cellular infiltrate
inferior-lateral displacement of the globe. D. Pleomorphic cellular infiltrate.
Imaging demonstrates clear sinuses and 207. In a young child with a subperiosteal
a large orbital mass. What step should be orbital abscess, in what location(s) would
considered next? medical therapy be preferred to surgical
A. Prompt biopsy with possible fro- drainage?
zen section diagnosis, bone marrow A. Lateral orbit
biopsy, and lumbar puncture B. Orbital apex
130 E. A. El Toukhy

C. Medial orbit D. Are often painful in men who have


D. Superior orbit. prostate cancer.
208. What is the prognosis of mucosal-associ- 213. Regarding pleomorphic adenoma of the
ated lymphoid tissue (MALT) mass in the lacrimal gland (benign mixed cell tumor),
orbit? which is incorrect:
A. Orbital enlargement, metastasis in half A. Occurs mainly in the palpebral lobe
of patients within 10 years B. Represents 50% of epithelial lacrimal
B. Usually progresses to large-cell gland tumors
lymphoma C. Most commonly occurs in the second
C. Rapid orbital enlargement, metastasis to fifth decades of life
in nearly all patients within 10 years D. May be cystic and contain calcification.
D. Benign ocular and systemic morbidity. 214. Regarding adenoid cystic carcinoma of the
209. A febrile 65-year-old diabetic has orbital lacrimal gland, which is incorrect:
cellulitis with severe edema, areas of gray A. Is rarely painful
skin discoloration and tissue necrosis. The B. Has a dismal prognosis, even with
sinuses are clear. What would be the pre- treatment
ferred treatment? C. May be seen at any age but is more
A. Amphotericin B common in the fourth decades of life
B. Surgical debridement, broad-spectrum D. Is the most common epithelial malig-
antibiotics, and probably ICU support nancy of the lacrimal gland.
C. System steroids 215. Regarding fibrous dysplasia of the orbit,
D. Hyperbaric oxygen. which is incorrect:
210. Regarding lymphangiomas, which is A. May cause vision loss from compres-
incorrect: sion of the optic nerve in the optic
A. Often appear at birth or during early canal
childhood B. Usually is poly-ostotic
B. May cause proptosis after they C. Replaces normal bone with immature
hemorrhage bone and osteoid
C. Are connected to the orbital lymphatic D. Usually is progressive until the second
system or third decade of life.
D. May enlarge with systemic viral illness. 216. Regarding rhabdomyosarcoma, which is
211. Regarding juvenile xanthogranuloma incorrect:
(JXG), which is incorrect: A. Requires immediate biopsy to confirm
A. Most commonly involves the skin of the diagnosis
the head and neck B. Is most commonly the embryonal form
B. May involve the iris, leading to spon- C. Involves a 5-year survival rate in 45%
taneous hyphema of affected individuals
C. Is a benign histiocytic proliferation D. Is the most common soft tissue pri-
D. If intraocular, usually involutes with mary malignant mesenchymal orbital
time. tumor in children.
212. Regarding metastatic tumors to the orbit, 217. Regarding plexiform neurofibroma, which
which is incorrect: is incorrect:
A. Develop before diagnosis of the pri- A. Is the most common benign peripheral
mary tumor in 30–60% of patients nerve tumor involving the eyelids and
B. Reach the orbit by hematogenous orbit
spread B. Is characteristic of neurofibromatosis
C. In children are usually carcinomas C. Is not invasive
D. Has a propensity for sensory nerves.
8 The Orbit 131

218. Regarding neuroblastoma that is meta- 224. Which of the following statements about
static to the orbit, which is incorrect: rhabdomyosarcoma is correct?
A. First appears as an orbital mass in 8% A. The most common extracranial solid
of cases childhood tumour
B. Is the second most common malignant B. Embryonal and alveolar subtypes have
orbital tumor of childhood distinct genetic alterations that may
C. Affects both orbits in 40% of children play in the pathogenesis of the tumors
D. Rarely advances to orbital bones. C. Orbital tumours are more likely to
219. Regarding malignant orbital lymphoma, have alveolar histologic subtype
which is incorrect: D. Orbital tumours commonly present
A. Usually contains proliferated B cells with ophthalmoplegia.
B. Is bilateral in 75% of cases 225. Which of the following statements about
C. Is associated with systemic lymphoma neurofibromatosis type 1 is correct?
in 40% of patients at the time of A. More than 50% of patients with NF1
diagnosis have learning difficulties
D. Is treatable and has an excellent visual B. Lab tests are useful in the diagnosis of
prognosis. NF1
220. Regarding orbital dermoid cysts, which is C. Lisch nodules are the most characteristic
incorrect: feature in children over six years of age
A. May be subtotally resected with good D. Choroidal hamartomas are well-
results defined, elevated lesions found in the
B. May lie deep in the orbit midperiphery of the retina.
C. Are lined with epithelium and filled 226. Which of the following signs would you
with keratinized material expect to see in a patient presenting with a
D. Represent 25% of all orbital and lid suspected direct carotico-cavernous fistula
masses. (CCF) after an injury but not in a sponta-
221. Diagnostic criteria for IgG4 related orbit- neous indirect CCF?
opathy includes all except: A. Acute painful proptosis
A. Characteristic swelling in the orbit B. Cranial bruit
B. Elevated serum IgG4 C. Dilated episcleral vessels
C. Histopathological evidence of lym- D. Raised intraocular pressure (lOP).
phocytic proliferation 227. A 4-year-old child presents as an emer-
D. Excellent response to steroid gency with a 2-day history of unilateral
treatment. periocular swelling, redness, and prop-
222. The biologic response modifier, rituximab, tosis. Which of the following is NOT an
binds to which of the following targets? essential emergency investigation?
A. CDlla A. Full blood count
B. CD20 B. Temperature
C. CD25 C. Plain film X-ray face
D. TNF-a. D. Weight.
223. Langerhans cell histiocytosis include all 228. During a surgical decompression for acute
except: compressive optic neuropathy which of the
A. Eosinophilic granuloma paranasal sinuses will NOT be entered?
B. Kassabach–Merritt syndrome A. Ethmoid
C. Hand-Schuller–Christian disease B. Frontal
D. Letterer–Siwe disease. C. Maxillary
D. Sphenoid.
132 E. A. El Toukhy

229. A 40-year-old female presents with a B. Dacryoadenitis


3-month history of painful swelling in C. Pleomorphic adenoma
the superotemporal quadrant of the left D. Sarcoidosis.
orbit. She undergoes an incision biopsy 230. In a patient with an isolated lacrimal gland
which demonstrates glandular tubules mass, what test would be the most sensi-
with lumina, excess basement membrane tive to differentiate between lymphoma
and mucin, and islands of anaplastic and orbital inflammatory disease (OlD)?
cells without squamous differentiation. A. Contrast enhanced MRI
Which of the following is the most likely B. Orbital biopsy
diagnosis? C. Serum LDH
A. Adenoid cystic carcinoma D. Steroid response.

231. The above histopathological figures B. Embryonal


points to the diagnosis of which type of C. Mixed
rhabdomyosarcoma: D. Pleomorphic.
A. Alveolar
8 The Orbit 133

Answer for the Orbit 30 C 60 C 90 C 120 D


121 B 151 C 181 D 211 D
122 B 152 C 182 C 212 C
1 B 31 B 61 C 91 C
123 A 153 B 183 A 213 A
2 C 32 A 62 D 92 B
124 C 154 D 184 C 214 A
3 B 33 B 63 B 93 C
125 B 155 C 185 A 215 B
4 A 34 B 64 B 94 A
126 B 156 C 186 C 216 C
5 B 35 B 65 B 95 C
127 A 157 D 187 C 217 C
6 B 36 D 66 C 96 B
128 C 158 D 188 A 218 D
7 D 37 B 67 A 97 A
129 B 159 B 189 A 219 B
8 B 38 C 68 D 98 B
130 A 160 D 190 B 220 A
9 C 39 A 69 D 99 A
131 A 161 B 191 D 221 D
10 D 40 C 70 D 100 C
132 C 162 A 192 B 222 B
11 C 41 D 71 A 101 C
133 B 163 D 193 A 223 B
12 C 42 A 72 B 102 B
134 D 164 B 194 B 224 B
13 C 43 C 73 B 103 A
135 A 165 D 195 D 225 A
14 D 44 D 74 C 104 A
136 D 166 A 196 A 226 B
15 B 45 A 75 D 105 C
137 A 167 D 197 A 227 C
16 B 46 A 76 A 106 A
138 B 168 B 198 D 228 B
17 B 47 D 77 C 107 A
139 A 169 C 199 C 229 A
18 B 48 B 78 D 108 D
140 D 170 B 200 A 230 B
19 C 49 D 79 C 109 A
141 D 171 D 201 B 231 B
20 C 50 B 80 B 110 D
142 A 172 B 202 A
21 D 51 A 81 B 111 A
143 B 173 B 203 D
22 C 52 C 82 C 112 A
144 B 174 A 204 D
23 C 53 C 83 D 113 A
145 B 175 A 205 B
24 B 54 D 84 B 114 B
146 B 176 A 206 D
25 D 55 A 85 C 115 B
147 D 177 D 207 C
26 C 56 D 86 D 116 C
148 C 178 C 208 A
27 B 57 D 87 B 117 B
149 C 179 D 209 B
28 A 58 B 88 C 118 D
150 C 180 D 210 C
29 A 59 D 89 B 119 A
Orbital Implants
and Prosthesis 9
Essam A. El Toukhy

Eye Removal Surgery (enucleation or evis- of the globe with or without keratectomy. It
ceration) is often considered as a lost battle in involves minimal disruption of the orbital con-
ophthalmology as there is no hope for restor- tents with the best cosmetic result over enuclea-
ing vision. The technique of eye removal is tion. It is contraindicated in a patient who has
constantly evolving. Sufferers bears the stigma a history of intraocular tumor. In a blind eye, a
of disfigurement and some gets disturbed psy- B scan ultrasound must be done to rule out an
chologically. One simple solution in most of occult tumor. Expansion sclerotomies have
these cases is to fit a custom-designed ocu- improved the outcomes in eviscervation ­surgical
lar prosthesis (or an artificial eye) that looks technique. The procedure takes less time than
like a natural eye and can even move (to varied enucleation surgery and can be done under
extent in different conditions). The psychologi- general or monitored assisted anesthesia with
cal aspects of loss of an eye should always be minimal complications.
addressed by the oculoplastic surgeon and the Evisceration from a practical point of view
ocularist and patients may even need the help of makes sense. Easy to perform, minimal compli-
a clinical psychologist particularly in children cations and provides good motility. It is a sur-
and young adults. Organizing meetings with the gical procedure in which the entire contents of
ocularist prior to eye removal surgery can be the globe are removed through a corneal, limbal
helpful in this regard. or scleral incision. The extraocular muscles are
The ideal socket is a centrally placed well not detached from the sclera, and the optic nerve
covered implant of adequate size fabricated and its surrounding meninges are left undis-
from an inert material. It should have deep turbed. Although the cornea was traditionally
unobstructed fornices with an inferior lid and always removed, most surgeons now preserve
fornix that can adequately support the prosthetic it. In the event that expansions cannot house
eye. The superior eyelid should be symmetrical an adequate implant, the posterior sclera can
with the normal eyelid, and finally prosthetic be totally transected and an alloplastic sphere
movement should approach the normal side. placed in the intraconal space.
Evisceration is a surgical procedure involving General anesthesia is not usually needed,
the surgical removal of the intraocular contents monitored attended local anesthesia is very
effective. A retrobulbar block and frontal block
anesthesia must be used for intraoperative and
E. A. El Toukhy (*) postoperative pain management due to postop-
Oculoplasty Service, Cairo University, Cairo, Egypt erative discomfort and swelling.
e-mail: eeltoukhy@yahoo.com

© The Author(s), under exclusive license to Springer Nature Switzerland AG 2021 135
E. A. El Toukhy (ed.), Oculoplasty for Ophthalmologists, https://doi.org/10.1007/978-3-030-68469-3_9
136 E. A. El Toukhy

Surgeons performing enucleation must take preoperative A-scan of the fellow eye. The use
into account movement of the prosthesis after of A-scan ultrasonography of the fellow healthy
surgery and the potential postoperative compli- eye to provide a tool for correct orbital implant
cations such as implant extrusion or socket con- size to replace 80% of the volume removed at
traction. The optimal size of the chosen sphere enucleation. This method allows a gap in the
should be such that when it is placed within the anterior socket for an ocular prosthetic volume
orbit, the muscles can be tied over the implant of 2 mL when the orbital implant is placed deep
without any tension. in the intraconal space. The algorithm divides
Enucleation or evisceration can be performed the preoperative A-scan values into hyper-
without placement of an orbital implant, but this opes and emmetropes/myopes for final orbital
will result in suboptimal cosmetic outcome. An implant size calculations. The algorithm can
orbital implant replaces the lost volume in evis- be used to preoperatively calculate the proper
cerated or enucleated globe, impart motility to orbital implant size for both adults and chil-
the prosthesis, supports surrounding structures dren undergoing enucleation or evisceration
and thus maintain cosmetic symmetry with the procedures.
fellow eye. A major focus of research and development
A large number of implants are available in the last couple of decades is the newer materi-
today. An ideal implant is the one which fulfills als for orbital implant and improvement in pros-
the following criteria: thesis motility. Porous materials are currently
preferred primarily because of vascularization
• Integration with orbital tissues and integration that occur. These implants are
• Biocompatible: It should not cause any aller- less likely to migrate than silicone or PMMA
gic or inflammatory reaction or rejection implants and are associated with better prosthe-
• Non-biodegradable sis motility especially when coupled with a peg.
• Free from complications like infection, extru- However, hydroxyapatite and porous polyeth-
sion and migration ylene are significantly more expensive and are
• Adequate volume replacement associated with higher rates of exposure than
• Adequate support for prosthesis traditional non integrated implants. Wrapping or
• Allow maximum motility with prosthesis “capping” these implants appears to reduce the
• Stimulate orbital growth exposure rate to acceptable levels. Implant size
• Readily available, inexpensive and easy to is crucial and should be customized. Implant
use/implant. motility is primarily determined by the attach-
ment of extraocular muscles to the implant.
It is crucial to place an optimum size implant. Placement of wrapped silicone or PMMA
Smaller implants tend to migrate and does not implant with extraocular muscle attachment
solve the purpose of adequate volume replace- provides excellent results in patients who do
ment. Larger implant interferes with the aesthet- not wish to consider a motility peg placement.
ics and tension on the conjunctival wound that Porous implants should be used in patients who
could result in wound gap and implant extru- are keen on further enhanced motility.
sion. Ideally, 65–70% of the volume should be Hydroxyapatite-First introduced by Perry in
replaced by implant and remaining 30–35% with 1985, the implant material is made of a com-
the prosthesis. The recent introduction of non- plex calcium phosphate salt normally found in
spherical (conical or egg-shaped) implants has human mineralized bone and derived from living
made it possible to increase the volume of the corals found deep in the oceans. It is biocom-
implant without the need to increase its anterior patible, non-biodegradable, non-toxic and non-
curvature. allergenic. The porous matrix is infiltrated by
An algorithm was developed to assess the the orbital fibrovascular tissue. Vascularisation
optimal orbital implant size when performing a of the implant can be assessed radiographically
9 Orbital Implants and Prosthesis 137

with contrast enhanced Magnetic Resonance possibility of extrusion of alloplastic implant.


Imaging (MRI) with surface coil. Implants However, fat atrophy can cause late volume loss
with grade 3 or 4 vascularisation (equal to or in adults implanted with dermis fat graft.
greater than the orbital rim) are considered as Finally a custom–made prosthesis is fabri-
adequately vascularized. The assessment of vas- cated. Most modern ocular protheses are made
cularization is essential before drilling a hole for using polymethyl methacrylate (PMMA).
pegging to identify the central avascular zone.
Implants with more than 75% vascularization Orbital Implants and Prosthesis
also tend to bleed more during drilling. The
1. Indications for enucleation, All are correct
extent of vascularization can be influenced by
except:
the pore structure and orientation of the pores.
A. Intraocular malignancy
Poor vascularization can lead to implant extru-
B. Painful blind eye
sion. The rough surface of the porous implants
C. Severely traumatized eye
can cause erosion of the conjunctiva and Tenon’s
D. Endophthalmitis.
and ultimately cause exposure of the implant.
2. The advantage of evisceration, one is false:
This can be prevented by wrapping the implant
A. Less disruption of orbital anatomy
with appropriate materials.
B. Less motility of prosthesis
Porous polyethylene implants are normally
C. Better treatment of endophthalmitis
well tolerated in the orbital soft tissue. They are
D. Technically is a simple procedure.
easier to implant because they have a smoother
3. Causes of contracted socket, one is false:
surface than HA implants, and potentially cause
A. Radiation treatment
less irritation of the overlying conjunctiva fol-
B. Extrusion of an enucleated implant
lowing placement. These implants have a high
C. Severe initial injury
tensile strength, yet they are malleable. This
D. Removal of the conformer for short
allows easy sculpting of the anterior surface of
period.
the implant. They may be used with or without a
4. The primary advantage of nonporous com-
wrapping material.
pared to porous orbital implant is:
Exposure of a porous implant necessitates
A. Better orbital volume replacement
immediate correction. Pegging was used in a
B. Lower exposure rates
trial to increase motility, however most ocu-
C. Increased implant stability
loplastic surgeons no longer support implant
D. Better prosthesis motility.
pegging.
5. Enucleation in childhood, one is false:
The choice of the implant, the implant size,
A. Associated with under development of
its placement, gentle tissue handling, attention to
the bony orbit
creation of fornices with adequate conjunctival
B. Autogenous dermis fat graft have been
lining, proper upper and lower eyelid position-
shown to grow along with expanding
ing and correction of any laxity are all important
orbit
parts of the surgical planning and execution.
C. Using implants that exert maximal ten-
Though dermis fat graft is not considered
sion on tissues allow the orbital bone to
often as a conventional orbital implant, it is an
grow
excellent material for implantation of pedi-
D. Loss of volume of the orbit occurs when
atric anophthalmic sockets and fulfills most
dermis fat graft is used as an implant in
of the criteria of an ideal implant. Dermis fat
adults.
graft is preferred in children with anophthalmic
6. True statement regarding enucleation is:
socket with both volume loss and conjunctival
A. Indicated for primary intraocular tumor
shortening (surface loss). It is also preferred in
B. Cannot be performed under local
reconstruction of irradiated sockets where the
anesthesia
vascularity is compromised and there is a higher
138 E. A. El Toukhy

C. Scleral shell trail can be an alternative 11. Characters of orbital implants, one is false:
to enculeation for non painful disfigured A. Inert spherical implants usually provide
eyes comfort
D. Early enucleation is indicated for all B. Hydroxyapatite and porous implants
severely traumatized eyes. allow for drilling and placement of a
7. The following is a complication of an ano- peg
phthalmic socket: C. Pegging is usually carried out one
A. Excessive fibrosis leading to increase month after the enucleation
orbital volume D. Pegged porous implants have the higher
B. Deep superior sulcus rates of postoperative complications.
C. Eyelids fusion 12. Enucleation is indicated in, one is false:
D. Excessive vascularization. A. Retinoblastoma
8. When an enucleation is performed in a B. Panophthalmitis
child: C. Ciliary staphyloma
A. The implant should not be placed until D. Penetrating injury of the eye with no
the child is 7 years old hope of vision.
B. A dermis–fat graft should be avoided 13. Regarding exenteration, one is true:
because it does not grow with the orbit A. Considered in management of huge
C. The optic nerve should be cut flush at rhabdomyosarcoma
the posterior sclera when retinoblas- B. Total exentration is the removal of all
toma is present intraocular tissue with or without the
D. An adult size implant should be placed skin of eyelids
as soon as possible to promote orbital C. Fixating of ossoointegrated implant is
growth. achieved with screw
9. The least likely indication for exenteration D. The exenteration prosthesis usually
is: blink and move.
A. Intraocular malignant melanoma that 14. Causes of contracted socket, one is false:
extended outside the globe with evi- A. Chemotherapy treatment
dence of distant metastasis B. Extrusion of an enucleated implant
B. Management of epithelial tumors of the C. Poor surgical technique
lacrimal gland D. Multiple socket operations.
C. Destructive tumors extending from the 15. In cases of implant extrusion following enu-
sinus to the orbit cleation or evisceration, which is false;
D. Primary orbital malignancy that do A. Early extrusion is associated with poor
not respond to non surgical treatment wound closure
modalities. B. Early extrusion is associated with
10. Regarding exenteration, one is false: implant that is too small
A. Considered in management of recurrent C. Late extrusion is associated with tumor
rhabdomyosarcoma non responding to recurrences
radio and chemotherapy D. Late extrusion is associated with con-
B. Total exenteration is the removal of all junctival cyst.
intraorbital soft tissues with or without 16. Evisceration is contraindicated in:
the skin of the eye lids A. Endophthalmitis
C. Fixation of osseo-integrated implant is B. Atrophia bulbi
achieved with glue C. Massive vitreous seedling retinal tumors
D. The exenteration prosthesis usually do D. Absolute glaucoma.
not blink or move.
9 Orbital Implants and Prosthesis 139

17. Blind painful left eye has secondarily devel- C. Bacterial conjunctivitis
oped perforation of the cornea. The surgical D. Eyelid malposition.
procedure of choice is, 22. What is an advantage of evisceration over
A. Evisceratiom with Keratectomy enucleation?
B. Evisceration without keratectomy A. Lower risk of sympathetic ophthalmia
C. Enucleation B. Better treatment of endophthalmitis
D. Exentration. C. No need for placement of orbital implant
18. The most common postoperative complica- D. Better histopathologic examination of
tion of enucleation is intraocular contents.
A. Socket contracture 23. Which of the following is not a reported
B. Enophthalmos complication of hydroxyapatite implants?
C. Superior sulcus deformity A. HIV transmission from the donor sclera
D. Extrusion of implant. B. Exposure of the hydroxyapatite surface
19. Regarding Evisceration for blind painful C. Chronic bacterial infection in the
eyes, which is incorrect: implant
A. The procedure is appropriate even in the D. Migration and extrusion of the implant.
setting of endophthalmitis 24. A patient has an NLP and painful eye
B. There is a low risk of developing granu- shortly after treatment for chronic endoph-
lomatous inflammation in the other eye thalmitis. When evaluating the surgical
C. Posterior incisions in the sclera allow options, which technique is preferable?
for placing a larger orbital implant A. Enucleation
D. The cornea may be retained provided B. Evisceration
the epithelium is removed. C. Subtotal exenteration
20. A patient undergoes an uncomplicated D. Total exenteration.
enucleation for a blind, painful eye. A 25. Regarding hydroxyapatite orbital implant
hydroxyapatite implant is placed in the after enucleation, which is incorrect:
socket. Six weeks after surgery, the patient’s A. Is usually wrapped in donor sclera
examination shows a well-healed socket B. Receives the four rectus muscles
with a deep superior and inferior fornix. C. Requires a peg to produce maximal
Movement of the orbital implant is excel- movement of the implant
lent. However, the patient is disappointed D. May be rejected by the body’s immune
that the prosthesis does not move well and system.
asks if any improvements are possible. You 26. Regarding evisceration, which is incorrect:
discuss the option of pegging the implant, A. Is contraindicated in cases of suspected
which can be coupled to the prosthesis and intraocular malignancy
improve prosthesis movement. What is the B. Always requires corneal removal
most appropriate next step? C. Does not obviate the risk of sympathetic
A. Schedule the patient for the next avail- ophthalmia
able surgical date D. Is contraindicated if precise histo-
B. Order MRI with contrast of the orbit pathologic examination of the globe is
C. Order a bone scan of the orbit needed.
D. Schedule the patient for a return visit in 27. Complications of exenteration include all
3 months. except:
21. Which of the following would not cause A. Severe blood loss
discharge in patients with an anophthalmic B. “Phantom limb” pain from the cut optic
socket and ocular prosthesis? nerve
A. An old prosthesis C. Skin graft infection
B. Dry socket D. Chronic sino-orbital fistulas.
140 E. A. El Toukhy

28. Regarding pegging of an integrated implant,


all are true except:
A. Generally undergoes drilling 6 to 12
months after placement
B. Improve prosthesis movement
C. Markedly increase the incidence of
complications
D. Is the standard of care now.

30. The above technique is used to allow:


A. Suturing of the extra ocular muscles
B. Fibrovascular ingrowth into a porous
implant
C. Insertion of a larger implant
D. Drainage of any postoperative
hemorrhage.
29. Regarding closure of evisceration, which is
false:
A. Must be done in multiple layers
B. A running locking suture is used for the
conjunctiva
C. Sutures should be removed after 1 week
D. Must eliminate potential space between
the layers.
9 Orbital Implants and Prosthesis 141

31. The above technique is: 33. Regarding orbital implant size, which is
A. Primarily used in children false:
B. Provides both volume and surface A. Should provide at least 70% of the
C. Allow for bony socket expansion removed volume
D. Has a higher rate of infection. B. Is essentially determined preoperatively
C. Is essentially determined intraoperatively
D. Is smaller if wrapping is used.
34. Regarding orbital implant size, which is
false:
A. May require examining the other eye
B. Depend on the surgical procedure used
C. Takes the thickness of the prosthesis
into consideration
D. Is larger if the eye was staphylomatous.

32. The above technique:


A. The extraocular muscles can be sutured
to it
B. Grows with the growth of the child
C. Has a higher exposure rate
D. Can be used when vascularity is
compromised.

35. Implantation of the above implant design


results in all except:
A. Less risk of exposure
B. More contact with orbital fat
C. Easier surgical technique
D. Insertion of a larger implant volume.
142 E. A. El Toukhy

36. The above patient had a history of retino- 40. Which of the following is recommended for
blastoma treated by exenteration and radio- 6-monthly follow-up of patients who have
therapy, his best option now is: had enucleation for ocular melanoma?
A. Insertion of a non integrated implant A. CT abdomen
B. Insertion of an integrated implant B. PET CT
C. Use of a dermis fat graft C. Serological liver function tests
D. Use of a pedicled local flap. D. Ultrasound of the liver.
37. During evisceration, removal of the uvea is
made easier by: Answers of Orbital Implants and Prosthesis
A. Keratectomy
B. Disinsertion of the scleral spur 1 D 16 C 31 D
C. Expansion sclerotomies 2 B 17 A 32 C
D. Bipolar cautery.
3 D 18 C 33 C
38. During evisceration, the risk of sympathetic
4 B 19 D 34 D
ophthalmia is further reduced by the use of:
A. 70% alcohol 5 C 20 C 35 C
B. Hydrogen peroxide 6 C 21 B 36 D
C. Betadine 7 B 22 B 37 B
D. Iodoform gauze. 8 D 23 A 38 B
39. The ideal size of the ocular prosthesis is: 9 A 24 B 39 B
A. 2 mL 10 C 25 D 40 D
B. 2.5 mL
11 C 26 B
C. 3 mL
12 B 27 B
D. 3.5 mL.
13 C 28 D
14 A 29 C
15 B 30 C
Oculoplasty Interactions
with Other Specialities 10
Essam A. El Toukhy

The interaction between oculoplasty and other any preexisting conditions that may lead to cer-
ophthalmic subspecialities is more than with tain operative challenges or possible postopera-
any other ophthalmic subspeciality. With pedi- tive complications. Abnormalities in lid margin,
atric ophthalmology; The development of the palpebral fissure, blinking pattern or lid inflam-
lids plays a crucial role for the normal function mations are such examples. Tear film evaluation
of the eyes as well as the impact of the cosmetic both quantitatively and qualitatively is essen-
appearance on the functional and psychologi- tial. Intraoperative challenges related to ocular
cal welfare of the child. The whole spectrum of adnexa should be anticipated, prevented and/
lid anomalies is an essential part of oculoplasty. or properly managed. Postoperative changes in
Ptosis, lid colobomas and epiblepharon are just corneal sensation and their effect on tear pro-
examples. Also, The position of the upper and duction, corneal healing and patient subjective
lower lids is usually changed after any surgery symptoms of dry eye and finally on his quality
on the extraocular muscles most likely due to of vision and satisfaction should be properly
the close embryological, anatomical and inner- understood and managed. Failure to identify
vational relations between the lid muscles, and manage these challenges before, during and
extraocular muscles and orbital connective tis- after refractive surgery may lead to serious com-
sues. This directly reflects on the management plications and can affect the final visual outcome
of patients requiring strabismus surgery and lid and patient satisfaction.
surgery. Ptosis, whether primary or residual after lid
Corneal Refractive Surgery is intimately surgery, has direct impact on patient refraction
related to Oculoplasty as the cornea is an inte- by pressing on the upper cornea leading to cor-
gral part of the ocular surface and is affected neal astigmatism and abnormal topography. If it
by the anatomical and functional status of the is severe enough to cover the pupil since birth,
ocular adnexa and tear production and func- it may lead to amblyopia that cannot be cor-
tion. Preoperative evaluation of patients seek- rected by refractive surgery. Ptosis or tight lids,
ing refractive correction can guide the surgeon if missed or left untreated, may also affect the
to the proper timing of surgery, select the best Lasik flap postoperatively leading to flap wrin-
technique for each patient and allow him to treat kles or striae. Lagophthalmos can follow facial
palsy or be a sequel of lid surgery like ptosis,
entropion or lid tumors. It may result in unsta-
E. A. El Toukhy (*) ble tear film, exposure keratitis, dry eye or even
Oculoplasty Service, Cairo University, Cairo, Egypt corneal opacity. Lid margin abnormalities like
e-mail: eeltoukhy@yahoo.com

© The Author(s), under exclusive license to Springer Nature Switzerland AG 2021 143
E. A. El Toukhy (ed.), Oculoplasty for Ophthalmologists, https://doi.org/10.1007/978-3-030-68469-3_10
144 E. A. El Toukhy

rubbing lashes, entropion or ectropion should be proper diagnosis and management of dry eye
checked as they may lead to intraoperative dif- patients or those at a higher risk of developing it
ficulties of proper corneal exposure or pressing and treating them before they undergo refractive
against the LASIK flap distorting it. surgery.
Lid margin inflammation like blepharitis and In patients with persistent lid problems like
Meibomian gland dysfunction (MGD) should lagophthalmos, infrequent blinking or lid mar-
not be missed during preoperative examina- gin abnormalities, it is better to avoid Lasik
tion. They can affect the accuracy and reliabil- and shift to either surface ablation, refractive
ity of preoperative investigations like corneal lenticule extraction (SMILE) or to intra-ocular
­topography and tomography leading to wrong surgery like phakic IOLs. Those cases can also
decision making. They may also lead to serious benefit from intra-operative punctal occlusion by
intraoperative challenges by pouring meibomian temporary punctal plugs to keep normal tears.
secretions on the corneal surface or under the Glaucoma, as a disease entity, can affect the
lasik flap. These secretions may interfere with cosmetic aspect of the eye in multiple ways. The
femtolaser pathway and gas bubble formation, disease in itself can be associated with angry
block excimer laser ablation or become trapped red looking eyes, due to either high eye pressure
under a Lasik flap or in a SMILE pocket. In or as a side effect of glaucoma medications. In
rare cases, they may also lead to diffuse lamel- advanced cases the disease itself or as a result
lar keratitis (DLK) or interface debris. All types of complicated surgery can result in a shrunken
of blepharitis can also be a source of postopera- phthisic eye, or an enlarged staphylomatous one.
tive inflammation like diffuse lamellar keratitis Effects of preservatives have been indicated
(DLK). as a causative factor of ocular surface disease
Patients with floppy eyelids and sleep apnea (OSD) associated with ophthalmic antiglauco-
are also more liable to eye rubbing with its serious matous agent administration. More than 60%
effect on Lasik flap leading to flap wrinkles, flap of patients with glaucoma have signs and symp-
striae or even flap displacement. Both infrequent toms of OSD.
blinking and excessive blinking can affect the Conjunctival allergy, conjunctival hyperemia,
ocular surface health, tear stability and tear clear- corneal epithelial disorders, and blepharitis are
ance. Not only can this exaggerate symptoms of common adverse reactions associated with most
dry eye but can also affect postoperative healing anti glaucoma eyedrops. With prostaglandin
and flap adherence, especially in surface ablation analogs, patients may also have eyelash bris-
procedures. Blinking abnormalities are very com- tling/lengthening, vellus hair, eyelid pigmenta-
mon in patients who have been contact lens wear- tion, iris pigmentation, and deepening of the
ers for a long time with subsequent diminished upper eyelid sulcus (DUES).
corneal sensation and lack of the stimulus to blink. Ocular adverse reactions associated with car-
Above all, the most important preopera- bonic anhydrase inhibitors include conjunctival
tive examination is for ocular surface health, allergy, conjunctival hyperemia, corneal epithe-
tear volume, tear stability and tear clearance as lial disorders, blepharitis, Stevens–Johnson syn-
indicators of dry eye disease (DED). Tear film drome, and toxic epidermal necrosis. With Rho
abnormalities whether epiphora or unstable tear khinase inhibitors, The most frequent adverse
film seriously affect preoperative investigations events were ocular: conjunctival hyperemia,
including placido-based corneal topography, conjunctival hemorrhage, and cornea verticillata.
optical scheimflug tomography or all types of Finally, glaucoma surgeries can result in pto-
wavefront aberrometry measuring systems. This sis or upper eyelid retraction due to mechanical
can lead to a false diagnosis of irregular corneal or myogenic mechanisms.
surface like keratoconus or can mask an existing A variety of conditions may present with symp-
abnormality. It is now advised to have a dry eye toms and signs that overlap between the subspe-
clinic in each refractive surgery center to ensure cialties of oculoplastics and neuro-ophthalmology.
10 Oculoplasty Interactions with Other Specialities 145

Neuro-ophthalmic disorders affecting lid and A variety of diseases are unique in their abil-
ocular muscles as Myasthenia gravis or CPEO ity to involve both the sinonasal (SN) cavities
is a classic example of the interaction between and the orbits. It is more common for SN pathol-
oculoplasty and neuro ophthalmology. These ogy to affect the orbit than the reverse, and
disorders present commonly with ptosis and primary sinus pathology may initially present
are usually first seen by the oculoplastic sur- with predominantly orbital, rather than sinus,
geon. The diagnosis of such conditions is based symptomatology.
on the clinical presentations, serological and Generally, there is more than one ocular
pharmacological tests and electrophysiologi- symptom found in each patient with sinonasal
cal assessments Proper diagnosis and manage- disease extending to the orbit. Proptosis is the
ment requires co-management between both commonest occurring in about 60% of cases. The
specialities. direction of proptosis is an important clue of the
Giant cell arteritis (GCA), also known as location of the involved sinus. Frontal sinus prop-
temporal arteritis, is one of the most important tosis occurs inferiorly and is accompanied by
emergencies in ophthalmology because of its swelling of the brow area. Direct lateral proptosis
irreversible and devastating effect on vision in occurs in ethmoid sinus disease. With maxillary
approximately half of patients. Temporal artery sinus pathology, the proptosis is upwards.
biopsy performed by an oculoplastic surgeon is Other less common symptoms include oph-
the gold-standard for diagnosis and should be thalmoplegia and visual loss. A decrease in vis-
done in every patient where clinical suspicion is ual acuity indicates of optic nerve involvement.
high; regardless of the results of any other test. The underlying pathophysiology may be caused
Horner syndrome (HS), an oculo-sympathetic by direct compression of the nerve fibers, non-
palsy which includes the triad of eyelid pto- perfusion of its blood vessels or inflammation/
sis, ipsilateral miosis and facial anhidrosis, is infection in proximity to the nerve.
another example where the oculoplastic surgeon Ophthalmoplegia can be caused by a
is involved in the diagnosis, localization and sur- mechanical restriction on extraocular muscles or
gical management. nerves paresis. Force duction test can distinguish
For idiopathic intracranial hypertension between both. Positive test denotes mechanical
(IIH), Treatment and management requires restrictions. Abnormal ocular motility can cause
multi-specialty team work. Optic nerve sheath diplopia both at the primary gaze position and
fenestration (ONSF) surgery is usually per- the position of the extremes gaze.
formed by the oculoplastic surgeon on request Disease entities affecting the sino-orbital
by the neuro ophthalmologist. region may arise primarily in the SN cavities,
Carotid cavernous sinus fistula (CCF), par- the orbits, or the surrounding bones; or they
ticularly the spontaneous type, should be con- may result from secondary involvement by sys-
sidered in the differential diagnosis of Graves’ temic disorders. Generally, sino-orbital patholo-
ophthalmopathy, orbital cellulitis and idiopathic gies can be classified broadly into four groups:
intra-orbital inflammation (1) Infectious and inflammatory conditions;
The orbit is closely related anatomically to bacterial sinusitis and orbital cellulitis, fungal
the paranasal sinuses. It is related superiorly infections, mucoceles and the silent sinus syn-
to the frontal sinus, medially to the ethmoid drome (2) Granulomatous disease; GPA, sar-
sinuses, inferiorly to the maxillary sinus and coidosis, and Rhinoscleroma (3) Fibro-osseous
posteromedially to the anterolateral wall of the lesions; osteomas and fibrous dysplasia and (4)
sphenoid sinus. Owing to this close anatomic Neoplasms: particularly malignant sinus tumors.
proximity, both can share same diseases, and/ Maxillofacial lesions involving the perior-
or extension from one of them to the other can bital area include primarily trauma and onco-
occur. logical lesions. A thorough knowledge of bony
146 E. A. El Toukhy

landmarks, lymphatic drainage and routes of Interaction with Other Specialities:


perineural invasion is a must for the oculoplastic
1. Recession of the inferior rectus results in:
surgeon.
A. Narrowing of the palpebral fissure
Dermatological disorders can be reflected in
B. Widening of the palpebral fissure
the eye and the periorbital area. Various skin dis-
C. No effect on the palpebral fissure
eases can affect the skin of the periorbital area
D. The effect will depend on the laxity of
e.g. dermatitis, vitiligo, xanthelasma, hidrocys-
the canthal tendons.
tomas, syringomas, milia, etc. Other cutaneous
2. Recession of the superior rectus results in:
disorders may have associated ocular involve-
A. Narrowing of the palpebral fissure
ment e.g. rosacea, port-wine stain and nevus of
B. Widening of the palpebral fissure
Ota. Some infectious diseases can affect both
C. No effect on the palpebral fissure
the skin and eyes e.g. herpes simplex and her-
D. The effect will depend on the laxity of
pes zoster. Systemic diseases can have ocular, as
the canthal tendons.
well as, dermatological involvement e.g. amyloi-
3. Recession of the medial or lateral rectus
dosis, dermatomyositis, sarcoidosis and Bechet’s
results in:
disease. Periorbital dermatological procedures
A. Narrowing of the palpebral fissure
e.g. periorbital chemical peeling and laser proce-
B. Widening of the palpebral fissure
dures, and their expected complications, can be
C. No effect on the palpebral fissure
of interest to the oculoplastic surgeon.
D. The effect will depend on the laxity of
The skin of the eyelid has a similar struc-
the canthal tendons.
ture to the skin elsewhere in the body with some
4. Weakening procedures of the inferior
unique features. The eyelid skin is the thinnest
oblique muscle results in:
in the body particularly the medial aspect of
A. Narrowing of the palpebral fissure in
the upper eyelid. Glands in the eyelids include
upgaze
the sebaceous glands (meibomian glands and
B. Narrowing of the palpebral fissure in
sebaceous gland of Zeis), and both eccrine and
downgaze
apocrine sweat glands (apocrine sweat gland of
C. Widening of the palpebral fissure in
Moll). Terminal hair is also present in the form
upgaze
of eyebrow and eyelashes. Therefore, a wide
D. Widening of the palpebral fissure in
variety of skin conditions that originate from
downgaze.
these skin structures can present in the peri-
5. Ocular complications of craniosynostosis
orbital area. Periorbital skin diseases can be
include all except:
classified into melanocytic and vascular nevi,
A. Exposure keratopathy
neoplastic, infectious, manifestations of sys-
B. Stabismus
temic diseases, nevi and disorders of pigmenta-
C. Proptosis
tion and disorders of eyebrow and eyelashes.
D. Ptosis.
Obtaining an incisional or excisional biopsy
6. Craniosynostosis:
is a day to day procedure in oculoplasty. proper
A. Is usually syndromic
handling of the samples, preservation tech-
B. Is mostly hereditary
niques and a presumed working diagnosis are
C. Usually presents at birth
essential. Eyelid lesions including melanocytic
D. Usually requires simple observation in
lesions and lymphoproliferative orbital lesions
most cases.
are amongst the common oculoplastic speci-
7. Blepharophimosis, all are true except:
mens requiring pathological differentiation.
A. Is mostly hereditary
Immunohistochemistry and tumor markers diag-
B. Usually presents at birth
nosis require special specimen handling.
C. Usually requires simple observation in
most cases
D. Is usually syndromic.
10 Oculoplasty Interactions with Other Specialities 147

8. Blepharophimosis can be associated with: 15. MGD can affect a refractive procedure by
A. Diabetes insipidus causing all except;
B. Early testicular failure A. Tear film disturbance
C. Early ovarian failure B. Exposure keratopathy
D. Cardiac anomalies. C. Block laser pathway
9. In craniosynostosis, the most serious mani- D. Entrapment under the flap.
festation is; 16. MGD can result in all of the following com-
A. Exposure keratopathy plications following LASIK except:
B. Proptosis A. Delayed epithelialization
C. Strabismus B. DLK
D. Raised intracranial pressure. C. Infections
10. Goldenhar syndrome: D. Regression.
A. Is due to an abnormality of chromo- 17. Which procedure produces most affection
some 21 of corneal sensation:
B. Is due to maldevelopment of all A. PRK
branchial arches B. PTK
C. Is mainly ocular C. Lasik
D. Epibulbar dermoids result in amblyopia. D. SMILE.
11. All of the following are syndromes involv- 18. Risk factors for post LASIK dry eyes
ing craniofacial synostosis except: include all except:
A. Crouzon syndrome A. Prior use of contact lenses
B. Treacher-Collins syndrome B. Thicker flap
C. Apert syndrome C. Deeper ablation
D. Pfeiffer syndrome. D. Smaller error of refraction.
12. All of the following clinical findings can 19. The silent sinus syndrome produces:
be associated with Goldenhar syndrome A. Exophthalmos
except: B. Enophthalmos
A. Eyelid colobomas C. Lid retraction
B. Lipodermoids D. Emphysema.
C. Duane syndrome
D. Proptosis.
13. Which systemic condition is incorrectly
paired with a skin lesion?
A. Sturge-Weber syndrome (encephalo-
trigeminal angiomatosis)-nevus flam-
meus (port-wine stain)
B. Ataxia-telangiectasia-cafe-au-lait spots
C. Incontinentia pigmenti-hyperpigmented
macules (“splashed paint”)
D. Tuberous sclerosis-facial angiofibromas
(adenoma sebaceum).
14. Ptosis can affect a refractive procedure by
causing all except:
A. Tear film disturbance
B. Abnormal topography
C. Amblyopia
D. Flap abnormalities.
148 E. A. El Toukhy

20. Regarding the above patient, all are true 22. In the above patient, all are true except;
except: A. The pathology is essentially vascular
A. Patient idiabetic or immunocompromised occlusion
B. It is fungal in origin B. MRI can demonstrate fungal hyphae
C. Spreads rapidly with tissue necrosis C. Early debridement is required
D. Has an excellent prognosis. D. IV antibiotics should be used
21. In the above patient, all are true except immediately.
A. Infection starts in the nose then spreads
B. Spread to the orbit is late
C. Palatal necrosis and perforation is
common
D. Intracranial spread can occur.

23. The above lesion: 24. All are true regarding the above lesion
A. Originates from the sphenoid sinus except:
B. Originates from the frontal and ethmoi- A. Benign in nature
dal sinuses B. Malignant
C. Originates from the lacrimal sac C. Progressive
D. Originates from the nasal cavity. D. Must be removed asap.
10 Oculoplasty Interactions with Other Specialities 149

25. All are true about the above lesion except: C. Acetylcholine esterase agonist
A. It produces frog face deformity D. Acetylcholine esterase inhibitor.
B. It arises from the nose and sinuses 33. Giant cell arteritis involves:
C. It is malignant A. Small-sized arteries
D. It is more common in adolescent males. B. Mid and large-sized arteries
26. The above lesion: C. All types of arteries
A. Is highly vascular D. All types of vessels.
B. Extends through the inferior orbital 34. The most common symptom in giant cell
fissure arteritis is:
C. May require embolization before A. Jaw claudication
excision B. Headache
D. Has a poor prognosis. C. Visual loss
27. Myasthenia gravis is due to: D. Cranial nerve palsy.
A. Presynaptic antibodies against 35. The most specific symptom in giant cell
acetylcholine arteritis is:
B. Postsynaptic antibodies against A. Jaw claudication
acetylcholine B. Headache
C. Presynaptic antibodies against acetyl- C. Visual loss
choline receptors D. Cranial nerve palsy.
D. Postsynaptic antibodies against acetyl- 36. Combined sensitivity of ESR and CRP in
choline receptors. giant cell arteritis is:
28. In myasthenia gravis; the following are nor- A. 70%
mal except: B. 80%
A. Sensory functions C. 90%
B. Pupillary reactions D. 99%.
C. Accommodation 37. The gold standard test for giant cell arteritis
D. Recti muscles. is;
29. All the following tests can be used to diag- A. Combined ESR and CRP
nose myasthenia except: B. Carotid angiography
A. Sleep test C. Temporal artery biopsy
B. Dark test D. Fluorescein angiography.
C. Ice test 38. Horner syndrome includes all except:
D. Fatigue test. A. Ptosis
30. All the following tests can be used to diag- B. Proptosis
nose myasthenia except: C. Miosis
A. Tensilon test D. Anhydrosis.
B. Edrophonium test 39. First order neuron Horner syndrome is
C. Atropine test mostly:
D. EMG testing A. Traumatic
31. Investigations to diagnose myasthenia B. Vascular
include all except: C. Tumor related
A. Serum AChr antibodies D. Postoperative.
B. EMG 40. Second order neuron Horner syndrome can
C. Nystagmography be due to all except:
D. Single fiber EMG. A. Traumatic
32. Mestinon (pyridostigmine) is: B. Vascular
A. Acetylcholine agonist C. Tumor related
B. Acetylcholine inhibtor D. Congenital.
150 E. A. El Toukhy

41. Third order neuron Horner syndrome can be C. 60% of patients


due to all except: D. 70% of patients.
A. Traumatic 49. Oculoplastic side effects of prostaglandin
B. Vascular analogs include all except:
C. Tumor related A. Deepening of the superior sulcus
D. Congenital. B. Eyelid pigmentation
42. The pupil in Horner syndrome exhibits all C. Eyelid retraction
except: D. Eyelash lengthening.
A. Miosis 50. Oculoplastic side effects of ROCK inhibi-
B. Dilation lag tors include:
C. Poor response to Apraconidine A. Conjunctival hemorrhage
D. Poor response to cocaine. B. Eyelid pigmentation
43. Intracranial pressure of above—is needed to C. Blepharitis
diagnose idiopathic intracranial hypertension: D. Eyelash lengthening.
A. 15 mm water 51. Oculoplastic complications of glaucoma
B. 20 mm water surgery can include all except:
C. 25 mm water A. Ptosis
D. 30 mm water. B. Lid retraction
44. Use of all of the following drugs can lead to C. Ectropion
increased intracranial pressure except: D. Dry eyes.
A. Tetracycline 52. What is the least likely ocular complication
B. Vitamin D from endoscopic sinus surgery?
C. Vitamin A A. Diplopia
D. Isotretinoin. B. Blindness
45. Indications for surgery in Idiopathic intrac- C. Tearing
ranial hypertension include all except: D. Ptosis.
A. Male gender 53. What is the most common eyelid condition
B. Bilateral affection associated with the use of topical latanoprost?
C. Progressive field loss A. Depigmentation of the iris and periocu-
D. Central visual loss. lar skin
46. Systemic examination of a patient with B. Eyelid margin necrosis
chronic progressive external ophthalmople- C. Hyperpigmentation of periocular skin
gia is necessary to exclude: and eyelid-margin hyperemia
A. Hepatic disease D. Severe hirsutism of periocular skin.
B. Neurological disease 54. Which nutritional supplement should be
C. Cardiac disease avoided prior to eyelid and orbital surgery
D. Renal disease. because of its effect on platelets?
47. Perineural spread is more common to occur A. Eicosapentaenoic acid (fish oil)
with: B. Echinacea
A. Basal cell carcinoma C. Glucosamine
B. Squamous cell carcinoma D. Valerian.
C. Sebaceous cell carcinoma 55. Which sinus system aerates first?
D. Malignant melanoma. A. Maxillary
48. Topical medications for glaucoma can result B. Frontal
in ocular surface disease in about: C. Sphenoid
A. 40% of patients D. Ethmoid.
B. 50% of patients
10 Oculoplasty Interactions with Other Specialities 151

56. The above lesion is; C. Keratoacanthoma


A. Chalazion D. Hemangioma.
B. Hidrocystoma

57. The above lesion is: C. Sebaceous cell carcinoma


A. Basal cell carcinoma D. Malignant melanoma.
B. Squamous cell carcinoma
152 E. A. El Toukhy

58. The above lesion is: C. Sebaceous cell carcinoma


A. Basal cell carcinoma D. Malignant melanoma.
B. Squamous cell carcinoma

59. The above lesion is: C. Sebaceous cell carcinoma


A. Basal cell carcinoma D. Malignant melanoma.
B. Squamous cell carcinoma
10 Oculoplasty Interactions with Other Specialities 153

60. Satellite lesions occur in: C. Sebaceous cell carcinoma


A. Basal cell carcinoma D. Malignant melanoma.
B. Squamous cell carcinoma

61. The above lesion is: C. Intradermal nevus


A. Pigmented basal cell carcinoma D. Malignant melanoma.
B. Lentigo maligna

62. The above lesion is; C. Keratoacanthoma


A. Chalazion D. Hemangioma.
B. Hidrocystoma
154 E. A. El Toukhy

63. The above lesion is; C. Schwannoma


A. Chalazion D. Molluscum contagiosum.
B. Xanthelasma

64. The above lesion is; C. Schwannoma


A. Chalazion D. Hemangioma.
B. Xanthelasma
10 Oculoplasty Interactions with Other Specialities 155

65. The above lesion is: C. Schwannoma


A. Chalazion D. Hemangioma.
B. Xanthelasma

66. The above lesion is: C. Hidrocystoma


A. Herpes simplex D. Syringoma.
B. Molluscum contagiosum
156 E. A. El Toukhy

67. The above lesion is: C. Multiple skin tags


A. Milia D. Dermatosis papulosa nigra.
B. Rosacea

68. The above lesion is: C. Contact dermatitis


A. Port wine stain D. Atopic dermatitis.
B. Rosacea
10 Oculoplasty Interactions with Other Specialities 157

69. The above lesion is: C. Multiple skin tags


A. Herpes simplex D. Multiple hidrocystomas.
B. Syringomas

70. The above lesion is: 72. Management of eyelid halos include all
A. Tuberous sclerosis except:
B. Xeroderma pigmentosa A. Peeling
C. Icthyosis B. Fillers
D. Dermatosis papulosa nigra. C. Blepharoplasty
71. Eyelid halos ( periorbital hyperpigmenta- D. Botox.
tion), may be due to all except: 73. All the following injections can be used to
A. The shadowing effect treat eyelid halos except:
B. Tear trough depression A. Fillers
C. Genetic susceptibility B. Botox
D. Lack of sleep. C. Platelets–rich plasma
D. Vitamin C.
158 E. A. El Toukhy

74. According to the direction; Epicanthus can 79. The preservative commonly associated with
be all except: OSD is:
A. Tarsalis A. Polyquaternium
B. Lateralis B. Polyvinyl alcohol
C. Inversus C. Benzalkonium chloride
D. Palpebralis. D. Sodium Purite.
75. The CSF in pseudotumor cerebri can show:
Answers of Oculoplasty Interactions with
A. High protein
Other Specialities
B. High cells
C. High ph
1 B 21 B 41 D 61 B
D. High pressure.
76. The surgical procedure of choice in CPEO 2 B 22 D 42 C 62 C
is: 3 B 23 B 43 C 63 D
A. Levator resection 4 A 24 B 44 B 64 B
B. Muller resection 5 D 25 C 45 B 65 C
C. Frontalis sling 6 C 26 D 46 C 66 B
D. Frontalis flap.
7 D 27 D 47 B 67 D
77. The frontalis sling material used in CPEO
8 C 28 D 48 C 68 B
must be:
A. Synthetic 9 D 29 B 49 C 69 B
B. Autologus 10 D 30 C 50 A 70 B
C. Easily removed 11 B 31 C 51 C 71 D
D. Permanent. 12 D 32 D 52 D 72 D
78. A characteristic radiological feature in 13 B 33 B 53 C 73 B
carotid cavernous fistula is: 14 A 34 B 54 A 74 B
A. Extraocular muscle enlargement
15 B 35 A 55 A 75 D
B. Superior ophthalmic vein enlargement
16 D 36 D 56 B 76 C
C. Orbital fat hypertrophy
D. Internal carotid artery aneurysm. 17 C 37 C 57 A 77 C
18 D 38 B 58 D 78 B
19 B 39 B 59 B 79 C
20 D 40 D 60 C
Thyroid Eye Disease
11
Essam A. El Toukhy

Thyroid eye disease (TED) is an autoimmune majority of patients with TED will experience
condition with an active and inactive phase mild disease with the most common presenta-
resulting in proptosis, eyelid retraction, and peri- tion being erythema and eyelid retraction. The
orbital edema of varying severity. Symptoms systemic effects of the disease should be man-
range from mild eye irritation to vision loss from aged with the patient’s primary care doctor or an
compressive optic neuropathy requiring medical endocrinologist.
and possibly acute surgical intervention. Active Pathologically, the key cell involved is the
disease typically lasts one to three years before orbital fibroblast, which has a CD40 marker.
burn-out occurs; reactivation and irreversible This marker allows T cells to bind and upregu-
vision loss is uncommon. An ophthalmologist late the fibroblast’s production of certain inflam-
can be of unique benefit to the patient in three matory markers (IL-6, IL-8, and prostaglandin
valuable ways: (1) early diagnosis and referral E2). This upregulation results in deposition of
for systemic treatment, (2) protection against the hyaluronan and glycosaminoglycans (GAGs)
vision threatening effects of the disease, and (3) to be deposited throughout the orbit and in the
restoration of the patient’s natural appearance. muscles. While this explains the thickening of
TED is the most common cause of unilateral the muscles, the orbital fat enlarges by a differ-
and bilateral proptosis. Women are six times ent mechanism. These various markers are the
more likely than men to have TED, and smok- targets of several treatment modalities, one of
ing is strongly associated with severity and risk which is teprotumumab.
of disease. Age has a first peak early in the third It should be noted that TED is an autoim-
to fourth decade of life and a second peak in the mune disease, not an endocrinal one. Testing
mid-60s. The course usually follows the typi- only for thyroid hormones is non-conclusive
cal one described by Rundle: early progression, and can be misleading. Testing for antibodies is
peak of inflammation at 6–24 months, followed more useful and is mandatory.
by an inactive phase. Only 5–10% of patients The clinical activity score (CAS) is a series
have reactivation. Patients have eye irritation, of symptoms including pain in the orbit, pain
edema, and finally proptosis from thickening with eye movements, redness of eyelids or con-
of the extraocular muscles or orbital fat. The junctiva, impaired movement or vision, and
swelling of the eyelids, conjunctiva, caruncle
or orbit (increasing proptosis). Each is given a
E. A. El Toukhy (*) point that when added together correlates with
Oculoplasty Service, Cairo University, Cairo, Egypt responsiveness to corticosteroids: the higher the
e-mail: eeltoukhy@yahoo.com

© The Author(s), under exclusive license to Springer Nature Switzerland AG 2021 159
E. A. El Toukhy (ed.), Oculoplasty for Ophthalmologists, https://doi.org/10.1007/978-3-030-68469-3_11
160 E. A. El Toukhy

score, the more likely symptoms will improve Thyroid Eye Diseases
with medications.
1. Regarding clinical features of thyroid eye
All patients with TED benefit from main-
disease, one is false:
tenance in a euthyroid state. Coordinated care
A. Lid retraction is the most common sign
with an endocrinologist should be maintained
B. More likely asymmetric
throughout the patient’s course. Concomitant
C. More common in females
steroids should be administered to all patients
D. Severity of the disease parallel serum
with TED that undergo radioactive iodine abla-
level of T4 or T3.
tion (RAI). Similarly, patients with active dis-
2. Poor prognosis for orbitopathy in thyroid
ease, both moderate to severe disease and
eye disease is associated with, one is false:
sight-threatening disease can be treated with oral
A. Old male patient
or IV corticosteroids. Typically, a high dose of
B. Perorbital myxedema
1mg/kg until resolution followed by slow taper.
C. Acropachy
To avoid systemic steroids, or if the inflam-
D. Myasthenia gravis.
matory process does not resolve or returns when
3. Regarding treatment options for lid retrac-
the oral corticosteroids are finished, intraorbital
tion in thyroid eye disease, one is false:
intermediate acting steroids (Triamcinalone) or
A. Mild lid retraction often resolve
long-acting steroids (Dexamethasone) have been
spontaneously
used successfully as an alternative to the sys-
B. Six months of disease stability should
temic steroids for reduced morbidity.
be passed before surgical intervention
Most recently, teprotumumab is a human
is indicated
insulin-like growth factor I (IGF-I) receptor
C. Lid splitting, lateral tarsorrhaphy with
inhibitory monoclonal antibody. Patients who
recession of lid retractors is indicated if
were treated had significant reduction in prop-
the patient have lateral flare
tosis and CAS score as well as improvement
D. Spacer graft is of no value in treatment
in quality of life at 6 months and a response as
of lower lid retraction.
soon as 6 weeks without clinically significant
4. Regarding orbital decompression in thyroid
side effects.
ophthalmopathy the most appropriate state-
Vision loss or threatened vision loss is treated
ment is:
with decompression surgery. Up to three walls
A. It is indicated when radiological evidence
may be decompressed: the lateral wall, the floor,
of swollen extraocular muscles is present
and the medial wall.
B. It allows the swollen extraocular mus-
If patients have diplopia after reaching the
cles to expand into periorbital space
chronic phase or after decompression, this
C. Orbital floor decompression can exac-
should be repaired prior to eyelid surgery.
erbate lagophthalmos
Inferior rectus recession can create or exac-
D. Removal of orbital fat during decom-
erbate lower eyelid retraction. Lid restorative
pression exacerbate lid ptosis.
surgery to reconstruct their natural look should
5. In the surgical treatment of upper lid retrac-
be undertaken afterwards. Common surgeries
tion due to thyroid associated ophthalmopa-
include recession of the upper and lower eye-
thy, one of the following statements is correct:
lids, mostly transconjunctivally. Blepharoplasty
A. Contraindicated in exposure keratopathy
can benefit the patient both in terms of derma-
B. Evidence of disease stability must be
tochalasis improvement and to facilitate dissec-
documented prior to surgery in the
tion superiorly via a lid crease incision to debulk
presence of exposure keratopathy
some of the enlarged or thickened eyebrow fat
C. Can be corrected with excision of Müller
pad inferior to the eyebrow cilia. The lower eye-
muscle
lid retraction can be repaired with or without a
D. Lateral tarsorrhapy is the surgical
spacer to elevate the eyelid.
modality of choice.
11 Thyroid Eye Disease 161

6. In a patient with Grave’s ophthalmopathy C. Strabismus surgery, then eyelid surgery,


and strabismus, how long should the angle then orbital wall decompression
of deviation be stable before performing the D. Eyelid surgery, then orbital wall decom-
surgical correction? pression, then strabismus surgery.
A. 1 month 11. In thyroid eye disease:
B. 3 months A. The medial and inferior recti are the
C. 6 months most commonly affected extraocular
D. 1 year. muscles
7. A patient who is a known case of Diabetes B. Optic nerve fenestration is useful in
type 1 and hypertension presents with his- patients with optic nerve compression
tory of decreased vision with abnormal visual C. Downgaze is most commonly affected
field, and RAPD in OD. His blood work up in patients with ocular motility problems
shows marked hyperthyroid status, high blood D. The thyroid function test always shows
sugar level of 15 mmol/L and normal renal hyperthyroidism.
function. General anaesthesia is felt to be 12. A patient with congestive thyroid orbit-
risky as the patient is on the brink of a thyroid opathy had been taking 60 mg daily oral
storm. What would be the most appropriate prednisone for 9 months. The dosage was
management modality for vision-threatening tapered to 0 mg over a week. The patient
thyroid ophthalmopathy in this case? now reports nausea, vomiting, muscle aches
A. Intravenous corticosteroid and pain. Initial considerations include
B. Lateral tarsorrhaphy which of the following?
C. Orbital radiation A. Acute adrenal insufficiency
D. Oral corticosteroid. B. Thyroid storm
8. Regarding orbital decompression in thyroid C. Acute thyroid insufficiency
eye disease, one is false: D. Steroid induced diabetes.
A. Indicated to restore globe position 13. Regarding thyroid eye disease, the incorrect
even if there is no sight threatening statement is;
conditions A. 20% of patients with thyroid eye dis-
B. Indicated if radiotherapy was not ease will need surgical treatment
effective B. Radiation therapy should be avoided in
C. Fat versus bone decompression is based patients with diabetes
on patient’s age C. Orbital decompression should be done
D. Orbital surgery should precede strabis- after strabismus surgery
mus surgery. D. Smokers appear to be at greater risk for
9. During decompression of the orbital floor, exacerbation of eye disease after radio-
diplopia and dystopia can be minimized by active iodine therapy.
preserving: 14. The most common cause of unilateral prop-
A. The palatine bone tosis in adults is;
B. The bone between the medial wall and A. Lymphoma
the floor B. Cavernous hemangioma
C. The zygomatic bone C. Thyroid eye disease
D. The ethmoidal bone. D. Meningioma.
10. The correct order of surgical procedures in 15. A 56 years old female with Grave’s oph-
thyroid-associated orbitopathy is: thalmopathy, on orbital CT most likely to be
A. Orbital wall decompression, then eyelid found is;
surgery, then strabismus surgery A. An increased amount of orbital fat in
B. Orbital wall decompression, then stra- the presence of normal size extra ocular
bismus surgery, then eyelid surgery muscles
162 E. A. El Toukhy

B. Diffuse fusiform enlargement of extra 19. First muscle to be involved in thyroid oph-
ocular muscle belly and tendon. thalmopathy is:
C. Pressure erosion of lateral orbital rim A. Medial rectus
from enlarged muscles B. Inferior rectus
D. Chronic ethmoidal and maxillary C. Lateral rectus
sinusitis. D. Superior rectus.
16. Orbital decompression in case of dysthyroid 20. Thyroid ophthalmopathy: All of the follow-
ophthalmopathy is indicated in the follow- ing are treatment modalities except:
ing except; A. Radiation
A. Severe proptosis B. Steroids
B. Optic neuropathy C. B-Blockers
C. Early active phase D. Orbital decompression.
D. Glucocorticoid side effects. 21. Which is a pathognomonic CT Finding in
17. Features of thyroid ophthalmopathy include Thyroid ophthalmopathy?
all except: A. Kinking of extraocular muscles
A. External ophthalmoplegia B. Nodular muscle enlargement
B. Internal ophthalmoplegia C. Fusiform muscle enlargement with
C. Enlargement of extraocular muscles sparing of tendons
D. Lid lag. D. Solitary muscle enlargement.
18. Dalrymple sign is seen in: 22. Which is the commonest eyelid finding in
A. Thyroid ophthalmopathy Thyroid ophthalmopathy?
B. Cavernous sinus thrombosis A. Lid lag
C. Orbital cellulitis B. Lagophthalmos
D. Cavernous haemangioma. C. Lid retaction
D. Von Graefe’s sign.
11 Thyroid Eye Disease 163

23. A 55-year-old white man presents A. Thyroid-related orbitopathy


with bilateral proptosis, double vision, B. Orbital cellulitis
and chemosis. What is the most likely C. Lymphangioma
diagnosis? D. Meningioma.

24. A CT is performed on this patient with thy- C. Fusiform extraocular muscle involvement
roid orbitopathy. Which feature, as demun- D. Bilateral extraocular muscle involvement.
strated by CT, helps to clarify that this 26. What are the 2 most commonly affected
process is more likely thyroid-related orbit- rectus muscles in thyroid eye disease?
opathy than orbital inflammatory syndrome? A. Superior and inferior
A. Enlarged extraocular muscle B. Superior and medial
B. Absence of a thickened tendon of the C. Medial and lateral
extraocular muscle insertion D. Inferior and medial.
C. Enlarged lacrimal glands 27. Which condition is closely associated with
D. Periorbital soft-tissue edema of the lids. thyroid eye disease?
25. Which of the following CT findings is A. Eczematous eyelid
not commonly seen with thyroid-related B. Parinaud’s syndrome
orbitopathy? C. Myotonic dystrophy
A. Sparing of extraocular muscle tendons D. Myasthenia gravis.
B. Involvement of extraocular muscle
tendons
164 E. A. El Toukhy

28. The patient is looking down, this is called:


C. Lid retraction
A. Lagophthalmos
D. Von Graefe’s sign.
B. Lid lag
11 Thyroid Eye Disease 165

29. This Thyroid patient requires: C. Blepharoplasty


A. Lid retraction surgery D. Spacer.
B. Strabismus surgery
166 E. A. El Toukhy

30. This patient requires: C. Blepharoplasty


A. Lid retraction surgery D. Spacer.
B. Strabismus surgery
11 Thyroid Eye Disease 167

31. In thyroid ophthalmopathy, this is a compli- B. Corneal exposure


cation of all except: C. Optic nerve compression
A. Severe proptosis D. Improper Streroid therapy.
168 E. A. El Toukhy

32. This Thyroid patient requires: C. Blepharoplasty


A. Lid retraction surgery D. IV steroid therapy.
B. Decompression surgery
11 Thyroid Eye Disease 169

33. This young patient will most likely benefit B. IV steroids


from; C. Intraorbital steroids
A. Decompression D. Radiation.

34. This patient will benefit from: C. Blepharoplasty


A. Lid retraction surgery D. IV steroid therapy.
B. Decompression surgery
170 E. A. El Toukhy

35. This appearance in a thyroid patient suggests; C. Steroid resistance


A. Noncompliance with medication D. Severe levator muscle affection.
B. Myasthenia gravis

36. This is a complication of: C. B-blockers


A. Inferior wall decompression D. Fillers.
B. Medial wall decompression 38. Promising results for the treatment of thy-
C. Two wall decompression roid ophthalmopathy has been achieved
D. Balanced decompression. with the use of:
37. All the following injections can be used in A. Ranibizumab
Thyroid ophthalmopathy except: B. Bevacizumab
A. Botulinium toxin C. Teprotumumab
B. Steroids D. Rituximab.
11 Thyroid Eye Disease 171

39. This complication occurs due to: 40. Rundle’s curve in thyroid ophthalmopathy
A. Proptosis describes:
B. Lid retraction A. Disease pathology
C. Proptosis and lid retraction B. Disease activity
D. Increased orbital pressure. C. Disease prognosis
D. Disease management.
172 E. A. El Toukhy

41. Management of this patient requires: C. Blepharoplasty


A. Lid retraction surgery D. IV steroid therapy.
B. Decompression surgery

42. This patient has:


A. Type 1 thyroid ophthalmopathy
B. Type 2 thyroid ophthalmopathy
C. A CAS score of 5
D. Mild disease by the EUGOGO scale.
11 Thyroid Eye Disease 173

43. This patient has: B. Transconjunctival approach


A. Type 1 thyroid ophthalmopathy C. Transantral approach
B. Type 2 thyroid ophthalmopathy D. Transnasal approach.
C. A CAS score of 5 48. Medial decompression can be done through
D. Mild disease by the EUGOGO scale. all except:
44. The CAS score has: A. Transcutaneous approach
A. 5 points B. Transantral approach
B. 7 points C. Transcaruncular approach
C. 8 points D. Transnasal approach.
D. 10 points. 49. Lateral decompression can be done through:
45. The EUGOGO scale has: A. Transcutaneous approach
A. 2 grades B. Transconjunctival approach
B. 3 grades C. Transcaruncular approach
C. 4 grades D. Transnasal approach.
D. 5 grades. 50. Advantages of intraorbital steroids in the
46. Balanaced decompression means removal active phase of thyroid ophthalmopathy
of: include all except:
A. Parts of medial wall and inferior wall A. No or lower doses of systemic steroids
B. Parts of medial wall and lateral wall B. Single injection is sufficient
C. Parts of lateral wall and inferior wall C. Improves lid retraction
D. Equal parts of all three walls. D. Reduces incidence of recurrence.
47. Inferior decompression can be done through 51. Corticosteroids are used in treatment of all
all except: except;
A. Transcutaneous approach A. Orbital lymphoma
174 E. A. El Toukhy

B. Thyroid eye disease Answers for Thyroid Eye Disease


C. Orbital mucocele
D. Non specific orbital inflammation. 1 D 16 C 31 C 46 B
52. Limitation of ocular motility occurs in all 2 D 17 B 32 B 47 D
except
3 D 18 A 33 C 48 B
A. Orbital rim fracture
4 B 19 B 34 A 49 A
B. Cavernous sinus thrombosis
C. Orbital cellulitis 5 C 20 C 35 B 50 B
D. Thyroid orbitopathy. 6 C 21 C 36 A 51 C
53. Which of the following is the recommended 7 A 22 C 37 C 52 A
management of dysthyroid optic neuropathy 8 B 23 A 38 C 53 A
(DON)? 9 B 24 B 39 C 54 B
A. Intravenous steroid 10 B 25 B 40 B
B. Oral steroid
11 A 26 D 41 B
C. Radiotherapy
12 A 27 D 42 A
D. Selenium.
54. Which of the following is the best indicator 13 C 28 B 43 B
of activity in thyroid eye disease? 14 C 29 C 44 D
A. Diplopia 15 A 30 A 45 B
B. Upper lid edema
C. Pain
D. Reduced color vision.

You might also like